You are on page 1of 86

PRELIMS

TEST SERIES 2023


PT MAXIMA
TEST: 11
CURRENT AFFAIR
(July – August 2022)

8448496262 iasscore.in
DO NOT OPEN THIS TEST BOOKLET UNTIL YOU ARE ASKED TO DO SO

PTS2023/PPP-01/092022/11

GENERAL STUDIES Test No.

CURRENT AFFAIRS - 2
(July-August, 2022) 11
Time Allowed: Two Hour Maximum Marks: 200

INSTRUCTIONS

RE
1. IMMEDIATELY AFTER THE COMMENCEMENT OF THE EXAMINATION, YOU SHOULD CHECK THAT
THIS TEST BOOKLET DOES NOT HAVE ANY UNPRINTED OR TORN OR MISSING PAGES OR ITEMS,
ETC. IF SO, GET IT REPLACED BY A COMPLETE TEST BOOKLET.
2. Please note that it is the candidate’s responsibility to encode and fill in the Roll Number carefully
without any omission or discrepancy at the appropriate places in the OMR Answer Sheet. Any
omission/discrepancy will render the Answer Sheet liable for rejection.
O
3. You have to enter your Roll Number on the test booklet in the Box provided
alongside. DO NOT write anything else on the Test Booklet.
4. This Test Booklet contains 100 items (questions). Each item is printed in English. Each item comprises four
SC
responses (answers). You will select the response which you want to mark on the Answer Sheet. In case you
feel that there is more than one correct response, mark the response which you consider the best. In any case,
choose ONLY ONE response for each item.
5. You have to mark all your responses ONLY on the separate Answer Sheet provided. See directions in the
Answer Sheet.
6. All items carry equal marks.
GS

7. Before you proceed to mark in the Answer Sheet in response to various items in the Test Booklet, you have to
fill in some particulars in the Answer Sheet as per instructions sent to you with your Admission Certificate.
8. After you have completed filling in all your responses on the Answer Sheet and the examination has concluded,
you should hand over to the Invigilator only the Answer Sheet. You are permitted to take away with you the
Test Booklet.
9. Sheets for rough work are appended in the Test Booklet at the end.
10. Penalty for wrong answers:
THERE WILL BE PENALTY FOR WRONG ANSWERS MARKED BY A CANDIDATE IN THE OBJECTIVE
TYPE QUESTION PAPERS.
(i) There are four alternatives for the answer to every question. For each question for which a wrong answer
has been given by the candidate, one-third of the marks assigned to that question will be deducted as
penalty.
(ii) If a candidate gives more than one answer, it will be treated as a wrong answer even if one of the given
answers happens to be correct and there will be same penalty as above to that question.
(iii) If a question is left blank, i.e., no answer is given by the candidate, there will be no penalty for that
question.

DO NOT OPEN THIS TEST BOOKLET UNTIL YOU ARE ASKED TO DO SO


1 PTS2023/PPP-01/092022/11

PTS2023/PPP-01/092022/11
1. With reference to the Plastic Waste of unmanned aerial combat vehicles
Management (Amendment) Rules, 2021, (UCAV).
consider the following statements:
Which of the above statements is/are correct?
1. They prohibit manufacture, import and
(a) 1 only
sale of single-use plastics including
the commodities made of compostable (b) 2 only
plastics.
(c) Both 1 and 2
2. They provide for increased thickness
of plastic carry bags in order to stop (d) Neither 1 nor 2
littering.
3. They aim to set-up a market mechanism 4. With reference to election of the Vice-President
for plastic waste management. of India, consider the following statements:
1. The Electoral College to elect a person
Which of the above statement(s) is/are
to the office of the Vice-President
correct?
consists of all members of both Houses of
(a) 1 only Parliament.
(b) 1 and 2 only 2. An election to fill a vacancy in the office

RE
of Vice-President shall be held within six
(c) 2 and 3 only
months from the date of occurrence of
(d) 3 only vacancy.
3. The Secretary-General of Rajya Sabha
2. Which of the following statement(s) regarding is usually appointed as the Returning
the Code on Wages, 2019 is/are correct? Officer to conduct the Vice-Presidential
elections.
1. According to the Code, the central
O
government may set different floor wages 4. A petition challenging the election of the
for different geographical areas. Vice-President is heard by a five-judge
2. The Code prohibits employers from bench of the Supreme Court of India.
SC
paying wages less than the minimum Which of the above statements are correct?
wage, which must be higher than the
floor wage. (a) 1 and 4 only

3. Employees working in excess of a normal (b) 2 and 3 only


working day are entitled to an overtime (c) 1, 3 and 4 only
wage, which must be at least twice the
normal rate of wages. (d) 2, 3 and 4 only
GS

Select the correct answer using the code given


5. With reference to chemical fertilisers in India,
below:
consider the following statements:
(a) 1 only 1. At present, the retail price of chemical
(b) 1 and 2 only fertilizers is market-driven and not
administered by the Government.
(c) 3 only
2. All Non-Urea based fertilisers are
(d) 1, 2 and 3 regulated under Nutrient Based Subsidy
Scheme.
3. Defence Research and Development 3. Supply disruptions due to sanctions on
Organisation (DRDO) recently carried out the Russia and export restrictions in China
maiden test flight of a Flying Wing Technology are the major reasons for the current rise
Demonstrator. In this context, consider the in fertiliser prices in India.
following statements:
Which of the above statements are correct?
1. The flying wing type of aircraft refers to a
tailless fixed-wing aircraft which houses (a) 1 and 2 only
its payload and fuel in its main wings
(b) 2 and 3 only
unlike the conventional aircrafts.
(c) 1 and 3 only
2. The Flying Wing Technology
Demonstrator aims help in development (d) 1, 2 and 3

PTS2023/PPP-01/092022/11 2
.
6. With reference to Indian history, who among 1. Reduced plant density
the following were the founders of the South 2. Reduced seed requirement
Indian Liberal Federation?
3. Reduced water application
1. Sir Pitti Theagaraya Chetty
4. Reduced labor costs
2. Dr. C Natesa Mudaliar
3. E.V. Ramaswamy Select the correct answer using the code given
below:
4. G. Subramania Iyer
(a) 1, 2 and 3 only
Select the correct answer using the code given
below: (b) 2, 3 and 4 only

(a) 1 and 2 only (c) 1 and 4 only


(b) 1, 2 and 4 only (d) 1, 2, 3 and 4
(c) 3 and 4 only
10. “Nord Stream” is a pair of offshore natural gas
(d) 1, 2, 3 and 4 pipelines that connects Russia and Germany
via:
7. Which of the following factors can lead to

RE
(a) Baltic Sea
emergence of zoonotic diseases?
(b) Black Sea
1. Intensive farming
2. Travel and transportation (c) Barents Sea
3. Unsustainable utilization of natural (d) Bothnian Sea
resources
4. Increased demand for animal protein 11. With reference to SARFAESI Act, 2022,

below:
O
Select the correct answer using the code given
consider the following statements:
1. Unlike Insolvency and Bankruptcy code,
the SARFAESI Act, 2022 covers only
(a) 1, 3 and 4 only secured financial creditors.
SC
(b) 3 and 4 only 2. Co-operative banks cannot invoke
(c) 1 and 2 only SARFAESI Act.
(d) 1, 2, 3 and 4 3. The act covers all assets, movable or
immovable, promised as security to the
lender.
8. Consider the following statements:
Which of the above statements are correct?
1. The constitution empowers the
GS

Parliament to appoint a Commission to (a) 1 and 2 only


investigate the conditions of socially and
(b) 1 and 3 only
educationally backward classes.
2. At present, there is no sub-categorisation (c) 2 and 3 only
within the 27% reservation for socially (d) 1, 2 and 3
and educationally backward classes in
jobs and education under the central
government. 12. What is ‘XpoSat’, sometimes seen in news?

Which of the above statement(s) is/are (a) It is a ISRO planned space observatory to
correct? study polarisation of cosmic X-rays.

(a) 1 only (b) It is a test vehicle for the first abort


demonstration for ISRO’s Gaganyaan
(b) 2 only mission.
(c) Both 1 and 2 (c) It is ISRO’s space docking experiment
(d) Neither 1 nor 2 which will be used for setting up modular
space stations.
9. Which of the following are the advantages (d) It is an Indian spacecraft going to the
of the System of Rice Intensification (SRI) L1 or Lagrangian point between the Sun
method of paddy cultivation? and Earth.

3 PTS2023/PPP-01/092022/11

Telegram - @upscmaterialonline1
.
13. Consider the following statements: (a) The Mediation council formed works
1. Under the new semiconductor policy, the with a very flexible process that can be
government will provide up to 50% of effectively used at the course of dispute.
Project cost for two semiconductor and (b) It allows for court annexed mediation.
two display fabrication units.
(c) Pre-litigation Mediation is mandatory
2. C-DAC (Centre for Development of for the litigants before presenting case
Advanced Computing) is the nodal
before the courts.
agency for implementation of the India
Semiconductor Mission (ISM). (d) The Higher courts can frame rules for the
sub-ordinate courts.
Which of the above statements is/are correct?
(a) 1 only
17. With reference to Forest Conservation Act,
(b) 2 only 1980, consider the following statements:
(c) Both 1 and 2 1. It gives to the Centre, the freedom to
define land as forest, not already classified
(d) Neither 1 nor 2
as forest in the Central or state records.
2. It puts restrictions on states for use of

RE
14. With reference to cloudbursts, consider the
forest land for non-forest purpose.
following statements:
1. If 10 cm rainfall is received at a station 3. It defines the concept of deemed forests
in one hour, the rain event is termed as in India.
cloud burst. Which of the above statements is/are
2. During a cloudburst, the relative humidity incorrect?
and cloud cover is at the maximum level
with low temperature and slow winds.
O (a) 1 and 2 only
(b) 1 and 3 only
3. Cloud bursts do not occur at plain areas.
(c) 2 only
Which of the above statements is/are correct?
SC
(d) 3 only
(a) 1 only
(b) 1 and 2 only
18. Regarding the Information Technology Act
(c) 2 and 3 only 2000, consider the following statements:
(d) 1, 2 and 3 1. Section 69A of the act mentions the
provision for jail up to 7 years under any
15. With reference to ‘derechos’, consider the online offence violating sovereignty or
GS

following statements: integrity of India.


1. Derechos are wide-spread and long- 2. The Government can issue Blocking
lived windstorms that are similar to a Rules against any individual under the
tornado. act.
2. These are a warm-weather phenomenon. Which of the statements given above is/are
3. These are limited to central and eastern correct?
parts of the United States. (a) 1 only
Which of the above statements is/are (b) 2 only
incorrect?
(c) Both 1 and 2
(a) 1 and 2 only
(d) Neither 1 nor 2
(b) 2 only
(c) 3 only 19. What do you understand by the term
(d) 1 and 3 only Shrinkflation seen in news recently?
(a) Shrinkflation allows companies to
16. Which of the following statements is incorrect increase their operating margin and
regarding Mediation Bill, recently put forward profitability by reducing costs whilst
by the parliamentary committee? maintaining sales volume.

PTS2023/PPP-01/092022/11 4

Telegram - @upscmaterialonline1
.
(b) It is a term which is used to define the Which of the above statement(s) is/are
economic situation when Inflation causes correct?
the rise in prices and consequently the
(a) 1 only
employment decreases.
(b) 2 only
(c) Shrinking economy with low foreign
reserves and High debt is known as (c) Both 1 and 2
Shrinkflation.
(d) Neither 1 nor 2
(d) The Consumer protection is considered
under Shrinkflation for welfare society 23. With reference to the Kerala Fibre Optic
and making goods prices less than the Network (KFON), consider the following
average price of the commodity. statements:
1. It aims to provide Internet connections
20. With reference to sources of direct information only to below-poverty-line (BPL)
about the Dark matter, which of the following
households in the state.
statements is/are correct?
2. Under the project, KFON Ltd. will offer
1. Chandra X-ray Observatory
its own Internet service rather than

RE
2. Big-Bang Theory leasing the KFON network to Internet
3. Motion of galaxies in Coma clusters Service Providers (ISPs).
4. Newton’s law of motion 3. The project will augment the launch of
5G services in the state.
Select the correct answer from the code given
below: Which of the above statement(s) is/are
incorrect?
(a)
(b)
1 and 3 only
2 and 4 only
O (a)
(b)
1 and 2 only
2 only
(c) 1 and 2 only
SC
(c) 1 and 3 only
(d) 3 only
(d) 3 only
21. Which of the following statements is not true
regarding the EU taxonomy, seen in news 24. Consider the following:
recently? 1. Russell’s Viper
(a) It is an agency which classifies the 2. Reticulated Pythons
investment projects as sustainable for
GS

3. Pit Viper
the environment.
4. Common Krait
(b) It provides ranking to the industries
as a clearance allowance for better Which of the following Snake species is
performance. endemic to India?
(c) The Nuclear power plants and Oil (a) 1 and 3 only
refineries were also considered
(b) 2 and 4 only
sustainable.
(c) 1 and 4 only
(d) None of the above.
(d) 2 and 3 only
22. With reference to ‘Right to be forgotten’,
consider the following statements: 25. Consider the following statement regarding
Mission Vatsalya launched recently:
1. The Supreme Court of India has
recognized the right to be forgotten as a 1. It is launched as a mission mode
fundamental right under Article 21. programme to focus on Child Protection
2. India, at present does not have any Services.
statutory provision that provides for 2. It is a renamed version of the scheme
right to be forgotten. Child Protection services.

5 PTS2023/PPP-01/092022/11

Telegram - @upscmaterialonline1
.
Which of the statements given above is/are 3. It is a centrally sponsored scheme divided
correct? in two components.
(a) 1 only Select the correct answer from the code given
below:
(b) 2 only
(a) 1 and 3 only
(c) Both 1 and 2
(b) 2 and 3 only
(d) Neither 1 nor 2
(c) 1 and 2 only
26. Arrange the following core industries in (d) 1, 2 and 3
increasing order of their weightage in the
Index of Eight Core Industries? 29. Consider the following statements:
1. Petroleum refinery 1. The writ of ‘certiorari’ is issued by a
2. Natural Gas higher court to a lower court or tribunal
to prevent the latter from exceeding its
3. Steel
jurisdiction or usurping a jurisdiction
4. Electricity that it does not possess.

RE
Select the correct answer using the code given 2. The writ of ‘prohibition’ can be issued
below: only against judicial and quasi-judicial
authorities.
(a) 1-4-3-2
Which of the above statements is/are correct?
(b) 2-3-4-1
(a) 1 only
(c) 3-2-1-4
(b) 2 only
(d) 4-1-2-3
O
(c) Both 1 and 2
27. Which of the following statement regarding the (d) Neither 1 nor 2
National Institutional Ranking Framework
SC
(NIRF) is correct? 30. Which of the following statement(s) regarding
1. The NIRF outlines a methodology to rank the ‘I2U2 group’ is/are correct?
higher education institutions across the 1. It is also known as ‘West Asian Quad’ as
country. it comprises of four west Asian nations.
2. National Board of Accreditation (NBA) is 2. It aims to strengthen the economic
the agency to rank Institutions. partnership in trade and investment in
the west Asian region.
GS

3. Participation in the rankings is currently


voluntary. 3. India participates as an observer in the
I2U2 group.
Select the correct answer from the code given
below: Select the correct answer using the code given
below:
(a) 1 and 2 only
(a) 1 and 2 only
(b) 2 only
(b) 2 only
(c) 1, 2 and 3
(c) 2 and 3 only
(d) 1 only
(d) 1 and 3 only
28. With reference to the Integrated Development
of Horticulture (MIDH) Scheme, consider the 31. Which of the following statements is correct
following statements: about ‘Pokkali’, seen in news recently?
1. Under the scheme, Local vegetables 1. Pokkali is a variety of rice endemic to
and fruits are promoted except exotic coastal Tamil Nadu.
species. 2. It is salt resistant variety of rice, grown
2. MIDH is implemented as a part of Green twice a year.
Revolution. 3. It got its GI tag in the year 2008.

PTS2023/PPP-01/092022/11 6

Telegram - @upscmaterialonline1
.
Select the correct answer from the code given 35. Which of the following Indian states has Red
below: Panda population, listed Endangered under
the IUCN?
(a) 1 only
1. West Bengal
(b) 2 and 3 only
2. Sikkim
(c) 1, 2 and 3
3. Meghalaya
(d) 2 only 4. Odhisa
Select the correct answer from the code given
32. For the purpose of Women Safety, Delhi below:
Police has organized an initiative to train
girls, working women and housewives by their (a) Only one
special police unit. Which of the following is (b) Only two
the name of the initiative?
(c) Only Three
(a) Sashakti
(d) All four are correct.
(b) NariShakti

RE
(c) NIRBHAYA 36. Which of the following adaptations help
mangroves survive in salinity stress and
(d) None of the above water-logged anaerobic conditions?
1. Viviparity mode of reproduction
33. Consider the following statements regarding
‘Ecological Sensitive Zone (ESZ)’: 2. Areal roots

1. The term ‘ESZ’ is defined in the Wild Life


O 3. Thin leaves to excrete more salt
(Protection) Act of 1972. 4. Foliar water uptake
2. An ESZ could go up to 50 kilometres 5. High nutrient recycling
around a protected area. Select the correct answer using the code given
below:
SC
3. Only Central government can declare a
region to be an ESZ. (a) 1, 2 and 3 only
Select the correct answer from the code given (b) 1, 2, 4 and 5 only
below:
(c) 3, 4 and 5 only
(a) 1 only
(d) 1, 2, 3, 4 and 5
(b) 2 and 3 only
GS

(c) 3 only 37. With reference to the criterion for declaring


heat wave in India, consider the following
(d) 1, 2 and 3
statements:
1. Heat wave is considered if maximum
34. Consider the following statements:
temperature of a station reaches at least
1. It is possible to observe different life 35°C or more for Plains and at least 30°C
stages of galaxies and stars by observing or more for Hilly regions.
light emitted by the galaxies in time 2. For coastal stations, when maximum
space. temperature departure is 4.5°C or more
2. The present James Webb Telescope can from normal, heat wave may be described
reveal the birth of stars and planets. provided actual maximum temperature
is 37°C or more.
Which of the statement(s) given above is/are
correct? 3. When temperature departure from
normal is >6.4°C, it is called a severe
(a) 1 only heat wave.
(b) 2 only Which of the above statements are correct?
(c) Both 1 and 2 (a) 1 and 2 only
(d) Neither 1 nor2 (b) 1 and 3 only

7 PTS2023/PPP-01/092022/11

Telegram - @upscmaterialonline1
.
(c) 2 and 3 only (c) 1 and 2 only
(d) 1, 2 and 3 (d) 1, 2 and 3

38. Consider the following statements regarding 41. Which of the following are the key provisions
Central Vista Redevelopment Project: of the draft Drugs, Medical Devices and
Cosmetics Bill, 2022?
1. It envisages demolishing the existing
Parliament building and constructing a 1. Complete prohibition on operation of
new Parliament and Secretariat in its online pharmacies.
place. 2. Regulation of Sowa Rigpa and
2. It is proposed by the Union Ministry of Homoeopathy system of medicines.
Parliamentary Affairs. 3. Regulation of medical devices as a
Which of the above statement(s) is/are separate entity.
correct? 4. Provision for compensation to participants
for injury or death suffered in clinical
(a) 1 only
trials.
(b) 2 only
Select the correct answer using the code given

RE
(c) Both 1 and 2 below:
(d) Neither 1 nor 2 (a) 1, 2 and 3 only
(b) 1, 3 and 4 only
39. Consider the following statements regarding
(c) 1, 2 and 4 only
the International Space Station (ISS):
(d) 2, 3 and 4 only
1. ISS is the first and the only operational
space laboratory as of now.
O
42. Consider the following statements regarding
2. Astronauts working and living on the
Public Health Emergency of International
ISS experience more than 10 sunrises
Concern (PHEIC):
and sunsets each day.
SC
1. As per the International Health
3. Blue Origin’s ‘New Shephard’ became
Regulations (IHR), only epidemic-
the first privately-owned spacecraft to
prone diseases that actually spread
transport people to the ISS.
internationally can be declared a
Which of the above statements is/are correct? PHEIC.
(a) 1 and 2 only 2. Determining the existence of a PHEIC is
the prerogative of the Director-General
(b) 2 only of WHO.
GS

(c) 1 and 3 only 3. PHEIC is highest alert level by the World


(d) 3 only Health Organisation (WHO).
Which of the above statements is/are correct?
40. Consider the following statements regarding (a) 1 only
Intergovernmental Science-Policy Platform on
Biodiversity and Ecosystem Services (IPBES): (b) 1 and 2 only

1. It aims to strengthen the science-policy (c) 2 and 3 only


interface for biodiversity and ecosystem (d) 3 only
services.
2. It is a United Nations (UN) Body. 43. Under the Universal Immunization Programme
3. The United Nations Environment (UIP), immunization is provided at national
Programme (UNEP) provides secretariat against which of the following diseases?
services to IPBES. 1. Pneumococcal Pneumonia
Which of the above statement(s) is/are 2. Japanese Encephalitis
incorrect? 3. Pertussis
(a) 1 and 3 only 4. Meningitis
(b) 2 only 5. Measles

PTS2023/PPP-01/092022/11 8

Telegram - @upscmaterialonline1
.
Select the correct answer using the code given Which of the above statements is/are correct?
below:
(a) 1 only
(a) 1, 2 and 3 only
(b) 2 only
(b) 1, 2 and 5 only (c) Both 1 and 2
(c) 3, 4 and 5 only (d) Neither 1 nor 2
(d) 1, 2, 4 and 5 only
47. Consider the following statements regarding
44. With reference to the Shanghai Cooperation the National Coal Index (NCI):
Organization (SCO), consider the following 1. NCI was created to provide a benchmark
statements: for revenue-sharing contracts being
1. SCO is an economic and security bloc executed after the auctions for commercial
headquartered in Beijing. mining of coal.
2. All the founding members of SCO were 2. Prices of coal from all the sales channels
previously members of the ‘Shanghai of coal, excluding import are taken into
Five’ mechanism. account for compiling the NCI.

RE
3. The Reserve Bank of India (RBI) is a Which of the following statement(s) is/are
member of the SCO Interbank Consortium correct?
(SCO IBC). (a) 1 only
Which of the above statements is/are correct? (b) 2 only
(a) 1 only (c) Both 1 and 2
(b) 1 and 2 only O (d) Neither 1 nor 2
(c) 2 and 3 only
48. Consider the following statements regarding
(d) 1, 2 and 3
Indians renouncing their citizenship:
1. The number of Indians renouncing
SC
45. Consider the following statements:
citizenship in 2021 is highest during last
1. Cervical cancer is the second-most five years.
common cancer in India and is one of the
2. There has been a consistent rise in people
only vaccine-preventable cancers.
renouncing citizenship during 2017-
2. ‘Cervavac’ is India’s first quadrivalent 2021.
human papillomavirus vaccine (qHPV) 3. Mentioning reasons/circumstances
which protects women against cervical for renouncing Indian citizenship is
GS

cancer. mandatory for applicant who intends to


3. ‘Cervavac’ has been developed by Bharat acquire foreign citizenship.
Biotech. Which of the above statement(s) is/are
Which of the above statements are correct? correct?
(a) 1 and 2 only (a) 1 and 2 only
(b) 2 and 3 only (b) 2 and 3 only
(c) 1 and 3 only (c) 1 and 3 only
(d) 1, 2 and 3 (d) All are correct

46. With reference to Asian games, consider the 49. Consider the following statements about
following statements: cowpea/ Black Eyed Pea:
1. India is the only country to have won 1. Cowpea is a legume-based plant.
at-least one gold medal in every event of 2. It is tolerant to low rainfall and sandy
Asian Games. soil.
2. The 19th edition of the Asian games 3. Ghana is the only country where
will be hosted in the Chinese city of genetically modified (GM) cowpea is
Guangzhou. cultivated.

9 PTS2023/PPP-01/092022/11

Telegram - @upscmaterialonline1
.
Which of the above statement(s) is/are Select the correct answer from the code given
correct? below:
(a) 1 only (a) 1 only
(b) 1 and 2 only (b) 2 and 3 only
(c) 2 and 3 only (c) 1, 2 and 3
(d) 1, 2 and 3 (d) 3 only

50. Which one of the following is the most likely 53. Consider the following:
measure the Government/RBI can take to stop
Indian rupee from depreciating? 1. Suffering From Mental breakdown
1. Government should contain borrowing 2. Unmarried women
2. RBI must stick to its legal mandate of 3. Foetal Abnormalities
containing inflation 4. Risk to life
3. Encouraging Indian borrowers to issue 5. Rape and sexual assault victims
rupee denominated Masala Bonds
Which of the criteria given above are correct in

RE
4. Easing conditions relating to external
accordance with aborting a child in 24 weeks
commercial borrowing
as per the medical termination of pregnancy
Which of the above statements are correct? Amendment Act, 2021?
(a) 1 and 2 only (a) 1, 2, and 4 only
(b) 1 and 3 only (b) 3 and 5 only
(c) 2 and 3 only
O (c) 1, 2, 3,4 and 5 only
(d) 1, 2, 3 and 4 (d) 1, 3, 4 and 5 only

51. Consider the following statements regarding


54. Regarding Open network to Digital Commerce
SC
rare earth metal reserves:
(ONDC), consider the following statements:
1. China holds more than 50 percent of the
World’s Lithium reserves. 1. It is an open network to exchange only
goods on an electronic platform.
2. India is the biggest importer of Lithium
from China. 2. Department for Promotion of Industry
and Internal Trade (DPIIT) is the agency
3. Mineral Security partnership is an
governing ONDC platforms.
alliance formed by the US to reduce
GS

dependence on China. Which of the statements given above is/are


incorrect?
Which of the statement(s) given above is/are
correct? (a) 1 only
(a) 1 and 3 only (b) 2 only
(b) 2 only (c) Both 1 and 2
(c) 3 only (d) Neither 1 nor 2
(d) 1, 2 and 3
55. Which of the following are parliamentary
52. With reference to Antarctica Bill 2022, consider privileges?
the following statements: 1. Freedom of Speech
1. The Bill includes environmental and 2. Freedom from Arrest
wastes management concern over
Antarctica. 3. Exemption from Attendance as
Witnesses
2. It is applicable to both foreigners and
private corporations. 4. Right to Publish Debates and
3. The Bill does not prohibit ongoing Proceedings
research expeditions in the region. 5. Right to Punish Members and Outsiders

PTS2023/PPP-01/092022/11 10

Telegram - @upscmaterialonline1
.
Select the correct code given below: (d) The index calculates the indicators as set
by World Economic forum (WEF).
(a) 1, 2, and 4 only
(b) 2, 3, and 5 only 59. Consider the following pairs:
(c) 1, 2, 3, 4, and 5 only Elephant Reserves State
(d) 1, 3, 4, and 5 only 1. Dandeli - Uttarakhand
2. Singphan - Mizoram
56. With reference to the personal data protection
bill 2019, consider the following statements: 3. Lemru - Chhattisgarh
1. Data localization rights from the big tech 4. Srivilliputhur - Tamil Nadu
companies have been transferred to the
How many pair(s) given above is/are correctly
Government.
matched?
2. The Bill has been abolished after
(a) Only one pair
the recommendations of the Joint
parliamentary committee. (b) Only two pairs
Which of the statements given above is/are (c) Only three pairs

RE
incorrect?
(d) All four pairs
(a) 1 only
(b) 2 only 60. With reference to the “Air Quality and Health
in Cities” report, consider the following
(c) Both 1 and 2 statements:
(d) Neither 1 nor 2 O 1. Indian cities see high PM emissions, but
low NO2 emissions.
57. With reference to blue bonds in India, consider 2. Delhi ranked first in the list of top 10
the following statements: most polluted cities in both PM2.5 and
1. They are issued by RBI against the Blue NO2 levels.
SC
economic activities to be traded in the 3. PM 2.5 has shorter lifetime compared
markets. with NO2 and thus show very high
2. SEBI is the watchdog to provide clearance variability in space and time.
of blue bonds in India. Which of the above statements is/are correct?
3. They can also be issued for fishing (a) 1 only
activities.
(b) 1 and 2 only
GS

Which of the statements given above is/are


correct? (c) 2 and 3 only

(a) 1 and 3 only (d) 1, 2 and 3

(b) 2 and 3 only


61. It is an Indian Classical Dance form inspired
(c) 1 and 2 only by neo-Vaishnavism and evolved during 15th
(d) 1, 2 and 3 Century AD in Assam. The act during the
dance is performed on ‘Borgeets’ sung by
artists. Which of the following is the correct
58. Regarding the financial Inclusion index 2022, description of the dance forms given below?
which of the statements is correct?
(a) Bihu
(a) The index aims to incorporate the
insurance and banking sector exempting (b) Odissi
postal services. (c) Sattriya
(b) It is released by the Reserve Bank of (d) Thang-ta
India annually.
(c) It measures the state-wise data for 62. With reference to the United Nations
collection of financial activities in the Convention on laws of Seas (UNCLOS),
country. consider the following statements:

11 PTS2023/PPP-01/092022/11

Telegram - @upscmaterialonline1
.
1. It is the only international convention 3. Environment conservation
which stipulates a framework for state 4. Marine life security
jurisdiction in maritime spaces.
5. Avoiding water pollution
2. There is no right of innocent passage
through internal waters. Which of the above are objectives of Swachh
Sagar, Surakshit Sagar campaign, seen in
3. A coastal state has the right to prohibit or news recently?
limit freedom of navigation or overflight
in its Exclusive Economic Zone (EEZ). (a) 1, 2 and 5 only

Which of the above statements are correct? (b) 2, 4 and 5 only

(a) 1 and 2 only (c) 1, 2, 3 and 4 only


(d) 1, 2, 3, 4 and 5
(b) 2 and 3 only
(c) 1 and 3 only 66. Consider the following regarding Diammonium
(d) 1, 2 and 3 Phosphate (DAP):
1. DAP is soluble compound used in
63. With reference to Combined Maritime forces Agriculture to maintain pH of the soil.

RE
(CMF), consider the following statements: 2. It is a fertilizer but also used as Fire
1. It has established in 2001 against the retardants in Cigarettes.
piracy activities in the Indian Ocean. 3. The Prices of DAP is fixed by the
2. India has joined as partner country with government in India.
United States at CMF in 2022. 4. The subsidy for DAP is regulated by the
3. It has 34 member nations. state governments.
O Select the correct answer from the code given
Select the correct answer from the code given
below:
below:
(a) 1 and 4 only
(a) 1 and 3 only
SC
(b) 2 and 3 only
(b) 2 only
(c) 1, 3 and 4 only
(c) 1, 2 and 3
(d) 1 and 2 only
(d) None of the above

67. Consider the following statements:


64. With reference to India- Maldives strategic
relations, consider the following: 1. ‘Exercise VINBAX’ is a joint exercise
GS

between the militaries of India,


1. Maldives has the largest number of Bangladesh and Vietnam.
Indian diaspora than any other country.
2. India is the first country with which
2. Both the countries are the founding Vietnam has signed a mutual logistics
members of SAARC. support agreement.
3. India signed a pact with Maldives for Which of the above statement(s) is/are
essential goods in 1981. correct?
Select the correct answer from the code given (a) 1 only
below:
(b) 2 only
(a) 1 and 2 only
(c) Both 1 and 2
(b) 3 only
(d) Neither 1 nor 2
(c) 2 and 3 only
(d) 1, 2 and 3 68. The ‘Red Data Books’ published by the
International Union for Conservation of
Nature and Natural Resources (IUCN) contain
65. Consider the following: lists of:
1. To boycott single use plastics 1. Endemic plant and animal species
2. Coastal cleaning present in the biodiversity hotspots.

PTS2023/PPP-01/092022/11 12

Telegram - @upscmaterialonline1
.
2. Threatened plant and animal species. 3. It has been discovered by the Hubble
3. Protected sites for conservation of Telescope developed by NASA.
nature & natural resources in various Select the correct answer from the code given
countries. below:
Which of the statement given above is/are (a) 1 and 3 only
correct?
(b) 2 only
(a) 1 and 3 only
(c) 1 and 2 only
(b) 2 only
(d) 1, 2 and 3
(c) 2 and 3 only
(d) 3 only 73. With reference to places under National
importance tag in India, consider the
69. With reference to invasive alien species (IAS), following:
consider the following statements:
1. Kaleshwar Mahadev Temple, an ancient
1. They occur in all taxonomic groups, site in Himachal Pradesh was included
including animals, plants, fungi and under the list.
microorganisms.

RE
2. Archeological Survey of India is
2. Islands are more vulnerable to IAS than responsible for maintenance of the places
dry regions. in the list.
Which of the above statement(s) is/are 3. Ministry of culture enlists the places as a
correct? site of National Importance.
(a) 1 only O Which of the statements given above is/are
(b) 2 only correct?
(c) Both 1 and 2 (a) 1 and 3 only
(d) Neither 1 nor 2 (b) 2 and 3 only
SC
(c) 1, 2 and 3
70. The ‘G219 highway’, seen in news recently,
passes through a region claimed by India. The (d) 2 only
region is:
(a) Doklam 74. With reference history of India, which among
the following events took place during the
(b) Aksai Chin viceroyship of Lord Curzon?
(c) Tibet
GS

1. Establishment of separate public works


(d) Transnistria department in every province
2. Establishment of Department of
71. Which of the Following country has developed Commerce and Industry
a Nano-technology based Tattoo to monitor 3. Creation of a separate Muslim majority
Health conditions using an ECG device? North-West Frontier Province
(a) Japan 4. Ganges canal declared open
(b) China Select the correct answer using the code given
(c) Indonesia below:
(d) South Korea (a) 1 and 2 only
(b) 2 and 3 only
72. Which of the following statement is correct
about the Cartwheel galaxy, seen in news (c) 3 and 4 only
recently? (d) 1 and 4 only
1. It is located in the Sculptor constellation
about 500 million light years away. 75. With reference to Prevention of Money
2. It contains ring structure made up of dust Laundering Act, 2002, consider the following
particles exist in transitory stage. statements:

13 PTS2023/PPP-01/092022/11

Telegram - @upscmaterialonline1
.
1. It enables government authorities (c) 2 and 3 only
to confiscate property and/or assets
(d) 1, 2 and 3
earned from illegal sources and through
laundering of public money.
78. With reference to the legal provisions regarding
2. The provision of this act applies to all
financial institutions, their financial corporal punishment in India, consider the
intermediaries and banks, including following statements:
RBI. 1. The Juvenile Justice Act prohibits
3. No person accused of an offence under ‘physical punishment’ and ‘mental
the act can be released on bail. harassment’ of children and makes it a
punishable offence.
Which of the above statement(s) is/are
correct? 2. Under the Right of Children to Free and
Compulsory Education (RTE) Act, 2009,
(a) 1 only every school has to constitute a ‘Corporal
(b) 1 and 2 only Punishment Monitoring Cell’ to look into
complaints of corporal punishments.
(c) 2 and 3 only
Which of the above statements is/are correct?
(d) 1, 2 and 3

RE
(a) 1 only
76. Which of the following is/are the grounds of (b) 2 only
disqualification for membership of a State
Legislative Assembly as mentioned in the (c) Both 1 and 2
Constitution? (d) Neither 1 and 2
1. Holding an office of profit under
Government of India or State.
O 79. With reference to the India State of Forest
2. Proven guilty of certain election Report, 2021, arrange the following states in
offences. increasing order of mangrove cover distribution
3. Convicted for any offence resulting in (area wise)?
SC
imprisonment for two or more years. 1. Maharashtra
4. Voluntary acquisition of citizenship of 2. Gujarat
foreign state.
3. West Bengal
Select the correct answer using the code given
4. Andaman & Nicobar Islands
below:
Select the correct answer using the code given
(a) 1 only
below:
GS

(b) 1 and 4 only


(a) 1-4-2-3
(c) 1, 2 and 3 only
(b) 2-3-4-1
(d) 1, 2, 3 and 4
(c) 3-2-1-4

77. With reference to monkeypox virus, consider (d) 4-1-3-2


the following statements:
1. Monkeypox virus is an enveloped double- 80. What is true regarding Ocean Thermal
stranded DNA virus that is less prone to Energy?
rapid mutations. (a) It results from the interaction of the wind
2. Smallpox vaccines can be used to treat with the surface of the sea.
monkeypox as well.
(b) It is the energy that can be extracted
3. Vaccines contribute to passive immunity from tides in oceans.
by providing us with a controlled way to
create an immune response. (c) It is energy derived from temperature
difference between surface and ocean
Which of the above statements is/are correct? depths.
(a) 1 only
(d) It is the electricity generated from
(b) 1 and 2 only underwater marine currents.

PTS2023/PPP-01/092022/11 14

Telegram - @upscmaterialonline1
.
81. With reference to carbon nanotubes (CNTs), (c) 1 and 3 only
consider the following statements:
(d) 1, 2 and 3
1. They exhibit remarkable electrical
conductivity, exceptional tensile strength
but poor thermal conductivity. 85. Consider the following statements regarding
the One Nation One Fertiliser Scheme:
2. They can be used for drug delivery inside
human body. 1. Under this scheme, companies are no
longer allowed to display their name,
3. Ink formulations based on CNT
dispersions can be used as biosensors. brand, logo etc. on fertilizer bags.

4. They are extremely resistant to 2. Both public and private sector companies
corrosion. will be required to sell subsidized
fertilisers under a single brand name.
Which of the above statements are correct?
3. It aims to establish ‘one stop shops’ for
(a) 1 and 2 only Indian farmers for all quality agri-inputs
(b) 2, 3 and 4 only like fertilizers, seeds, pesticides at
reasonable prices.
(c) 1, 3 and 4 only
Which of the above statements is/are correct?

RE
(d) 1, 2, 3 and 4
(a) 1 and 2 only
82. Arrange the following languages in correct (b) 2 only
chronological order of their being declared a
‘classical language’? (c) 1 and 3 only
1. Sanskrit (d) 2 and 3 only
2. Telugu
3. Malayalam
4. Tamil
O 86. Consider the following statements regarding
‘Manusmriti’:
1. It is a Sanskrit text belonging to the
5. Odia
Dharmasutra, literary tradition of
SC
Select the correct answer using the code given Hinduism.
below:
2. While it classifies the society into four-
(a) 1-4-3-2-5 fold varna system, it does not cover
(b) 2-4-1-5-3 subjects such as the social obligations
and duties of the various castes.
(c) 4-1-2-3-5
3. It was one of the first Sanskrit texts to be
(d) 1-2-4-5-3
GS

translated into English and was used to


construct the Hindu law code, for the East
83. Bati’, ‘Kang’, ‘Cheena’ and ‘Gujro’ are local India Company administered enclaves.
names for different varieties of:
Which of the above statements is/are
(a) Wheat incorrect?
(b) Millets (a) 1 and 2 only
(c) Rice
(b) 2 only
(d) Maize
(c) 1 and 3 only

84. Consider the following: (d) 3 only


1. High in silica and alumina
87. Government of India recently undertook the
2. High ash fusion temperature
“Aashwasan Campaign” to:
3. Low Sulphur content
(a) Address the acute shortage of certain
Which of the above are the characteristics of Antiretroviral (ARV) drugs in Anti-
Bituminous coal found in India? Retroviral Therapy (ART) centers.
(a) 1 and 2 only (b) Identify and rehabilitate persons engaged
(b) 2 and 3 only in begging across India.

15 PTS2023/PPP-01/092022/11

Telegram - @upscmaterialonline1
.
(c) Detect instances of Tuberculosis (TB) Select the correct answer using the code given
among tribal populations. below:
(d) Identify children that have been (a) 1 and 4 only
orphaned, abandoned or have lost a
parent during the COVID pandemic. (b) 2 and 3 only
(c) 1, 2 and 3 only
88. Consider the following statements: (d) 2, 3 and 4 only
1. A swarm drone system consists of a
number of small drones which are AI-
91. Consider the following statements:
enabled and capable of communicating
with the control station as well as among 1. The Vienna Convention of 1985 requires
themselves. countries to take concrete actions to
2. ‘Zorawar’ is an Autonomous Surveillance control ozone-depleting substances.
and Armed Drone Swarm (A-SADS) 2. The Vienna Convention and its Montreal
system being developed indigenously. Protocol are the first treaties to achieve
Which of the above statements is/are correct? universal ratification.
3. Hydro-fluorocarbons do not deplete the

RE
(a) 1 only
ozone layer.
(b) 2 only
Which of the above statements are correct?
(c) Both 1 and 2
(a) 1 and 2 only
(d) Neither 1 nor 2
(b) 2 and 3 only
89. Which of the following statements regarding (c) 1 and 3 only
O
the Rashtriya Vayoshri Yojana (RVY) are
correct? (d) 1, 2 and 3
1. It is a scheme for providing physical
aids and assisted-living devices only to 92. With reference to Anang Tal Lake, seen in news
SC
those senior citizens belonging to BPL recently, consider the following statements:
category. 1. It was built by Alauddin Khalji to provide
2. 30% of the beneficiaries in each district water for construction of Qutab Minar.
shall be women.
2. It has been declared as a monument of
3. The devices are provided at doorsteps national importance.
only for the senior citizens aged 80 years
or more. Which of the above statement(s) is/are
GS

correct?
Select the correct answer using the code given
below: (a) 1 only
(a) 1 and 2 only (b) 2 only
(b) 2 only (c) Both 1 and 2
(c) 1 and 3 only (d) Neither 1 nor 2
(d) 1, 2 and 3
93. Consider the following statements regarding
90. Which of the following can be the possible Pradhan Mantri Jan Arogya Yojana
impacts associated with a La Nina event? (PMJAY):
1. Rainfall associated with the northeast 1. It aims to make secondary and tertiary
monsoon in India tends to be greater healthcare completely cashless for the
than normal. underprivileged sections of the society.
2. Rainier-than-normal conditions are over 2. It is implemented by the National Health
southeastern Africa. Authority (NHA) in all states and UTs.
3. Drier-than-normal conditions along the 3. The Ayushman Bharat health cards
pampas region of South America. issued under the scheme can be used only
4. Flooding in Australia. for the central health schemes.

PTS2023/PPP-01/092022/11 16

Telegram - @upscmaterialonline1
.
Which of the above statements is/are (c) Celebes Sea
incorrect?
(d) Sea of Japan
(a) 1 only
(b) 1 and 2 only 98. Which of the following statements regarding
the Construction and Demolition Waste
(c) 2 and 3 only
Management Rules, 2016 is/are correct?
(d) 3 only
1. Under the rules, the Central Pollution
Control Board is responsible for preparing
94. With reference to ‘Mandala’ art, consider the a code of practices and standards for
following statements: products of construction and demolition
1. A mandala is defined by a geometric waste.
configuration that usually incorporates 2. As per these rules, the local bodies have to
the circular shape in some form.
utilize 10-20% material from construction
2. It is rooted in Hinduism and Buddhism. and demolition waste in municipal and
3. This art form appears in the Thangka government contracts.
paintings. 3. Large generators of wastes are liable

RE
Which of the above statements are correct? to pay relevant charges for collection,
transportation, processing and disposal
(a) 1 and 2 only of construction and demolition waste.
(b) 2 and 3 only
Select the correct answer using the code given
(c) 1 and 3 only below:
(d) 1, 2 and 3 (a) 1 only
O
95. With reference to history of India, Kedambady
Ramayya Gowda led an army of farmers to
(b)
(c)
2 and 3 only
3 only
fight against the British in which among the
SC
(d) 1, 2 and 3
following rebellions?
(a) Amara Sullia Rebellion 99. Martand Sun Temple is located in which State/
(b) Punnapara-Vayalar Revolt UT, seen in news recently?

(c) Moplah Rebellion (a) Karnataka

(d) Narkelberia Uprising (b) Jammu and Kashmir


GS

(c) Gujarat
96. Consider the following statements:
(d) Bihar
1. Narmada is the largest west flowing river
of the Indian peninsula.
100. With reference to the tribal communities
2. The Narmada Canal is the largest lined of central and eastern India, the term ‘Johar’
irrigation canal in the world. relates to:
Which of the above statements is/are correct? (a) A traditional way of greeting which
(a) 1 only essentially means ‘salutation and
welcome’.
(b) 2 only
(b) The practice of mass self-immolation
(c) Both 1 and 2
by women to avoid capture and
(d) Neither 1 nor 2 enslavement.
(c) A community-owned traditional
97. Taiwan Strait connects South China Sea harvested rainwater storage wetland
with: principally used for effectively harnessing
(a) Philippine Sea water resources.
(b) East China Sea (d) None of the above

17 PTS2023/PPP-01/092022/11

Telegram - @upscmaterialonline1
Space for Rough Work

.
RE
O
SC
GS

PTS2023/PPP-01/092022/11 18

Telegram - @upscmaterialonline1
.
Test No.

IAS Prelims - 2023


11
PTS2023/PPP-01/092022/11

GENERAL STUDIES

RE
CURRENT AFFAIRS - 2
(July-August, 2022)
O
Answer Key

Q. 1 (c) Q. 21 (d) Q. 41 (d) Q. 61 (c) Q. 81 (b)


SC
Q. 2 (d) Q. 22 (c) Q. 42 (c) Q. 62 (a) Q. 82 (c)
Q. 3 (c) Q. 23 (b) Q. 43 (c) Q. 63 (c) Q. 83 (b)
Q. 4 (a) Q. 24 (c) Q. 44 (a) Q. 64 (c) Q. 84 (d)
Q. 5 (b) Q. 25 (c) Q. 45 (a) Q. 65 (d) Q. 85 (b)
Q. 6 (a) Q. 26 (b) Q. 46 (d) Q. 66 (d) Q. 86 (a)
GS

Q. 7 (d) Q. 27 (c) Q. 47 (a) Q. 67 (b) Q. 87 (c)


Q. 8 (b) Q. 28 (b) Q. 48 (c) Q. 68 (b) Q. 88 (a)
Q. 9 (a) Q. 29 (b) Q. 49 (b) Q. 69 (c) Q. 89 (d)
Q. 10 (a) Q. 30 (b) Q. 50 (d) Q. 70 (b) Q. 90 (d)
Q. 11 (b) Q. 31 (b) Q. 51 (c) Q. 71 (d) Q. 91 (b)
Q. 12 (a) Q. 32 (a) Q. 52 (c) Q. 72 (c) Q. 92 (b)
Q. 13 (a) Q. 33 (c) Q. 53 (c) Q. 73 (c) Q. 93 (c)
Q. 14 (b) Q. 34 (c) Q. 54 (c) Q. 74 (b) Q. 94 (d)
Q. 15 (d) Q. 35 (c) Q. 55 (c) Q. 75 (b) Q. 95 (a)
Q. 16 (d) Q. 36 (b) Q. 56 (a) Q. 76 (b) Q. 96 (c)
Q. 17 (b) Q. 37 (c) Q. 57 (b) Q. 77 (b) Q. 97 (b)
Q. 18 (c) Q. 38 (d) Q. 58 (b) Q. 78 (d) Q. 98 (b)
Q. 19 (a) Q. 39 (b) Q. 59 (b) Q. 79 (a) Q. 99 (b)
Q. 20 (a) Q. 40 (b) Q. 60 (a) Q. 80 (c) Q. 100 (a)

1
PTS2023/PPP-01/092022/11
PTS2023/PPP-01/092022/11

Telegram - @upscmaterialonline1
1. Correct Option: (c)

.
 States/UTs have been requested to constitute
a Special Task Force for elimination of single
Explanation: use plastics and effective implementation
 Statement 1 is incorrect: These rules of the rules. A National Level Taskforce
prohibit specific single-use plastic items has also been constituted for coordination
which have “low utility and high littering efforts.
potential” by 2022. The ban will not apply to Why in News?
commodities made of compostable plastic.
 The ban on certain single-use plastic (SUP)
Supplementary notes: items kicked in recently. The violation of the
Plastic Waste Management (Amendment) ban will invite punitive action, including a
Rules, 2021 fine or a jail term or both, detailed under
Section 15 of the Environment Protection
 These rules prohibit specific single-use Act (EPA) and under the bylaws of the
plastic items which have “low utility and respective municipal corporations.
high littering potential” by 2022. The
manufacture, import, stocking, distribution,  State governments have initiated an
sale and use of the identified single-use enforcement campaign and close down units
plastic will be prohibited with effect from engaging in the production, distribution,
stocking and sale of such items.

RE
the 1st July, 2022.
 The ban will not apply to commodities made  For effective enforcement of the ban,
of compostable plastic. Instead of using national and State-level control rooms have
plastic made from petrochemicals and fossil been set up, and special enforcement teams
fuels, compostable plastics are derived from formed to check the illegal manufacture,
renewable materials like corn, potato, and import, stocking, distribution, sale and use
tapioca starches, cellulose, soy protein, of banned SUP items
O
and lactic acid. These are non-toxic and
decompose back into carbon dioxide, water,
 The Central Pollution Control Board (CPCB)
has also launched a grievance redressal
and biomass when composted.
application to empower citizens to help curb
 The permitted thickness of the plastic bags, the use of plastic.
currently 50 microns, will be increased to 75
SC
microns from 30th September, 2021, and to
120 microns from the 31st December, 2022. 2. Correct Option: (d)
Currently the Plastic Waste Management Explanation:
Rules, 2016, prohibits manufacture, import,
stocking, distribution, sale and use of carry  All statements are correct
bags and plastic sheets less than 50 microns
Supplementary notes:
in thickness in the country.
The Code on Wages
GS

 Plastic bags with higher thickness are more


easily handled as waste and have higher  The Code on Wages, 2019 seeks to
recyclability. regulate wage and bonus payments in all
 The plastic packaging waste shall be employments where any industry, trade,
collected and managed in an environmentally business, or manufacture is carried out.
sustainable way through the Extended  The Code replaces the following four laws:
Producer Responsibility (EPR) of the (i) the Payment of Wages Act, 1936, (ii)
Producer, importer and Brand owner. the Minimum Wages Act, 1948, (iii) the
 For effective implementation of EPR, the Payment of Bonus Act, 1965, and (iv) The
Guidelines for the same being brought out Equal Remuneration Act, 1976.
have been given legal force through Plastic  The Code will apply to all employees. The
Waste Management Amendment Rules, central government will make wage-related
2021. decisions for employments such as railways,
 The guidelines allow for sale and purchase mines, and oil fields, among others. State
of surplus extended producer responsibility governments will make decisions for
certificates. This allows to set-up a market all other employments. Wages include
mechanism for plastic waste management. salary, allowance, or any other component
expressed in monetary terms. This does not
 The Central Pollution Control Board, along
include bonus payable to employees or any
with state pollution bodies, will monitor
travelling allowance, among others.
the ban, identify violations, and impose
penalties already prescribed under the  According to the Code, the central government
Environmental Protection Act. will fix a floor wage, taking into account

PTS2023/PPP-01/092022/11 2

Telegram - @upscmaterialonline1
.
living standards of workers. Further, it if executed with precision, as demonstrated
may set different floor wages for different by some advanced bombers in the USA.
geographical areas. Before fixing the floor
 The Autonomous Flying Wing Technology
wage, the central government may obtain
Demonstrator is a precursor to an
the advice of the Central Advisory Board
autonomous stealthy unmanned aerial
and may consult with state governments.
combat vehicles (UCAV) being developed
The minimum wages decided by the
by the DRDO’s Aeronautical Development
central or state governments must
Establishment (ADE), primarily for the
be higher than the floor wage. In case
Indian Air Force. A deck launched version
the existing minimum wages fixed by the
for the Indian Navy is also said to be in
central or state governments are higher
the pipeline. The UCAV will be capable of
than the floor wage, they cannot reduce the
launching missiles and precision-guided
minimum wages.
munitions.
 The Code prohibits employers from
 The Unmanned Aerial Vehicle (UAV) is
paying wages less than the minimum
powered by a small turbofan engine. The
wages. Minimum wages will be notified
airframe, undercarriage and entire flight
by the central or state governments. This control and avionics systems used for the
will be based on time, or number of pieces aircraft were developed indigenously.
produced. The minimum wages will be

RE
revised and reviewed by the central or state  The engine is Russian TRDD-50MT
governments at an interval of not more than originally designed for cruise missiles.
five years. While fixing minimum wages, the  DRDO is in the process of developing UAVs of
central or state governments may take into different classes to meet the requirements of
account factors such as: (i) skill of workers, the armed forces. Rustom-2, the indigenous
and (ii) difficulty of work. Medium Altitude Long Endurance (MALE)
The central or state government may fix the UAV under development, had crossed a


working day. In case employees work in


O
number of hours that constitute a normal

excess of a normal working day, they will be


milestone by reaching an altitude of 25,000
feet and an endurance of 10 hours and is
being designed to reach an altitude of 30,000
entitled to overtime wage, which must be at feet and 18 hours endurance.
SC
least twice the normal rate of wages.
Why in News?
Why in News?  The Defence Research and Development
 Recently, there have been speculations Organisation (DRDO) recently carried out
that the Union government may implement the maiden test flight of a new Unmanned
four labour codes on wages, social security, Aerial Vehicle (UAV), an autonomous Flying
industrial relations and occupation safety, Wing Technology Demonstrator, from the
health and working conditions from July 1. Aeronautical Test Range, Chitradurga,
GS

Karnataka.
 If these labour codes are implemented,
the new wage code will impact employees’  Operating in a fully autonomous mode, the
working hours, salary restructuring, PF aircraft exhibited a perfect flight, including
contribution, gratuity aspect and encasing take-off, way point navigation and a smooth
of Earned Leaves among the prominent touchdown.
ones.  This flight marks a major milestone in terms
of proving critical technologies towards the
development of future unmanned aircraft
3. Correct Option: (c)
and is significant step towards self-reliance
Explanation: in such strategic defence technologies.
 Both statements are correct
Supplementary notes:
4. Correct Option: (a)
Explanation:
Flying Wing Technology Demonstrator
 Statement 2 is incorrect: Under Article
 The flying wing type of aircraft refers to a
68, an election to fill a vacancy in the office
tailless fixed-wing aircraft which houses its
of Vice-President occurring by reason of his
payload and fuel in its main wings and does
death, resignation or removal, or otherwise
not have a defined fuselage-like structure
shall be held as soon as possible after the
found in conventional aircrafts.
occurrence of the vacancy, and the person
 The design, scientists say, has the potential elected to fill the vacancy shall, subject to
to deliver high fuel efficiency and stability, the provisions of article 67, be entitled to

3
PTS2023/PPP-01/092022/11

Telegram - @upscmaterialonline1
.
hold office for the full term of five years from  Any person qualified to be elected and
the date on which he enters upon his office. intending to stand for election as Vice-
President is required to be nominated by at
 Statement 3 is incorrect: The Returning
least 20 MPs as proposers and at least 20
Officer usually appointed to conduct the
MPs as seconders.
Vice-Presidential elections is the Secretary-
General of either House of the Parliament,  Nomination papers are to be presented
by rotation. to the Returning Officer at the place
and upto the time and date, specified in the
Supplementary notes: public notice. A maximum of 4 nomination
Election of the Vice-President papers by, or on behalf of, any candidate
may be presented to, or accepted by, the
 Under Article 66 of the Constitution, Returning Officer.
the Vice-President is elected by an electoral
college consisting of members of both  A candidate seeking election as Vice-
Houses of Parliament, in accordance with President is required to make a security
the system of proportional representation deposit of Rs.15,000/-. This is the only
by means of the single transferable vote amount that is required to be deposited by
and the voting in such election is by secret a candidate irrespective of the number of
ballot. nomination papers filed on his behalf.

RE
 The Vice-President is not a member of  All doubts and disputes arising in connection
either House of Parliament or of a House of with the election of the Vice-President are
a Legislature of any state. If a member of enquired into and decided by the Supreme
either House of Parliament or of a House of Court of India whose decision is final.
a Legislature of any state is elected as Vice-  A petition challenging the election of the
President, he is deemed to have vacated his Vice-President is heard by a five-judge
seat in that House on the date he/she enters bench of the Supreme Court of India.
his office as Vice-President.
O Why in News?
 A person cannot be elected as Vice-
President unless he -  The Election Commission recently
announced that the poll for the post of Vice-
 is a citizen of India; President will be held on August 6, as M.
SC
 has completed the age of 35 years, and Venkaiah Naidu’s term was coming to an
end on August 10.
 is qualified for election as a member of
the Council of States (Rajya Sabha)  The Lok Sabha Secretary-General would be
appointed as the Returning Officer.
 A person is not also eligible if he holds
any office of profit under the Government
of India or a State Government or any 5. Correct Option: (b)
subordinate local authority.
GS

Explanation:
 An election to fill a vacancy caused by the
expiry of the term of office of Vice-President  Statement 1 is incorrect: The Centre pays
is completed before the expiry of the term. subsidy on urea to fertilizer manufacturers
In case a vacancy arises by reasons of death, on the basis of cost of production at each
resignation or removal or otherwise, the plant and the units are required to sell the
election to fill that vacancy is held as soon fertilizer at the government-set Maximum
as possible after the occurrence. The person Retail Price (MRP).
so elected is entitled to hold office for a full Supplementary notes:
term of 5 years from the date he enters
office. Fertilizers’ Price Rise
 The Election Commission of India conducts  Fertilizer prices have risen nearly 30 per
the election to the office of the Vice- cent since the beginning of 2022, following
President. last year’s 80 per cent surge. The rise was
driven by factors like surging input costs,
 The Returning Officer usually
supply disruptions due to sanctions (Belarus
appointed to conduct the Vice-
and Russia) as well as export restrictions in
Presidential elections is the Secretary-
China.
General of either House of the Parliament,
by rotation. The Returning Officer issues  According to the Centre’s monthly bulletin,
a public notice of the intended election in the international prices of DAP have
a prescribed form, inviting nomination of increased by about 65.66 per cent from $565
candidates and specifies the place where (Rs 44,645) per tonne in May, 2021 to $936
the nomination papers are to be delivered. per tonne in May, 2022. Urea prices have

PTS2023/PPP-01/092022/11 4

Telegram - @upscmaterialonline1
6. Correct Option: (a)

.
increased by about 94.09 per cent from $372
per tonne in May, 2021 to $722 per tonne
in May, 2022. Urea production is based Explanation:
on natural gas, prices of which are closely  Option (a) is correct: The South Indian
related to international oil prices. Sulphur Liberal Federation, popularly known as
prices have increased by about 138.8 per Justice Party, was founded in 1917 by Sir
cent from $216 per tonne in May, 2021 to Pitti Theagaraya Chetty, Dr. T M Nair, and
$516 per tonne in May, 2022. Dr. C Natesa Mudaliar.
 In efforts to control price rise, the  It was the first party to raise the flag of anti-
government increased the nutrient-based Brahminism, and oppose the caste system
subsidy rate for Kharif 2022. The Centre’s that put Brahmins at the top of the social
fertiliser subsidy expenditure is likely to be hierarchy.
in the range of Rs 2.10-2.30 lakh crore in
the current financial year Supplementary notes:
 Apart from being the second-largest Dravida Nadu
importer of fertilisers, India imports raw  The movement for a separate Dravida
materials for domestic production such as country has passed through several stages
natural gas to make urea and phosphoric and meanings. The early 20th century

RE
acid to make DAP. agitations against the colonial government
 The government pays a subsidy to fertilizer of Madras state (of which, along with Tamil
producers to make this critical ingredient Nadu, portions of modern-day Andhra
in agriculture affordable to farmers. This Pradesh, Karnataka, and Kerala were
allows farmers to buy fertilisers at below- part) often included expressions of regional
market rates. The difference between the aspirations.
cost of production/import of a fertilizer and
O  The South Indian Liberal Federation,
the actual amount paid by farmers is the popularly known as Justice Party, which
subsidy portion borne by the government. was founded in 1917 by Sir Pitti Theagaraya
 Subsidy on Urea: The Centre pays subsidy Chetty, Dr. T M Nair, and Dr. C Natesa
on urea to fertilizer manufacturers on the Mudaliar, was the first to raise the flag of
basis of cost of production at each plant and anti-Brahminism, and oppose the caste
SC
the units are required to sell the fertilizer at system that put Brahmins at the top of the
the government-set Maximum Retail Price social hierarchy.
(MRP).  At the time, the presence of Brahmins in the
Madras government was disproportionately
 Subsidy on Non-Urea Fertilisers: The MRPs
higher than their population in the state, and
of non-urea fertilisers are decontrolled
the Justice Party demanded opportunities
or fixed by the companies. The Centre,
for those lower in the caste hierarchy.
however, pays a flat per-tonne subsidy on
GS

these nutrients to ensure they are priced at  In 1920, the Justice Party won the first
“reasonable levels”. legislative council elections held under the
Government of India Act, 1919, and formed
 All Non-Urea based fertilisers are regulated
the government. The Congress had boycotted
under Nutrient Based Subsidy (NBS)
the elections. The Justice Party remained in
Scheme. Under NBS, a fixed amount power until 1926, and then from 1930-37.
of subsidy decided on an annual basis,
is provided on each grade of subsidized  Periyar, the founder of the Self-Respect
Phosphatic & Potassic (P&K) fertilizers Movement (1925), was both anti-caste and
depending on its nutrient content. It aims anti-religion. He advocated major social
at ensuring the balanced use of fertilizers, reforms, including equality for women in
improving agricultural productivity, society, and supporting birth control for
promoting the growth of the indigenous women for their health and well-being. He
fertilizers industry and also reducing the also opposed the domination of Hindi and
burden of Subsidy. emphasised the distinct cultural identity of
the Tamil nation.
Why in News?
 In 1938, the Justice Party and Self-Respect
 The recent global rise in fertiliser prices Movement came together, representing the
due to shortage has dealt a severe blow to merger of the party and the movement. In
farmers across India, which is the second- 1944, the new outfit was named Dravidar
largest importer of fertilisers. Many states Kazhagam. DK was anti-Brahmin, anti-
have seen cultivators erupting in protest, Congress, and anti-Aryan (read North
even as the Centre dismissed news of low Indian), and launched a movement for an
availability of fertilisers. independent Dravida nation.

5
PTS2023/PPP-01/092022/11

Telegram - @upscmaterialonline1
.
 Post-independence, DK continued to  According to the World Organization
demand Dravida Nadu. and Periyar refused for Animal Health, more than 65% of
to contest elections. In 1949, Annadurai split communicable diseases are zoonotic in
from Periyar due to ideological differences, origin.
and his DMK joined the electoral process.
The DMK’s platforms were social democracy  The One Health strategy consisting of
and Tamil cultural nationalism, but comprehensive health of human beings,
Annadurai was silent on Dravida Nadu. In animals and the environment needs to be
1967, Annadurai became Chief Minister. implemented to control hundreds of zoonotic
diseases.
 The States Reorganisation Act, 1956 redrew
the boundaries of states along linguistic  Adopting a One Health approach, which
lines, and created the states of Tamil Nadu, unites medical, veterinary and environmental
Andhra Pradesh, Mysore and Kerala in expertise, will help governments, businesses
southern India. In the process, a major and civil society achieve enduring health for
demand of the linguistic movements was people, animals and environments alike.
fulfilled, and the idea of an independent
Dravida Nadu was weakened further. Why in News?

Why in News?  Indian Immunological Limited (IIL)


recently organized a nationwide free

RE
 Andimuthu Raja, the DMK MP from vaccination camp against zoonotic diseases
Nilgiris, recently said in the presence of on account of World Zoonosis Day 2022.
Chief Minister M K Stalin that if the central
government did not give Tamil Nadu greater  The ‘Anti-Rabies vaccine drive’ aimed to
autonomy, the DMK could be “compelled” to spread awareness on the disease and need
revive the demand for a separate “Dravida to restrain it.
Nadu”.

7. Correct Option: (d)


O 8. Correct Option: (b)
Explanation:
Explanation:
 Statement 1 is incorrect: Article 340
 All statements are correct of the Indian Constitution empowers
SC
Supplementary notes: the President to appoint a Commission
consisting of such persons as he thinks fit
Zoonotic Diseases to investigate the conditions of socially and
 Zoonosis or zoonotic disease is a disease educationally backward classes within the
that has passed into the human population territory of India.
from an animal source directly or through  A Commission so appointed shall investigate
an intermediary species. the matters referred to them and present
GS

 Zoonotic infections can be bacterial, viral, to the President a report setting out the
or parasitic in nature, with animals playing facts as found by them and making such
a vital role in maintaining such infections. recommendations as they think proper.
Examples of zoonosis include HIV-AIDS,
 The President shall cause a copy of the report
Ebola, malaria, rabies, West Nile fever,
and the current novel coronavirus disease so presented together with a memorandum
(COVID-19) disease. explaining the action taken thereon to be
laid before each House of Parliament.
 A UN Report on Zoonotic Diseases identifies
seven anthropogenic driving factors leading Supplementary notes:
to the emergence of zoonotic diseases:
Sub-categorisation of OBCs
 Increased demand for animal protein
 The Kalelkar Commission, set up in 1953,
 Rise in intense and unsustainable was the first to identify backward classes
farming other than the Scheduled Castes (SCs)
 Increased use and exploitation of and Scheduled Tribes (STs) at the national
wildlife level.

 Unsustainable utilization of natural  The Mandal Commission Report, 1980


resources estimated the OBC population at 52% and
classified 1,257 communities as backward.
 Travel and transportation
 It recommended increasing the existing
 Changes in food supply chains
quotas, which were only for SC/ST, from
 Climate change crisis 22.5% to 49.5% to include the OBCs.

PTS2023/PPP-01/092022/11 6

Telegram - @upscmaterialonline1
.
 The central government reserved 27% of Why in News?
seats in union civil posts and services for
 The Union Cabinet recently gave the
OBCs [Article 16(4)]. The quotas were
subsequently enforced in central government 13th extension to the Justice Rohini
educational institutions [Article 15 (4)]. Commission, giving it time until January
31, 2023 to submit its report.
 In 2008, the Supreme Court directed the
central government to exclude the creamy  The commission was set up in 2017 under
layer (advanced sections) among the OBCs. Article 340 of the Constitution. It was
tasked with sub-categorization of the Other
 OBCs are granted 27% reservation in jobs Backward Classes (OBCs) and equitable
and education under the central government distribution of benefits reserved for them.
but only a few affluent communities among Its initial deadline to submit its report was
the over 2,600 included in the Central List 12 weeks — by 2018.
of OBCs have secured a major part of this.
At present, there is no sub-categorisation
and 27% reservation is a monolithic entity. 9. Correct Option: (a)
 Sub-categorisation or creating categories Explanation:
within OBCs for reservation would ensure
“equitable distribution” of representation  Statement 4 is incorrect: SRI is initially

RE
among all OBC communities. labour intensive as it needs 50% more man-
days for transplanting and weeding.
 However, sub-categorisation can be used to
appease one vote-bank or the other within Supplementary notes:
the category and thus a cause of social
justice would end up being politicized. System of Rice Intensification (SRI)

 The commission requested for an  The System of Rice Intensification is an


appropriate budget provision for a
O agro-ecological methodology for increasing
proposed all-India survey for an estimate the productivity of irrigated rice by changing
of the caste-wise population of OBCs. the management of plants, soil, water and
nutrients.
 The reason given was the absence of data
for the population of various communities  SRI originated in Madagascar in the 1980s
SC
to compare with their representation in and is based on the cropping principles of
jobs and admissions as the data of the significantly reducing plant population,
Socio-Economic Caste Census (SECC) improving soil conditions and irrigation
were not considered reliable. methods for root and plant development, and
improving plant establishment methods.
 Justice G Rohini Commission was
constituted under Article 340 of the  SRI methodology is based on four main
Constitution with the approval of the principles that interact with each other:
President, to complete the task of sub-
GS

 Early, quick and healthy plant


categorizing 5000-odd castes in the
establishment
central OBC list so as to ensure more
equitable distribution of opportunities in  Reduced plant density
central government jobs and educational
institutions.  Improved soil conditions through
enrichment with organic matter
 Commission’s Terms of References:
 Reduced and controlled water
 To examine the uneven distribution of application
reservation benefits among different
castes in the central OBC list.  Based on these principles, farmers can
adapt recommended SRI practices to
 To work out the mechanism, criteria, respond to their agro-ecological and
norms and parameters in a scientific socioeconomic conditions. Adaptations are
approach for sub-categorisation often undertaken to accommodate changing
within such OBCs. weather patterns, soil conditions, labor
 To take up the exercise of availability, water control, access to organic
identifying the respective castes/ inputs, and the decision whether to practice
communities/sub-castes/synonyms fully organic agriculture or not.
for comprehensive data coverage.
 The benefits of SRI include: 20%-100%
 To study and recommend correction or more increased yields, up to a 90%
of any repetitions, ambiguities, reduction in required seed, and up to 50%
inconsistencies and errors of spelling water savings. SRI principles and practices
or transcription have been adapted for rainfed rice as well

7
PTS2023/PPP-01/092022/11

Telegram - @upscmaterialonline1
.
as for other crops (such as wheat, sugarcane  The area sown this time is also 24 per cent
etc.), with yield increases and associated less than the corresponding period of 2021.
economic benefits. The primary reason being attributed to
the reduced sown area is the failure of the
Why in News? monsoon in the month of June in most parts
 According to data released by the of the country.
Department of Agriculture and Farmers
Welfare, just 7.22 million hectares (mha) 10. Correct Option: (a)
have been sown with paddy, the lowest
area sown till the second week of July when Explanation:
compared to the last 10 years.  Option (a) is correct

RE
O
SC
Supplementary notes: the original 2011 Nord Stream, which runs
parallel to the new project.
Nord Stream 1
Why in News?
 Nord Stream 1 is a 1,224 km underwater gas
pipeline that runs from Vyborg in northwest  The Nord Stream 1, Germany’s main source
Russia to Lubmin in northeastern Germany of gas from Russia, was shut down recently
via the Baltic Sea. for 10 days of scheduled maintenance work.
 Majority owned by the Russian energy giant  There are growing concerns in European
GS

Gazprom, the pipeline is the primary route countries that Russia would extend the
through which its gas enters Germany. temporary suspension of gas supplies in
retaliation against the current sanctions
 It transports 55 billion cubic metres of leveled against Moscow.
gas a year, of which most goes directly to
Germany, while the rest travels west and  If Nord Stream 1 does not resume its supply
southwards through onshore links to other to Europe, it will not have adequate gas
countries and into storage caverns. supply by the end of the year.

 Germany is Russia’s biggest European gas  As an alternative source for energy,


consumer, and most of it comes through the European countries have increasingly
Nord Stream Pipeline. Its share of Russian turned towards the US, from whom they
gas supplies was 55% in 2021, and currently purchase liquefied natural gas (LNG) that
lies at 35%. comes via ships. Since ship delivered gas
ends up being far more expensive, there are
 Nord Stream 2 is a 1200-km long Russian also attempts to get non-Russian pipeline
undersea gas pipeline project that intends gas from Norway and Azerbaijan.
to deliver natural gas from Siberia to
Germany.
11. Correct Option: (b)
 It runs from Ust-Luga in Russia to
Greifswald in Germany through the Baltic Explanation:
Sea.
 Statement 2 is incorrect: According to a
 The pipelines will double supplies of cheap 2020 Supreme Court judgment, co-operative
natural gas from Russia to Germany from banks can also invoke SARFAESI Act.

PTS2023/PPP-01/092022/11 8

Telegram - @upscmaterialonline1
.
Supplementary notes: corporate debtor liquidation (which falls
under the jurisdiction of the NCLT) and
SARFAESI Act individual and company liquidation
 The Securitization and Reconstruction (which falls under the jurisdiction of the
of Financial Assets and Enforcement of DRT), while the same has not been done
Security Interest (SARFAESI) Act, 2002 was by the SARFAESI Act.
brought in to guard financial institutions
Why in News?
against loan defaulters. To recover their
bad debts, the banks under this law can  Banks have invoked the Securitization
take control of securities pledged against and Reconstruction of Financial Assets
the loan, manage or sell them to recover and Enforcement of Security Interest
dues without court intervention. (SARFAESI) Act against telecom
infrastructure provider GTL to recover their
 The law is applicable throughout the country
pending dues.
and covers all assets, movable or immovable,
promised as security to the lender.  The recovery action has been initiated by
IDBI Bank on behalf of lenders, whose total
 Before the law was enacted in December
exposure to GTL stood at Rs.7,250 crore as
2002, banks and other financial institutions
of December, 2021.
were forced to take a lengthy route to recover

RE
their bad debts. The lenders would appeal
in civil courts or designated tribunals to 12. Correct Option: (a)
get hold of ‘security interests’ to recovery
of defaulting loans, which in turn made the Explanation:
recovery slow and added to the growing list  Option (a) is correct: The X-ray
of lender’s non-performing assets. Polarimeter Satellite is a ISRO planned
 The Act comes into play if a borrower O space observatory to study polarisation of
defaults on his or her payments for more cosmic X-rays. It is planned to be launched
than six months. The lender then can send in second Quarter of 2023 on a Small
a notice to the borrower to clear the dues Satellite Launch Vehicle, with mission life
within 60 days. In case that doesn’t happen, of at least five years.
the financial institution has the right to
SC
Supplementary notes:
take possession of the secured assets and
sell, transfer or manage them. ISRO Missions
 The defaulter, meanwhile, has recourse to  To Study how radiation is polarised gives
move an appellate authority set up under away the nature of its source, including the
the law within 30 days of receiving a strength and distribution of its magnetic
notice from the lender. According to a 2020 fields and the nature of other radiation
Supreme Court judgment, co-operative around it.
GS

banks can also invoke SARFAESI Act.


 XPoSat will study the 50 brightest
According to the Finance Ministry, the non-
known sources in the universe,
banking financial companies (NBFCs) can
including pulsars, black hole X-ray
initiate recovery in Rs 20 lakh loan default
binaries, active galactic nuclei, and non-
cases.
thermal supernova remnants.
 SARFAESI Act is different from IBC in the
 The Indian Space Research Organisation
following ways:
(ISRO) has set new deadlines for major
 The SARFAESI Act protects financial missions, with its first solar mission and
creditors, who are mostly banks and third lunar mission set to take place in the
other financial institutions, by allowing first quarter of the year 2023.
them to impose their security interests
 The space agency’s third scientific mission
without the need for court interference.
scheduled for next year is the space
The IBC, on the other hand, guarantees
observatory, XpoSat, designed to study
the interests of all forms of creditors,
cosmic x-rays. The XpoSat will be India’s
which are further divided into Financial
second astronomical observatory in space
and Operational Creditors by the IBC.
after the Astrosat.
 IBC takes precedence over SARFAESI
 ISRO would also carry out a ‘space docking
during the Insolvency Resolution
experiment’ in the third quarter of 2024.
Process.
Space docking is a process of joining two
 Individuals and unincorporated separately launched spacecraft, and is
companies are handled differently under mainly used for setting up modular space
the IBC, which distinguishes between stations.

9
PTS2023/PPP-01/092022/11

Telegram - @upscmaterialonline1
.
Why in News? manufacturing Logic / Memory / Digital
ICs / Analog ICs / Mixed Signal ICs /
 The Indian Space Research Organization
SoCs will be eligible.
(ISRO) has recently set new deadlines for
its solar and lunar missions.  India Semiconductor Mission (ISM) has been
setup as an Independent Business Division
 All these scientific missions have been
repeatedly pushed back since 2020 amidst within Digital India Corporation having
the pandemic that slowed down all activities administrative and financial autonomy
of the space agency, including the number to formulate and drive India’s long term
of launches. strategies for developing semiconductors
and display manufacturing facilities
and semiconductor design ecosystem.
13. Correct Option: (a) Envisioned to be led by global experts in the
Semiconductor and Display industry, ISM
Explanation: will serve as the nodal agency for efficient,
 Statement 2 is incorrect: ISM has been coherent and smooth implementation of the
setup as a specialized and independent schemes.
business division of ‘Digital India
 Design Linked Incentive (DLI) Scheme
Corporation’. C-DAC (Centre for
under the India Semiconductor Mission
Development of Advanced Computing)

RE
(ISM) offers financial incentives, design
is the nodal agency for implementation
of the Design Linked Incentive (DLI) infrastructure support across various
scheme. stages of development and deployment of
semiconductor design for Integrated Circuits
Supplementary notes: (ICs), Chipsets, System on Chips (SoCs),
Systems & IP Cores and semiconductor
New Semiconductor Policy
linked design. The scheme provides
 The Ministry of Electronics and Information “Product Design Linked Incentive” of up to
O
Technology unveiled the new semiconductor 50% of the eligible expenditure subject to a
policy. Key provisions are: ceiling of Rs. 15 Crore per application and
“Deployment Linked Incentive” of 6% to 4%
 Government will provide up to 50% of
Project cost for two semiconductor and of net sales turnover over 5 years subject
to a ceiling of Rs. 30 Crore per application.
SC
two display fabs.
C-DAC (Centre for Development of Advanced
 Support shall be provided for a period Computing), a scientific society operating
of six years. The tenure of the actual under MeitY, will serve as the nodal agency
fiscal support outflow may be extended for implementation of the DLI scheme.
based on the approval of the Minister
of Electronics and Information Why in News?
Technology.  India, which currently imports 100 per cent
GS

 Further, for additional support, the of its chips, was hit hard by the worldwide
government will offer infrastructure chip shortage, exacerbated by COVID and
support through “Modified Electronics the Russia-Ukraine conflict. Like the rest
Manufacturing Clusters (EMC 2.0) of the world, the country is keen to achieve
Scheme for development of infrastructure some chip self-sufficiency.
/ Common Facility Centre subject to
the proposal satisfying the EMC 2.0
framework requirements. 14. Correct Option: (b)
 The semiconductor fabs set up in India Explanation:
will be supported through purchase
 Statement 3 is incorrect: Cloud bursts
preference in procurement of electronic
do occur at plains, however, mountainous
products by the Government under the
Public Procurement (Preference to Make regions are more prone to cloud bursts due
in India) Order 2017. to orography.

 Additionally, up to 5% of the outlay Supplementary notes:


of the scheme shall be earmarked for Cloudburst
meeting the research and development
(R&D), skill development and training  A cloudburst refers to an extreme amount
requirements for the development of of rain that happens in a short period,
semiconductor ecosystem in India. sometimes accompanied by hail and
thunder.
 Companies / Consortia / Joint Ventures
proposing to set up a Silicon CMOS  The India Meteorological Department
based Semiconductor Fab in India for (IMD) defines it as unexpected precipitation

PTS2023/PPP-01/092022/11 10

Telegram - @upscmaterialonline1
.
exceeding 100mm (or 10 cm) per hour over a of the United States but have also been
geographical region of approximately 20 to documented elsewhere across the world. In
30 square km. 2010, Russia witnessed its first documented
derecho. They have also swept through
 Significant amounts of rainfall such as this
can result in floods. Germany and Finland, and more recently in
Bulgaria and Poland.
 Basically, all instances of cloudbursts
involve heavy rain in a short period, but Supplementary notes:
all instances of heavy rain in a short period
Derecho
are not cloudbursts if they do not fit this
criterion.  A derecho, is “a widespread, long-lived,
straight-line windstorm” that is associated
 It is difficult to predict when exactly a
with a “band of rapidly moving showers or
cloudburst will occur, and there is little
thunderstorms”.
definitive data on the exact number of
cloudbursts that occur in India. Due to their  The name comes from the Spanish word ‘la
definition dealing with a very small area, it derecha’ which means ‘straight’.
is difficult to accurately predict and identify
cloudbursts immediately. However, they are  Straight-line storms are those in which
more likely to occur in mountainous zones thunderstorm winds have no rotation unlike
a tornado. These storms travel hundreds of

RE
mainly because of terrain and elevation.
This is because, in hilly areas, sometimes miles and cover a vast area.
saturated clouds ready to condense into  Being a warm-weather phenomenon, a
rain cannot produce rain, due to the upward derecho generally – not always – occurs
movement of the very warm current of air. during summertime beginning May, with
Instead of falling downwards, raindrops are most hitting in June and July. However,
carried upwards by the air current. New
they are a rare occurrence as compared
drops are formed and existing raindrops
to other storm systems like tornadoes or
O
increase in size. After a point, the raindrops
become too heavy for the cloud to hold on
to, and they drop down together in a quick
flash.

hurricanes.
For a storm to be classified as a derecho it
must have wind gusts of at least 93 km per
hour; wind damage swath extending more
SC
 A study published in 2020 examined the
meteorological factors behind the cloudburst than 400 km.
over the Kedarnath region, where a  Severe thunderstorms result in a ‘green
cloudburst aided the devastating 2013 sky’ due to light interacting with the huge
floods. It found that during a cloudburst, amount of water they hold. It is believed
the relative humidity and cloud cover was that the big raindrops and hail scatter
at the maximum level with low temperature away all but the blue wavelengths due to
and slow winds. “It is expected that because which primarily blue light penetrates below
GS

of this situation a high amount of clouds the storm cloud. This blue then combines
may get condensed at a very rapid rate and with the red-yellow of the afternoon or the
result in a cloudburst.
evening sun to produce green.
Why in News?  Derechos fall into three categories –
 Sudden, “highly-localised rains” in progressive, serial and hybrid.
Amarnath, Jammu and Kashmir, recently  A progressive derecho is associated with
caused flooding and led to the deaths of at
a short line of thunderstorms that may
least 16 people and injuries to more than 20
travel for hundreds of miles along a
others.
relatively narrow path. It is a summer
phenomenon.
15. Correct Option: (d)  A serial derecho, on the other hand, has
Explanation: an extensive squall line – wide and long
– sweeping across a large area. It usually
 Statement 1 is incorrect: A derecho, is occurs during spring or fall.
“a widespread, long-lived, straight-line
windstorm” that is associated with a “band of  Hybrid ones have the features of both
rapidly moving showers or thunderstorms”. progressive and serial derechos.
Straight-line storms are those in which  Derechos mostly occur across central and
thunderstorm winds have no rotation unlike
eastern parts of the United States but have
a tornado.
also been documented elsewhere across the
 Statement 3 is incorrect: Derechos mostly world. In 2010, Russia witnessed its first
occur across central and eastern parts documented derecho. They have also swept

11
PTS2023/PPP-01/092022/11

Telegram - @upscmaterialonline1
.
through Germany and Finland, and more  Hence higher courts cannot define rules for
recently in Bulgaria and Poland. lower courts, as per the Panel.
Why in News?  Clause 26 of the Bill: It provides for the
 Recently, the states of Nebraska, Minnesota court-annexed mediation. This clause gives
and Illinois in the US were hit by a storm the powers to the court to make rules for
system called a derecho. ‘court-annexed mediation’, which is
unconstitutional.
 As the storm rolled in, winds gusting at
around 140 km per hour, snapped power  Pre-litigation mediation mandatory:
lines and knocked down trees. As the storm Making pre-litigation mediation mandatory
hit, it turned the skies green. may actually result in the delaying of cases
and may prove to be an additional tool in
the hands of truant litigants to delay the
16. Correct Option: (d) disposal of cases.
Explanation:  Non-applicability of provisions to non-
 Option (d) is correct commercial matters: Non Applicability
of the provisions of the Bill to disputes/
Supplementary notes: matters of non-commercial nature involving

RE
Mediation Bill the Government and its agencies.

 The panel has cautioned against making  Appointment of chairperson and


pre-litigation mediation compulsory members: Appointment of the Chairperson
and warned the Centre against the and Members of the Mediation Council
provision to give higher courts the power to of India should be made by a Selection
frame rules for mediation. Committee constituted by the Centre.
O
SC
GS

PTS2023/PPP-01/092022/11 12

Telegram - @upscmaterialonline1
.
Why in News? whether designated as reserved, protected
or otherwise for the purpose of Section 2
 The Parliamentary Standing
(1) of the Act and also includes any areas
Committee on Law and Justice, headed
recorded as forest in the government record
by Sushil Kumar Modi, has recommended
irrespective of the ownership. The provisions
substantial changes to the Mediation
for the conservation of forest and the
Bill, meant for the institutionalization
matters connected therewith apply clearly
of mediation and establishment of the
to all forests irrespective of the ownership
Mediation Council of India.
or classification.
 Thus the freedom to define which tracts
17. Correct Option: (b) of forest qualify as forest has been the
Explanation: prerogative of States since 1996.

 Statement 1 is incorrect: In India, the  The Forest Conservation Rules deal with the
freedom to define land as forest, not already implementation of the Forest Conservation
classified as forest in the Central or state Act (FCA), 1980. They prescribe the
records, is the prerogative of states. procedure to be followed for forest land to
be diverted for non-forestry uses such as
 Statement 3 is incorrect: The concept of road construction, highway development,
deemed forests has not even been clearly

RE
railway lines, and mining.
defined in any law including the Forest
Conservation Act 1980. Why in News?
Supplementary notes:  The Congress and the Bharatiya Janata
Party (BJP) recently sparred on the latest
Forest Conservation Act, 1980 version of the Forest Conservation Rules.
 The act aims to provide for the conservation  The new rules, according to the Centre,
of forests and its resources. The broad aims
O
of the Forest Conservation Act are to protect
forest and wildlife, put brakes on State
governments’ attempts to hive off forest
“streamline” the process of approvals. The
rules make a provision for private parties to
cultivate plantations and sell them as land to
companies who need to meet compensatory
land for commercial projects and striving to forestation targets. This will help India
SC
increase the area under forests.
increase forest cover as well as solve the
 As per this act, no State Government or problems of the States of not finding land
other authority shall make, except with the within their jurisdiction for compensatory
prior approval of the Central Government, purposes.
any order directing that any reserved forest
 The latest point of contention is the absence of
or any portion thereof, shall cease to be
wording, in the updated Forest Conservation
reserved and that any forest land or any
Rules, of what happens to tribals and forest-
portion thereof may be used for any non-
dwelling communities whose land would
GS

forest purpose. “Non-forest purpose” means


the breaking up or clearing of any forest be hived off for developmental work. Prior
land or portion thereof for any purpose to the updated rules, state bodies would
other than reforestation. forward documents to the FAC that would
also include information on the status of
 The Central Government may constitute whether the forest rights of locals in the
a Committee (Forest Advisory Committee) area were settled.
consisting of such number of persons as it
may deem fit to advise that Government
with regard to the grant such an approval. 18. Correct option: (c)
 The Act gives complete authority to the Explanation:
Central government to carry out the
objectives of the act.  Both statements are correct

 The Act levies penalties in case of violations Supplementary notes:


of the provisions of FCA. Information Technology Act, 2000
 The FCA does not clearly define the concept  The Information Technology Act, 2000
of deemed forests. In the T N Godavarman was enacted by the Indian Parliament in
Thirumalpad case 1996, the Supreme
2000. It is the primary law in India for
Court (SC) accepted a wide definition of
matters related to cybercrime and
forests under the Act and held that the
e-commerce.
word ‘forest’ must be understood according
to its dictionary meaning. This description  The act was enacted to give legal sanction
covers all statutorily recognized forests, to electronic commerce and electronic

13
PTS2023/PPP-01/092022/11

Telegram - @upscmaterialonline1
.
transactions, to enable e-governance, and  Shrinkflation is a rise in the general price
also to prevent cybercrime. level of goods per unit of weight or volume,
brought about by a reduction in the weight
 Under this law, for any crime involving
or size of the item sold.
a computer or a network located in
India, foreign nationals can also be  The price for one piece of the packaged
charged. The law prescribes penalties for product remains the same or could even be
various cybercrimes and fraud through raised.
digital/electronic format.
 This sometimes does not affect inflation
 It also gives legal recognition to digital measures such as the consumer price
signatures. index or Retail Price Index, i.e. might not
increase in the cost of a basket of retail
 Section 69A grants to the government the
goods and services but many indicators of
power to issue directions to intermediaries
price levels and thus inflation are linked to
for blocking access to any information that
units of volume or weight of products, so that
it considers prejudicial to, among other
shrinkflation also affects the statistically
things, the sovereignty and integrity of
represented inflation figures.
India, national security, or public order.
 Section 69A (3) envisages a jail sentence Why in News?

RE
for up to seven years for intermediaries who  As India’s inflation surges, the increase
fail to comply. in prices of commodities in the market is
 In 2009, the government also issued hitting the common man for several months
“Blocking Rules”, which set up the in the country.
procedure for blocking (including regular
review by government committees), and 20. Correct Option: (a)
also stated that all requests and complaints
would remain strictly confidential.
O Explanation:
Why in News?  Option (a) is correct

 There is a growing push and pull between Supplementary notes:


Big Tech and the state over the governance
SC
Dark Matter
of global public platforms.
 All interactions in the universe are a
 The social media platform, Twitter,
result of four fundamental forces acting on
moved the Karnataka High Court over
particles — strong nuclear force, weak
the government’s orders to block certain
nuclear force, electromagnetic force
tweets and handles under Section 69 of the
and gravitation.
Information Technology Act 2000.
 Dark matter is made up of particles that
GS

do not have a charge — which means they


19. Correct option: (a) do not interact through electromagnetic
Explanation: interactions.

 Option (a) is correct  So, these are particles that are “dark”,
namely because they do not emit light,
Supplementary notes: which is an electromagnetic phenomenon,
and “matter” because they possess mass like
Shrinkflation
normal matter and hence interact through
 In economics, shrinkflation, also gravity.
known as the grocery shrink
 The existence of dark matter can be traced
ray, deflation or package downsizing, is
back to the pioneering discoveries of Fritz
the process of items shrinking in size or
Zwicky and Jan Oort that the motion of
quantity, or even sometimes reformulating
galaxies in the Coma cluster, and of nearby
or reducing quality, while their prices
stars in our own Galaxy, do not follow the
remain the same or increase.
expected motion based on Newton’s law of
 Shrinkflation allows companies to increase gravity and the observed visible masses.
their operating margin and profitability
 Since then a host of experimental data
by reducing costs whilst maintaining sales
from precise measurements of the cosmic
volume, and is often used as an alternative
microwave background, of gravitational
to raising prices in line with inflation.
lensing of galaxy clusters, and of the
 Consumer protection groups are rotational speeds of stars and galaxies,
critical of the practice. provide strong, internally consistent,

PTS2023/PPP-01/092022/11 14

Telegram - @upscmaterialonline1
.
evidence for the existence of dark matter as What does it say about gas and nuclear
a particle. energy?
 The Chandra X-ray Observatory (CXO),  The European Commission made a
previously known as the Advanced proposal in February 2022 to add gas and
X-ray Astrophysics Facility (AXAF), nuclear power plants to the taxonomy if
is a Flagship-class space they meet certain criteria. The European
telescope launched aboard the Space Parliament supported that proposal in a
Shuttle Columbia during STS- vote on Wednesday, paving the way for it
93 by NASA on July 23, 1999. to become law and apply from 2023. Twenty
 Chandra is sensitive to X-ray sources 100 of the EU’s 27 member states could veto the
times fainter than any previous X-ray proposal, but that is seen as unlikely.
telescope, enabled by the high angular
 Under the Commission proposal, for a gas-
resolution of its mirrors.
fuelled power plant to be deemed green,
 Since the Earth’s atmosphere absorbs it must emit no more than 270 grams of
the vast majority of X-rays, they are not CO2 equivalent per kilowatt hour, or have
detectable from Earth-based telescopes; average emissions of 550g CO2e/kW over 20
therefore space-based telescopes are years. It must also commit to switch to low-
required to make these observations. carbon gases by 2035.

RE
 Chandra is an Earth satellite in a 64-hour  The Commission’s original proposal for gas,
orbit, and its mission is on-going as of published in late 2020, had included a lower
2022. 100g CO2 limit. It was amended following
 NASA claimed to have the direct backlash from countries including Poland
information regarding Dark matter using and Bulgaria, who say gas investments are
the observatory. O needed to quit more-polluting coal. Others,
such as Denmark and Luxembourg, say it
Why in News? is not credible to label gas, a fossil fuel, as
 Many physicists strongly believe that the green.
entire visible part of the universe forms
Why in News?
only 5% of all matter in it. They believe the
SC
rest is made up of dark matter and dark  It is considered as the world’s most ambitious
energy. Once this was convincingly green investment rulebook and could direct
demonstrated through various indirect huge sums of money into fighting climate
observations and calculations, experiments change. However, the Critics say it’s a
started being set up to hunt for these elusive “green washing” exercise that puts the
particles. European Union’s climate change targets at
risk.
GS

21. Correct option: (d)


Explanation:
22. Correct Option: (c)
 Option (d) is correct Explanation:

Supplementary notes:  Statement 1 is correct: In the landmark


case of K.S. Puttaswamy v. Union of
EU taxonomy India, the Supreme Court recognised the
 The EU taxonomy is a complex system to right to be forgotten as part of the right to
classify which parts of the economy may life under Article 21.
be marketed as sustainable investments.  Statement 2 is correct: The proposed
It includes economic activities, as well as Personal Data Protection Bill introduces
detailed environmental criteria that each the concept of right to be forgotten.
economic activity must meet to earn a green
label. Rules for most sectors came into effect  The draft law envisages creation of an
this year, covering investments including authority where one can apply to restrict
steel plants, electric cars and building dissemination of information that may
renovations. be private in nature and is sensitive,
impacting one’s life. But till now it has not
 The rules for gas and nuclear energy,
been envisaged.
however, have been long delayed amid
intense lobbying from governments who  India, at present does not have any
disagree on whether the fuels help fight statutory provision that provides for
climate change. ‘Right to be forgotten (RTBF)’.

15
PTS2023/PPP-01/092022/11

Telegram - @upscmaterialonline1
.
Supplementary notes: of litigants entangled in matrimonial
litigation.
Right to be forgotten
 A bench led by Justice Sanjay Kishan Kaul
 The right to be forgotten refers to the right saw merit in the woman’s plea, which
to have private information removed from said her personal information, such as her
the internet under certain circumstances. husband’s name and residential address
It proceeds on the basis that an individual should either be deleted or masked from the
must decide the course of their life in judgment of her case that is now uploaded
an autonomous manner, without being on the SC website, from where it has been
‘stigmatised’ due to an action of the past. shared by different online platforms.
Likewise, a traceable procedure must be in
place to ensure that removed data is also
erased from backup storage media. 23. Correct Option: (b)
 The right to privacy has been recognised Explanation:
implicitly in Article 21 (the right to life
and liberty). In Puttaswamy, the Supreme  Statement 2 is incorrect: It will receive
Court had noted the existence of such a IP-1 license from the Department of
right. Referring to a regulation which had Telecommunications (DoT), KFON can
allowed the Unique Identification Authority lease out, rent out or sell the equipment

RE
of India (UIDAI) to retain authentication to other providers. Moreover, with the
transaction data for six months and archive ISP license, it can now provide Internet
the same for five years, the court noted the connection as well. So, it will not provide
its own internet services.
infringement of such a right.
 There is an intricate system envisaged Supplementary notes:
under the Section 20 of Personal Data Kerala Fibre Optic Network (KFON)
Protection Bill (PDP) for setting off the
O
right to be forgotten. The Bill articulates  KFON project is aimed to provide Internet
that the right can be sanctioned only on connections to 20 lakh below-poverty-line
the order of an adjudicating officer after an (BPL) households in the State, and ensure
application recorded by the data principal. universal Internet access and address the
problem of digital divide.
SC
 Whereas, the choice on whether the right
to be forgotten can be granted with respect  KFON Ltd is primarily an infrastructure
to any information will rely upon “the provider for other ISPs operating in the
right to the right to freedom of speech and State.
expression and the right to information of  Under the project, a venture of equal
some other citizen”. partnership between Kerala State
 The Supreme Court had stated that the Information Technology Infrastructure Ltd
right to be forgotten was subject to certain (KSITIL) and Kerala State Electricity Board
GS

restrictions, and that it could not be used (KSEB), 30,000 km of fibre optic cable will
if the material in question was required for be laid connecting all parts of the State. The
the— cable is being laid using the existing network
of KSEB that has a network capacity of 2
 Exercise of the right to freedom of lakh km in the State.
expression and information;
 ISPs can lease the KFON network to
 Fulfillment of legal responsibilities; provide Internet service to customers across
 Execution of a duty in the public interest the State at an affordable rate. It will also
or public health; provide free high-speed Internet connectivity
to over 20 lakh financially disadvantaged
 Protection of information in the public people, and 30,000 government officers and
interest; educational institutions.
 For the purpose of scientific or historical  However, KFON will also be offering its
study, or for statistical purposes; or own Internet service.
 The establishment, executing, or  The initiative also aims to augment the
defending of legal claims. launch of 5G services in the state. At
present, 80% of the towers are not fiberized,
Why in News?
an issue for the 4G to 5G transition. High-
 Recognizing ‘right to be forgotten’ as part speed wireless connectivity (3G/4G) is
of ‘right to privacy’, the Supreme Court limited in rural areas as more telecom towers
recently ordered its registry to work out need to be set-up to provide such services.
a mechanism to remove personal details Moreover, the majority of the existing

PTS2023/PPP-01/092022/11 16

Telegram - @upscmaterialonline1
.
towers are connected via radio, which limits are a sub-family of venomous vipers found
the bandwidth that can be provided to the in Eurasia and the Americas.
public. KFON is expected to solve this issue
 They are distinguished by the presence of
by connecting the nearly 8,000+ mobile
a heat-sensing pit organ located between
towers in Kerala and significantly enhance
the eye and the nostril on both sides of the
the mobile call quality and accelerate the
head.
transition to 5G.
 The common krait (Bungarus caeruleus),
 KFON has shortlisted six ISPs to provide
also known as Bengali krait, is
internet connection to 14,000 BPL families.
a species of highly venomous snake of
A total of 100 BPL families in each of the
the genus Bungarus native to the Indian
State’s 140 assembly constituencies are
subcontinent.
being identified in the first phase through
the respective local bodies to provide the  It is a member of the “Big Four” species
free Internet connection. that inflict the most snake bites on humans
 K-FON will contribute substantially to in Pakistan, India and Bangladesh.
reducing the digital divide and scale-up  Reticulated Python was not common in
e-governance in the State. It would also India but is found in the Andaman and
make Internet access more affordable as the Nicobar islands.
State Government is effectively subsidizing

RE
the infrastructure cost for ISPs and cable  The reticulated python (Malayopython
operators who can provide last mile reticulatus) is a python species native
Internet connectivity, and thus facilitating to South and Southeast Asia.
competition. Further, the extensive fibre  It is the world’s longest snake, and
optic network is also expected to greatly listed as least concern on the IUCN Red
improve the Internet speed, especially in List because of its wide distribution.
rural areas. O  In several countries in its range, it is hunted
 All these would in turn help boost for its skin, for use in traditional medicine,
economic growth by providing robust and for sale as pets.
digital infrastructure support required for
industrial growth, education, healthcare, Why in News?
SC
skill development, smart urban and transport  The Pilikula Park had brought five
management, and rural engagement via Reticulated Pythons from Chennai about
concepts such as smart village. five years ago. Of them, one python had laid
Why in News? seven eggs for the first time in the park.
Later, another one laid 25 eggs.
 Having recently received both the
Infrastructure Provider and Internet  Reticulated Python was under threat owing
Service Provider (ISP) licenses from the to the high demand for its skin as it was
GS

Central government, the Kerala Fibre Optic hunted.


Network (KFON) is just a few bureaucratic  Malabar Pit Viper, Stump Nosed Pit Viper,
steps away from reaching its beneficiaries. Common Krait and many other species of
 Having received the IP-1 license from the snakes have bred under captivity in the
Department of Telecommunications (DoT), park.
KFON can lease out, rent out or sell the
equipment to other providers. Moreover,
with the ISP license, it can now provide
25. Correct Option: (c)
Internet connection as well. Explanation:
 Both statements are correct
24. Correct Option: (c)
Supplementary notes:
Explanation:
Mission Vatsalya Scheme
 Option (c) is correct
 It focuses on Child Protection Services
Supplementary notes: and child welfare services.
 Russell’s viper is a venomous snake in  It is essentially a renamed version of
the family Viperidae native to the Indian the pre-existing scheme called Child
subcontinent and one of the big four snakes Protection Services.
in India.
 It will include statutory bodies; service
 The Crotalinae, commonly known as pit delivery structures; institutional care/
vipers, crotaline snakes or pit adders, services; non-institutional community-

17
PTS2023/PPP-01/092022/11

Telegram - @upscmaterialonline1
.
based care; emergency outreach services  It maps the volume of production in the
(through Childline or the national helpline eight most fundamental industries of our
1098 for children); training and capacity economy.
building.
 Petroleum Refinery (weight: 28.04 per cent)
 Implementation: has the highest weightage in the index of
eight core industries followed by Electricity
 Under the mission, the Government generation (weight: 19.85 per cent), Steel
plans to partner with the private sector (weight: 17.92 per cent), Coal (weight: 10.33
as well as volunteer groups for its scheme per cent), Crude Oil Production (weight:
for the protection of vulnerable children 8.98 per cent), Natural Gas (weight: 6.88
such as those abandoned or missing. per cent), Cement Production (weight: 5.37
 For this, a Vatsalya portal will be per cent) and Fertilizers (weight: 2.63 per
developed that will allow volunteers cent) respectively.
to register so that State and District  These core industries consist of 40.27 per
Authorities can engage them in executing cent of the total weight of items included in
various schemes. the Index of Industrial Production (IIP).
Objectives of the Mission:  At present, the base year for the Index of
 To secure a healthy and happy childhood for Eight Core Industries (ICI) index is 2011-

RE
every child in India. 12.

 To foster a sensitive, supportive and Why in News?


synchronized ecosystem for the development  The combined Index of Eight Core Industries
of children. recently increased by 12.7 per cent
 To assist States/UTs in delivering the (provisional) as compared to the Index of
mandate of the Juvenile Justice Act 2015. June 2021. The production of Coal, Cement,
Electricity, Refinery Products, Fertilizers,
 To achieve the SDG goals.
O Steel, and Natural Gas industries increased
in June 2022 over the corresponding period
Why in News?
of last year.
 The Ministry of Women and Child
SC
Development has sent its draft guidelines
for Mission Vatsalya Scheme to States and 27. Correct Option: (c)
Union Territories to seek their suggestions. Explanation:
 Mission Vatsalya is one of the new triad  Option (c) is correct
of schemes along with Mission Shakti,
and Poshan 2.0, that aims at securing Supplementary notes:
a healthy and happy childhood for every National Institutional Ranking
child.
Framework (NIRF)
GS

 The NIRF outlines a methodology to


26. Correct option: (b) rank institutions across the country.
The methodology draws from the overall
Explanation:
recommendations of broad understanding
 Option (b) is correct: Petroleum Refinery arrived at by a core committee set up by
(weight: 28.04 per cent) has the highest MHRD to identify the parameters for ranking
weightage in the index of eight core various universities and institutions.
industries followed by Electricity generation
 The parameters broadly cover “Teaching,
(weight: 19.85 per cent), Steel (weight:
Learning and Resources,” “Research and
17.92 per cent), Coal (weight: 10.33 per
Professional Practices,” “Graduation
cent), Crude Oil Production (weight: 8.98 Outcomes,” “Outreach and Inclusivity,”
per cent), Natural Gas (weight: 6.88 per and “Perception”. These parameters have
cent), Cement Production (weight: 5.37 per been grouped into five clusters and these
cent) and Fertilizers (weight: 2.63 per cent) clusters were assigned certain weightages.
respectively. The weightages depend on the type of
Supplementary notes: institution.
 Depending on their areas of operation,
Index of Eight Core Industries
institutions have been ranked under 11
 Office of the Economic Adviser, Department different categories – overall, university,
for Promotion of Industry and Internal colleges, engineering, management,
Trade is responsible for the release of the pharmacy, law, medical, architecture,
Index of Eight Core Industries. dental and research.

PTS2023/PPP-01/092022/11 18

Telegram - @upscmaterialonline1
.
 The first NIRF ratings were released in  MIDH is implemented under Green
2016. Revolution - Krishonnati Yojana.
 The National Board of Accreditation (NBA)  Funding Pattern: Under MIDH,
is the Ranking Agency on behalf of NIRF. Government of India (GoI) contributes
60% of total outlay for developmental
 NIRF is the first-ever effort by the
government to rank higher education programmes in all the states except states
institutions (HEIs) in the country. Before in North East and Himalayas, 40% share is
NIRF’s launch in 2016, HEIs were usually contributed by State Governments.
ranked by private entities, especially news  In the case of North Eastern States and
magazines. Himalayan States, GoI contributes 90%.
 Participation in the rankings is currently Why in News?
voluntary.
 Following the footsteps of the Gujarat and
Why in News? Haryana Governments, the Centre has
 The Indian Institute of Technology (IIT) decided to promote the cultivation of dragon
Madras stayed on top of the latest National fruit, known as a “super fruit” for its health
Institutional Ranking Framework (NIRF) benefits.
rankings for the fourth consecutive year.  The Centre feels that considering the cost

RE
 IIT was followed in the overall ranking effectiveness and global demand for the fruit
by the Indian Institute of Science (IISc) due to its nutritional values, its cultivation
Bengaluru and IIT Bombay in the seventh can be expanded in India.
edition of the rankings released recently.  At present, this exotic fruit is cultivated
 IISc also topped the universities and in 3,000 hectares; the plan is to increase
research institutions category, Delhi’s cultivation to 50,000 hectares in five years.
Miranda House, the colleges category, IIT
O
Madras, the engineering category, the Indian
Institute of Ahmedabad, the management
schools category, the All India Institute of 29. Correct Option: (b)
Medical Sciences, Delhi, the medical colleges Explanation:
SC
category, and the National Law School of
India, Bengaluru, the law schools category.  Statement 1 is incorrect: The writ of
NIRF also ranks pharmacy, dental, and ‘prohibition’ is issued by a higher court to a
architecture colleges. lower court or tribunal to prevent the latter
from exceeding its jurisdiction or usurping a
 The first non-engineering institute in the
jurisdiction that it does not possess.
ranking is AIIMS, at 9; Delhi’s Jawaharlal
Nehru University is in 10th rank. Supplementary notes:
GS

Writs of Prohibition and Certiorari


28. Correct Option: (b)  Prohibition: Literally, it means ‘to forbid’.
Explanation: It is issued by a higher court to a lower
court or tribunal to prevent the latter from
 Statement 1 is incorrect: MIDH exceeding its jurisdiction or usurping a
is a scheme for the holistic growth of
jurisdiction that it does not possess. Thus,
the horticulture sector covering fruits,
unlike mandamus that directs activity,
vegetables, root & tuber crops, mushrooms,
the prohibition directs inactivity. The writ
spices, flowers, aromatic plants, coconut,
of prohibition can be issued only against
cashew, cocoa and bamboo with exotic
judicial and quasi-judicial authorities. It
species.
is not available against administrative
Supplementary notes: authorities, legislative bodies, and private
individuals or bodies.
MIDH
 Certiorari: In the literal sense, it means
 MIDH is a Centrally Sponsored Scheme ‘to be certified’ or ‘to be informed’. It is
for the holistic growth of the horticulture issued by a higher court to a lower court or
sector covering fruits, vegetables, root & tribunal either to transfer a case pending
tuber crops, mushrooms, spices, flowers, with the latter to itself or to squash the
aromatic plants, coconut, cashew, cocoa and
order of the latter in a case. It is issued on
bamboo.
the grounds of excess of jurisdiction or lack
 Nodal Ministry: The Ministry of Agriculture of jurisdiction or error of law. Thus, unlike
and Farmers Welfare is implementing prohibition, which is only preventive,
MIDH with effect from 2014-15. certiorari is both preventive as well as

19
PTS2023/PPP-01/092022/11

Telegram - @upscmaterialonline1
.
curative. Previously, the writ of certiorari 2020 had led to Israel formally normalising
could be issued only against judicial and diplomatic ties with the UAE and two
quasi-judicial authorities and not against other countries in the region, marking an
administrative authorities. However, important shift in the stance of West Asian
in 1991, the Supreme Court ruled that countries on Israel.
the certiorari can be issued even against
 It could serve Indian foreign-policy goals:
administrative authorities affecting rights
The arrangement enables New Delhi to
of individuals. Like prohibition, certiorari is
expand its role on the world stage, to
also not available against legislative bodies
engage more deeply in the Middle East, and
and private individuals or bodies.
to strengthen cooperation with the United
Why in News? States.
 The Supreme Court recently decided that the  As with the other Quad, this arrangement
Patna High Court exceeded its jurisdiction is a loose grouping, not an alliance.
by summoning Sahara group chief Subrata
Roy in an anticipatory bail case completely Why in News?
unrelated to him and ordering him to come  Prime Minister Narendra Modi will recently
up with a plan to return investors’ money. participate in the first-ever I2U2 Virtual
Summit along with the heads of state of

RE
Israel, the UAE, and the US.
30. Correct Option: (b)
Explanation:
31. Correct option: (b)
 Statement 1 is incorrect: I2U2 stands for
India, Israel, the UAE, and the US, and was Explanation:
also referred to as the ‘West Asian Quad’ by  Statement 1 is incorrect: Pokkali is a
Ahmed Albanna, Ambassador of the UAE to variety of rice endemic to coastal Kerala,
India.
O and is unique because it can grow in.
 Statement 3 is incorrect: India is a
Supplementary notes:
member of this grouping.
Pokkali Rice
Supplementary notes:
SC
 It is endemic to coastal regions of three
I2U2 Group Kerala districts—Ernakulam, Thrissur and
 I2U2 stands for India, Israel, the UAE, Alappuzha.
and the US, and was also referred to as
 However, while this variety of rice seems
the ‘West Asian Quad’ by Ahmed Albanna,
ideal to farm, several Pokkali farmers allege
Ambassador of the UAE to India.
that they are not receiving adequate support
 Back in October 2021, a meeting of the to keep up with the crop cycle, which is vital
foreign ministers of the four countries had to pokkali farming.
GS

taken place when External Affairs Minister S


Jaishankar was visiting Israel. At that time, Key Features
the grouping was called the ‘International  Pokkali rice is famous for their salinity
Forum for Economic Cooperation’. tolerance gene SalTol QTL and is
 Its stated aim is to discuss “common areas of significant for the International rice
mutual interest, to strengthen the economic improvement programmes for salinity
partnership in trade and investment in our tolerance.
respective regions and beyond”.  The rice is cultivated once a year from April
 Six areas of cooperation have been identified to November, in the low-lying fields adjacent
by the countries mutually, and the aim is to Kerala’s backwaters and the Arabian
to encourage joint investments in water, Sea.
energy, transportation, space, health, and  The fields are mostly submerged in salt
food security. With the help of “private water for a major portion of the year, and
sector capital and expertise”, the countries are used for the cultivation of shrimp during
will look to modernise infrastructure,
this time.
explore low carbon development avenues
for industries, improve public health,  Shrimp cultivation aids the Pokkali rice in
and promote the development of critical providing nutrition, since the paddy does not
emerging and green technologies. require any artificial fertilisers or pesticides
to grow.
 The grouping also points to India’s growing
engagement with countries in West Asia  Pokkali farms are supposed to adhere to a
including Israel. The Abraham Accords of double crop calendar — paddy between April

PTS2023/PPP-01/092022/11 20

Telegram - @upscmaterialonline1
.
and October during the low saline phase of  Eco Sensitive Zones are notified by the
monsoon and saline aquaculture between Ministry of Environment Forests and
November and March when salinity soars. Climate Change. State Governments
to forward proposals for declaration of
 It has got Geographical Indication Certificate
eco-sensitive zones around its Protected
(GI) and Genome Community Award.
Areas. However, only States like Haryana,
Why in News? Gujarat, Mizoram, Meghalaya, Assam,
Goa have forwarded proposals. However,
 Pokkali rice farming in Ernakulam several other States/Union Territories
district of coastal Kerala is under threat have not come forward, perhaps for want of
due to ‘mismanaged’ infrastructure. guidelines in this regard.
 An ESZ could go up to 10 kilometres around
32. Correct Option: (a) a protected area as provided in the Wildlife
Conservation Strategy, 2002.
Explanation:
 Moreover, in case where sensitive corridors,
 Option (a) is correct connectivity and ecologically important
Supplementary notes: patches, crucial for landscape linkage, are
beyond 10 kilometers width, these should
Sashakti be included in the Eco-Sensitive Zones.

RE
 Delhi Police will provide self-defence training Why in News?
to girls, working women and housewives
with a view of empowering them.  Three Indian skimmers, an endangered
bird species on the International Union for
 These classes are conducted free of cost both Conservation of Nature (IUCN) list, were
online and offline. This prepares women to spotted in Nal Sarovar Bird Sanctuary
deal with any situation at any given point in
O (NSBL)—the famous wetland near
case they don’t have immediate help around Ahmedabad. According to birders, although
or when police may take time to reach the spotted before in Gujarat, Indian skimmers
spot. are rare sightings in Nal Sarovar.
Why in News?
SC
 Delhi Police’s Special Police Unit for Women
34. Correct Option: (c)
and Children (SPUWAC) organised the 18th Explanation:
Summer Camp 2022 under its special
initiative ‘Sashakti’ for the purpose of  Both statements are correct
creating awareness about women’s safety. Supplementary notes:
James Webb Telescope
33. Correct option: (c)
GS

 JWST is a general-purpose observatory


Explanation: with a large aperture telescope optimised
for infrared observations and a suite of
 Statement 1 is incorrect: ESZ draws its
state-of-the-art astronomical instruments
power from the Environment Protection Act
capable of addressing many outstanding
of 1986 issues in astronomy.
 Statement 2 is incorrect: An ESZ could go  It was named after former administrator
up to 10 kilometres around a protected area of NASA James E. Webb. It was launched
as provided in the Wildlife Conservation as an international collaboration between
Strategy, 2002. NASA, ESA (European Space Agency) and
Supplementary notes: the Canadian Space Agency. It costs around
$9.7 billion, and is billed as the next-
Ecological Sensitive Zone (ESZ) generation space telescope (NGST).
 The Environment (Protection) Act, 1986 Key features of JWST are-
does not mention the word “Eco-Sensitive
Zones”. However, Section 3(2)(v) of the Act,  JWST will operate in an orbit around
says that Central Government can restrict the Earth-Sun L2 Langrage point, ~ 1.5
areas in which any industries, operations or million kilometres away from Earth. This
makes its operation, pointing and stability
processes or class of industries, operations
requirements much simpler in comparison
or processes shall not be carried out or shall
with HST
be carried out subject to certain safeguards.
Besides Rule 5(1) of the Environment  It orbits around the Earth at an altitude of
(Protection) Rules, 1986. ~570 km above it.

21
PTS2023/PPP-01/092022/11

Telegram - @upscmaterialonline1
.
 The telescope and the instruments will on fir trees. They are 50 – 60 cm long and
operate at the extremely low temperature weigh between three and six kgs.
of -233°C, which prevents the instrument’s
own infrared emission from overwhelming  Red pandas are generally quiet, but
the signals from the astronomical targets. subtle vocalizations—such as squeals and
twitters—can be heard at close proximity.
Potential benefits of the JWST Mission- They are also capable of creating a high-
 The launch of JWST is touted as the next pitched whistle.
big event in study of space events. It has  Red Pandas are listed as endangered, this
the potential to explore following thing in is due to various factors, but the main one
universe- being humans.
 Universe’s first galaxies-  Loss of habitat and poaching has seen
 It is possible to observe different life numbers of Red Pandas in the world drop to
stages of galaxies and stars by observing as low as 10,000.
light emitted by the galaxies in time
 Diet
space.
 Reveal the birth of stars and planets and  Bamboo constitutes 85 to 95 per cent of the
how they die. red panda’s diet. Unlike giant pandas which

RE
feed on nearly every portion of bamboo,
Why in News? red pandas feed selectively on the most
 President Joe Biden and NASA released the nutritious leaf tips and, when available,
clearest-ever image of our young Universe, tender shoots.
dating to the time soon after the Big Bang.
 The first image from the $10 billion James
Webb Space Telescope is the farthest
O
humanity has ever seen in both time and
distance, closer to the dawn of time and the
edge of the universe.
SC
35. Correct Option: (c)
Explanation:
 Option (c) is correct
 Red Panda population is found in the
following states: Why in News?
 West Bengal The Singalila National Park, the highest
GS


 Arunachal Pradesh protected area in West Bengal, launches
programme that aims to release about
 Sikkim 20 Red Pandas in a period of five years.
 Meghalaya
Supplementary notes: 36. Correct Option: (b)
Red Panda Explanation:
 Although sharing the same name as the  Statement 3 is incorrect: Mangroves
Giant Panda, they are only distantly related have thick leaves to minimize water loss.
to them, and are actually more closely
related to raccoons. Supplementary notes:
 Distribution: Most Red Pandas can be found Mangroves
in the temperate forests of the Himalayas
and some high mountain areas of China and  Mangroves are the characteristic littoral
Myanmar (Burma). plant formation of tropical and subtropical
sheltered coastlines with various limiting
 Physical Features: Their fur is long, thick factors like lack of oxygen, high salinity and
and fuzzy to protect it against rain and cold diurnal tidal inundation. They are basically
in its mountainous habitat of up to 5000m evergreen land plants growing on sheltered
altitude. shores, typically on tidal flats, deltas,
 Its colour helps the panda to blend with estuaries, bays, creeks and the barrier
the reddish moss and white lichen growing islands.

PTS2023/PPP-01/092022/11 22

Telegram - @upscmaterialonline1
37. Correct Option: (c)

.
 Mangroves are trees and bushes growing
below the high water level of spring tides
which exhibits remarkable capacity for salt Explanation:
water tolerance (halophytes).  Statement 1 is incorrect: In India, heat
 The best locations are where abundant silt is wave is considered if maximum temperature
brought down by rivers or on the backshore of a station reaches at least 40°C or more for
of accreting sandy beaches. Plains and at least 30°C or more for Hilly
regions.
 They require high solar radiation and have
the ability to absorb fresh water from saline/ Supplementary notes:
brackish water.
Heat Wave
 Mangroves exhibit Viviparity mode of
 Qualitatively, heat wave is a condition of air
reproduction i.e. seeds germinate in the tree
temperature which becomes fatal to human
itself (before falling to the ground). This is
body when exposed. Quantitatively, it is
an adaptive mechanism to overcome the
defined based on the temperature thresholds
problem of germination in saline water.
over a region in terms of actual temperature
 Leaves are thick and succulent with sunken or its departure from normal.
stomata for limiting water loss and contain
 In certain countries it is defined in term of

RE
salt secreting glands.
the heat index based on temperature and
 It produces pneumatophores (areal roots) humidity or based on extreme percentile of
to overcome respiration problem in the the temperatures.
anaerobic soil conditions. Adventitious  In India, heat wave is considered if
roots which emerged from the main trunk maximum temperature of a station reaches
of a tree above ground level are called stilt at least 40°C or more for Plains and at least
roots. O 30°C or more for Hilly regions.
 Mangrove plants have special roots such as  Based on Departure from Normal
prop roots, pneumatophores which help to
impede water flow and thereby enhance the  Heat Wave: Departure from normal is
deposition of sediment in areas, stabilize 4.5°C to 6.4°C
SC
the coastal shores, provide breeding ground  Severe Heat Wave: Departure from
for fishes. normal is >6.4°C
 Mangroves moderate monsoonal tidal floods  Based on Actual Maximum Temperature
and reduce inundation of coastal lowlands.
 Heat Wave: When actual maximum
 It prevents coastal soil erosion. temperature ≥ 45°C
 It protects coastal lands from tsunami,  Severe Heat Wave: When actual
hurricanes and floods. maximum temperature ≥47°C
GS

 Mangroves enhance natural recycling of  If above criteria met at least in 2 stations


nutrients and acts as a sink for heavy in a Meteorological sub-division for at least
metals. two consecutive days and it is declared on
Why in News? the second day.

 Researchers from Kerala Forest Research  When maximum temperature departure is


Institute and Ghent University, Belgium 4.5°C or more from normal, Heat Wave may
have recently found that mangrove plants be described for coastal stations provided
are heavily equipped to fight climate actual maximum temperature is 37°C or
change. more.

 Mangrove plants have a special phenomenon  Heat wave generally occurs over plains of
northwest India, Central, East & north
called foliar water uptake (FWU), which is
Peninsular India during March to June.
a mechanism that enables plants to acquire
It covers Punjab, Haryana, Delhi, Uttar
water from the atmosphere through their
Pradesh, Bihar, Jharkhand, West Bengal,
leaves.
Odisha, Madhya Pradesh, Rajasthan,
 As mangroves live in a saline sediment water Gujarat, parts of Maharashtra & Karnataka,
environment, the mechanism of FWU might Andhra Pradesh and Telengana. Sometimes
be of vital importance to acquiring fresh it occurs over Tamil Nadu & Kerala also.
water and growth. The amazing ability of
 Favorable conditions for Heat wave are:
mangrove plants to take up water from the
rain and atmospheric water makes them a  Transportation / Prevalence of hot dry
good candidate to answer climate change. air over a region (There should be a

23
PTS2023/PPP-01/092022/11

Telegram - @upscmaterialonline1
.
region of warm dry air and appropriate usage may change. The rest of the buildings
flow pattern for transporting hot air over that came up post-1947, including Shastri
the region). Bhavan and Krishi Bhavan, are likely to be
demolished.
 Absence of moisture in the upper
atmosphere (As the presence of moisture  Central Vista Project also includes a joint
restricts the temperature rise). central secretariat, revamp of the Rajpath,
a new Prime Minister’s residence, a new
 The sky should be practically cloudless
Prime Minister’s Office, and a new Vice-
(To allow maximum insulation over the
President’s enclave.
region).
 The new Parliament building will coming up
 Large amplitude anti-cyclonic flow over
adjacent to the existing Parliament complex,
the area.
and will be of almost the same size. Bimal
Why in News? Patel of HCP Designs, Ahmedabad, is the
architect in charge of the building, which
 Temperatures across Europe are soaring is triangular in shape, and incorporates
recently, at or near triple digits from Spain architectural styles from around India.
to the British Isles and spreading east.
Wildfires stoked by the heat are burning in  The new complex has 888 seats in the
many countries, and much of the continent Lok Sabha chamber and 384 seats in the

RE
is in the throes of a lengthy drought. Rajya Sabha chamber. Unlike the present
Parliament building, there is no Central
 Heat waves in Europe are increasing in Hall, and the Lok Sabha chamber will be
frequency and intensity at a faster rate able to house 1,272 members in case of a joint
than almost any other part of the planet, session. The building will have an open-sky
including the Western United States. area of 2,000 sq metre for a banyan tree.
 The existing building was designed by
38. Correct Option: (d)
O Edwin Lutyens and Herbert Baker who were
responsible for the planning and construction
Explanation: of New Delhi. It was inaugurated in 1927, by
 Statement 1 is incorrect: The project then Governor General of India Lord Irwin.
envisages: Constructing a triangular
SC
Why in News?
Parliament building next to the existing one,
constructing a common Central Secretariat  Prime Minister Narendra Modi recently
and revamping of the 3-km-long Rajpath unveiled the National Emblem cast on the
— from Rashtrapati Bhavan to India Gate. roof of the new Parliament building.
The Parliament House and North and South
 Cast at the top of the Central Foyer of the
Blocks will not be demolished, but their
new Parliament building, the 6.5-metre-
usage may change.
high National Emblem is made of bronze,
GS

 Statement 2 is incorrect: The Union and weighs 9,500 kg. A supporting structure
Ministry of Housing and Urban Affairs of steel weighing around 6,500 kg has been
proposed a Central Vista redevelopment constructed to support the Emblem.
project in 2019.
 The new building will have six granite statues
Supplementary notes: of important personalities, four galleries
each for the two Houses of Parliament, three
Central Vista Redevelopment Project ceremonial foyers, three India galleries, and
 The Central Vista is a 3 km stretch in one Constitution gallery.
the heart of New Delhi that runs from  Each wall in the building will have a
the Rashtrapati Bhavan to India Gate. dominant theme — dedicated, for instance,
It is flanked by large green spaces and to the contribution of tribal leaders, or to
significant structures such as Parliament, showcasing the contribution of women.
the Secretariat buildings and the National Displays including a mix of portraits,
Archives. illustrative arts, installations, sculptures,
 The Central government is redeveloping and decorative art will frame storylines.
the three-km-long Central Vista and
Parliament. A common Central secretariat 39. Correct Option: (b)
will be constructed for all ministries that
are currently spread over many buildings Explanation:
across Delhi.
 Statement 1 is incorrect: The ISS is
 The Parliament House and North and South not the first space station to be built and
Blocks will not be demolished, but their operated. Several smaller space stations

PTS2023/PPP-01/092022/11 24

Telegram - @upscmaterialonline1
.
have been used earlier, the most famous space station that operated in the 1980s,
of which have been the Russian Mir space and the American Skylab.
station that operated in the 1980s, and the
 ISS is not entirely in zero-gravity space. It
American Skylab.
does encounter a little bit of gravity. Also,
 Statement 3 is incorrect: In 2020, it loses a bit of energy as it moves around
SpaceX’s Crew Dragon capsule became the the earth. Left to itself, the ISS would
first privately-owned spacecraft to transport fall down. The Russians periodically send
people to the ISS. thrusters that attach themselves to the ISS
and impart the required momentum to keep
Supplementary notes:
it going.
International Space Station (ISS)  The ISS has been in operation since 1998
 Launched in 1998 and involving the U.S., and is expected to continue at least till
Russia, Canada, Japan, and the participating 2028. Russia, however, has indicated that it
countries of the European Space Agency— might pull out of the collaboration earlier,
the International Space Station is one of the possibly by 2024. Replacements for ISS are
most ambitious international collaborations already being planned. Several countries
ever attempted. The largest space station including India and China have a desire
ever constructed, the ISS continues to be to put their own space stations. China has

RE
assembled in orbit. already tested at least two prototypes.
 The International Space Station is the only Why in News?
operational space laboratory as of now,
orbiting the earth in a trajectory that is  Moscow’s space agency Roscosmos ‘ newly-
about 400 km above the land surface. It is appointed chief Yury Borisov recently
operated by more than 15 partner countries. announced that Russia will quit the
It can be seen from Earth without the use
O International Space Station after 2024.
of a telescope by night sky observers who  The announcement comes as tensions rage
know when and where to look. between the Kremlin and the West over
 The football-field-sized ISS travels at a Moscow’s military operation in Ukraine and
speed of about 28,000 km per hour. It several rounds of unprecedented sanctions
completes one journey around the earth against Russia.
SC
in about one and a half hours. In one day,  Russia and the United States have worked
therefore, it makes about 16 trips around side by side on the space station which has
the world. Astronauts working and living
been in orbit since 1998.
on the Station experience 16 sunrises and
sunsets each day.
 The ISS has been manned continuously 40. Correct Option: (b)
since the start of operations in 1998. At any Explanation:
GS

given time, there is a crew of six astronauts


on board. Right now, seven astronauts, four  Statement 2 is incorrect: It is not a
from the United States, two from Russia (it United Nations body. However, at the
prefers to call them cosmonauts) and one request of the IPBES Plenary and with
from Germany, are at ISS. According to the authorization of the UNEP Governing
NASA, 240 individuals from 19 countries Council in 2013, the United Nations
have so far been to the ISS. The facility Environment Programme (UNEP)
is used for carrying out a variety of zero- provides secretariat services to
gravity experiments, space exploratory IPBES.
studies, and technology development.
Supplementary notes:
 Typically, astronauts travel to the space
station via a Russian Soyuz capsule (first Intergovernmental Science-Policy
launched in 1967), which has long been the Platform on Biodiversity and Ecosystem
only spacecraft that ferries people to the Services (IPBES)
ISS, since NASA’s space shuttle programme  Intergovernmental Science-Policy
retired in 2011. However, in 2020, SpaceX’s Platform on Biodiversity and Ecosystem
Crew Dragon capsule became the first
Services (IPBES) is an independent
privately-owned spacecraft to transport
intergovernmental body established by
people to the ISS.
States to strengthen the science-policy
 The ISS is not the first space station to be interface for biodiversity and ecosystem
built and operated. Several smaller space services for the conservation and sustainable
stations have been used earlier, the most use of biodiversity, long-term human well-
famous of which have been the Russian Mir being and sustainable development.

25
PTS2023/PPP-01/092022/11

Telegram - @upscmaterialonline1
.
 IPBES recently released a report which Supplementary notes:
states that about 50,000 wild species globally
can meet the needs of billions of people. Draft Drugs, Medical Devices and
Cosmetics Bill, 2022
 Key highlights of the report are:
 The draft focuses on regulating medical
 With about 50,000 wild species used devices as a separate entity, makes
through different practices, including more provision for fines and imprisonment for
than 10,000 wild species harvested directly
injury and death related to clinical trials
for human food, rural people in developing
or investigations, and seeks to regulate
countries are most at risk from unsustainable
e-pharmacies.
use, with lack of complementary alternatives
often forcing them to further exploit wild  For the first time, regulations for conduct
species already at risk. of clinical trials for new drugs and medical
 One out of five sources their food from wild devices have been brought under the draft
plants, algae and fungi, while 2.4 billion New Drugs, Medical Devices and Cosmetics
depend on firewood for cooking and around Bill, 2022 which seeks to replace the existing
90 per cent of the 120 million population Drugs and Cosmetics Act of 1940.
pursuing fisheries rely on small-scale  The draft bill has a separate chapter for
fishing. AYUSH drugs which proposes to regulate

RE
 The use of wild species defines identities Sowa Rigpa and Homoeopathy for the first
and livelihoods and also holds cultural time. The existing act regulates Ayurveda,
significance. Certain species have cultural Unani and Siddha drugs and cosmetics.
importance as they offer multiple benefits
 The draft bill introduces various new
that define tangible and intangible features
definitions or provisions like bioequivalence
of people’s cultural heritage.
study, bioavailability study, clinical trial,
 Helping indigenous and local communities clinical investigation, controlling authority,
O
maintain their ability to use wild species manufacturer, medical device, new drugs,
sustainably and protecting their cultural over the-counter (OTC) drugs, adulterated
practices associated with them would ensure cosmetics, etc. for more clarity and smooth
their survival. functioning and implementation.
SC
 Indigenous people and local communities  It proposes the constitution of a separate
used local knowledge, practices and Drugs Technical Advisory Board (DTAB)
spirituality for the sustainable use of wild
and Medical Devices Technical Advisory
species. They respected nature and only
Board (MDTAB), comprising experts from
took what they needed. This ensured that
various associations to advise the central
healthy populations of wild species were
government in technical matters.
maintained.
 It stressed that the sustainable use of wild  The penalties for offences related to import
GS

species needed “constant negotiation and of drugs and cosmetics have been enhanced
adaptive management” along with a common appropriately.
understanding of “sustainable use”.  It mentions where any person permitted
Why in News? under sub-section (1) of section 72 fails to
provide the required medical management
 The ninth session of the Plenary of the or compensation under section 73, shall
Intergovernmental Science-Policy Platform be punishable with imprisonment which
on Biodiversity and Ecosystem Services may extend to one year or with fine which
(IPBES 9) was recently concluded in shall not be less than twice the amount of
Germany. compensation.
 In the interest of public health or extreme
41. Correct Option: (d) urgency of drugs, the central government
Explanation: is empowered to make provisions for
Central Licensing Authority to waive the
 Statement 1 is incorrect: Permission requirement of conducting clinical trials
has to be taken to operate an e-pharmacy. for manufacture or import of new drugs or
No person shall himself or by any other investigational new drugs in the country.
person on his behalf sell, or stock or
exhibit or offer for sale, or distribute, any  Where a participant during a clinical trial
drug by online mode (e-pharmacy) except suffers injury or death on account of his
under and in accordance with a licence or participation in such investigation, provision
permission issued in such manner as may has been made to provide compensation and
be prescribed. medical management to such participants.

PTS2023/PPP-01/092022/11 26

Telegram - @upscmaterialonline1
.
 Permission has to be taken to operate an beyond the affected country’s border. The
e-pharmacy. No person shall himself or by agency reserves the designation for only
any other person on his behalf sell, or stock those diseases that need a coordinated
or exhibit or offer for sale, or distribute, any international response to prevent it from
drug by online mode (e-pharmacy) except potentially escalating into a pandemic.
under and in accordance with a licence or
 In the past decade, WHO has declared public
permission issued in such manner as may
health emergencies for outbreaks including
be prescribed.
swine flu, polio and Ebola.
 Provisions have been incorporated to
designate or establish medical device Why in News?
testing centres by the central government  The World Health Organization (WHO)
for testing and evaluation of medical devices recently declared a ‘public health emergency’
for regulators and industry. over monkeypox.
Why in News?  So far this year, there have been more than
16,000 cases of monkeypox in more than 60
 The Union Health Ministry recently released
countries, and five deaths in Africa. The viral
a draft of a proposed The Drugs, Medical
disease — which spreads via close contact
Devices and Cosmetics Bill, 2022 to replace
and tends to cause flu-like symptoms and
the existing The Drugs and Cosmetics Act,

RE
pus-filled skin lesions — has been spreading
1940, and several sets of Rules by which the
chiefly in men who have sex with men in the
industry is currently run.
recent outbreak outside Africa, where it is
endemic.
42. Correct Option: (c)
Explanation: 43. Correct Option: (c)


concern” is not limited to epidemic-


O
Statement 1 is incorrect: The concept of
a “public health emergency of international

prone diseases, but extends to biological,


Explanation:
 Statement 1 and 2 are incorrect: Rubella,
JE and Rotavirus vaccine is provided in
select states and districts only.
chemical and nuclear hazards, including
SC
the chemical or nuclear contamination of Supplementary notes:
the environment, and contaminated food
and pharmaceuticals. The International Immunization in India
Health Regulations do not require actual  Expanded Programme on Immunization
international spread but only a high was launched in 1978. It was renamed as
potential for that spread. Universal Immunization Programme in
Supplementary notes: 1985 when its reach was expanded beyond
urban areas. In 1992, it became part of Child
GS

Public Health Emergency of International Survival and Safe Motherhood Programme


Concern (PHEIC) and in 1997 it was included in the ambit of
National Reproductive and Child Health
 Under the International Health Regulations
Programme. Since the launch of National
(IHR), a public health emergency is
Rural Health Mission in 2005, Universal
defined as “an extraordinary event which
Immunization Programme has always been
is determined, as provided in these
an integral part of it.
Regulations: to constitute a public health
risk to other States through the international  Universal Immunization Programme
spread of disease; and to potentially require (UIP) is one of the largest public health
a coordinated international response”. programmes targeting close of 2.67 crore
newborns and 2.9 crore pregnant women
 Declaring public health emergencies
annually.
of international concern (PHEIC) is a
cornerstone of the IHR. The responsibility  It is one of the most cost-effective public
of declaring an event as an emergency lies health interventions and largely responsible
with the Director-General of the WHO and for reduction of vaccine preventable under-5
requires the convening of a committee of mortality rate.
members.
 Under UIP, immunization is providing
 PHEIC is WHO’s highest alert level. free of cost against 12 vaccine preventable
diseases:
 The definition implies that the situation
is serious, sudden, unusual or unexpected,  Nationally against 9 diseases -
carries implications for public health Diphtheria, Pertussis, Tetanus, Polio,

27
PTS2023/PPP-01/092022/11

Telegram - @upscmaterialonline1
44. Correct Option: (a)

.
Measles, Rubella, severe form of
Childhood Tuberculosis, Hepatitis B
and Meningitis & Pneumonia caused by Explanation:
Hemophilus Influenza type B  Statement 2 is incorrect: Creation of the
 Sub-nationally against 3 diseases - Shanghai Cooperation Organization (SCO)
Rotavirus diarrhoea, Pneumococcal was proclaimed on June 15, 2001 in Shanghai
Pneumonia and Japanese Encephalitis; (China) by the Republic of Kazakhstan,
of which Rotavirus vaccine and the People’s Republic of China, the Kyrgyz
Pneumococcal Conjugate vaccine are in Republic, the Russian Federation, the
process of expansion while JE vaccine is Republic of Tajikistan and the Republic of
provided only in endemic districts. Uzbekistan. Prior to that, all of the above
countries, except for Uzbekistan, were
 A child is said to be fully immunized if child members of the Shanghai Five, a political
receives all due vaccine as per national association based on the Agreement on
immunization schedule within 1st year age Confidence-Building in the Military Field in
of child. the Border Area (Shanghai, 1996).
 The two major milestones of UIP have  Statement 3 is incorrect: The SCO
been the elimination of polio in 2014 and Interbank Consortium (SCO IBC) was
maternal and neonatal tetanus elimination established by the Council of Heads of

RE
in 2015. Government in 2005 to provide funding
 Mission Indradhanush (MI) was launched and bank services for investment projects
in December 2014 and aims at increasing sponsored by the governments of the
the full immunization coverage to children SCO member states. The members of
to 90%. the SCO IBC are the Development Bank
of Kazakhstan, the State Development
 Under this drive focus is given on pockets Bank of China, the Settlement & Savings
of low immunization coverage and hard Company of the Kyrgyz Republic “RSK
to reach areas where the proportion of
O Bank”, the Bank for Development and
unvaccinated and partially vaccinated Foreign Economic Affairs of the Russian
children is highest. Federation “Vnesheconombank”, the State
 A total of six phases of Mission Indradhanush Savings Bank of the Republic of Tajikistan
SC
have been completed covering 554 districts “Amonatbonk”, and the National Bank for
across the country. Foreign Economic Activity of the Republic
of Uzbekistan.
 It was also identified as one of the flagship
schemes under Gram Swaraj Abhiyan Supplementary notes:
(16,850 villages across 541 districts) and
Shanghai Cooperation Organization
Extended Gram Swaraj Abhiyan (48,929
villages across 117 aspirational districts). (SCO)
 The Shanghai Cooperation Organization
GS

Why in News?
(SCO) is a permanent intergovernmental
 According to data published by the WHO international organization, the creation
and UNICEF recently, the number of of which was announced on 15 June 2001
children in India who were unvaccinated in Shanghai (China) by the Republic of
or missed their first dose of diphtheria- Kazakhstan, the People’s Republic of
tetanus-pertussis (DTP) combined vaccine China, the Kyrgyz Republic, the Russian
doubled due to the pandemic, rising from Federation, the Republic of Tajikistan, and
1.4 million in 2019 to 2.7 million in 2021. the Republic of Uzbekistan.
 The increase in zero dose, or those who  It was preceded by the Shanghai Five
missed their first dose of diphtheria-tetanus- mechanism. All of the above countries,
pertussis, between 2019 and 2021 is the except for Uzbekistan, were members of the
“first time ever there has been a decline Shanghai Five, a political association based
in evaluated coverage in immunization for on the Agreement on Confidence-Building
India as a whole.” in the Military Field in the Border Area
(Shanghai, 1996).
 Though immunisation coverage continues
to decline the world over, India was able to  The SCO’s main goals are as follows:
arrest the trend between 2020 and 2021. strengthening mutual trust and
The number of children with zero dose neighborliness among the member states;
rose sharply to three million in 2020. But a promoting their effective cooperation in
further backsliding was prevented and the politics, trade, the economy, research,
number dropped to 2.7 million in 2021 due technology and culture, as well as in
to catch-up programmes such as the third education, energy, transport, tourism,
Intensified Mission Indradhanush (IMI). environmental protection, and other areas;

PTS2023/PPP-01/092022/11 28

Telegram - @upscmaterialonline1
.
making joint efforts to maintain and ensure  Cervical cancer is by far the most common
peace, security and stability in the region; HPV-related disease. A majority of this type
and moving towards the establishment of cancer – more than 95 per cent – is caused
of a democratic, fair and rational new because of HPV.
international political and economic order.
 HPV infections often clear up on their own
 The organization has two permanent bodies and most pre-cancerous lesions resolve
— the SCO Secretariat based in Beijing and spontaneously. However, there is a risk for
the Executive Committee of the Regional all women that HPV infection may become
Anti-Terrorist Structure (RATS) based in chronic and pre-cancerous lesions progress
Tashkent. to invasive cervical cancer, says the WHO.
 The Heads of State Council (HSC) is the  It takes 15 to 20 years for cervical cancer
supreme decision-making body in the to develop in women with regular immune
SCO. It meets once a year and adopts systems. It can take only five to 10 years in
decisions and guidelines on all important women with weakened immune systems,
matters of the organization. The SCO such as those with untreated HIV infection.
Heads of Government Council (HGC) meets
 Cervical cancer is a type of cancer that occurs
once a year to discuss the organization’s
in the cells of the cervix — the lower part of
multilateral cooperation strategy and
the uterus that connects to the vagina. It

RE
priority areas, to resolve current important
is the fourth most common cause of cancer
economic and other cooperation issues, and
among women in the world.
also to approve the organization’s annual
budget.  In India, cervical cancer is the second-most
common cancer, especially among women
 SCO was first expanded in 2017 when
between 15 and 44 years of age.
India and Pakistan were admitted as its
members.  Cervical cancer continues to be one of the
only vaccine-preventable cancers. There are
Why in News?

O
The Shanghai Cooperation Organisation is
all set for expansion with Iran joining the
two to four types of HPV that lead to almost
70% to 80% of all the cervical cancers and
the vaccines prevent these infections. Since
grouping as its ninth member while Belarus the virus is sexually transmitted, it has to
SC
has applied for membership recently. be given to teenage girls before they are
sexually active. Once an infection happens,
 The decision to admit Iran was made in last the vaccine cannot clear it out. Hence, the
year’s Dushanbe summit and Belarus has effectiveness of the vaccine keeps dropping
submitted its application. with age.
 Iran will become a full member after  At present, there are two HPV vaccines
completing the formalities while Shanghai available – GSKs Cervarix and Merck’s
Cooperation Organisation (SCO) member Gardasil. The first one protects against the
GS

states who follow the system of consensus two most common types HPV 16 and 18,
in deciding admission of new members will whereas the second one protects against
take a call on Belarus’s application. two additional types 6 and 11.
Why in News?
45. Correct Option: (a)
 The Drugs Controller General of India (DCGI)
Explanation: recently granted market authorisation to
the Pune-based Serum Institute of India
 Statement 3 is incorrect: ‘Cervavac’ has
(SII) to manufacture ‘Cervavac’, India’s
been developed by Serum Institute of India
first quadrivalent human papillomavirus
(SII).
vaccine (qHPV). It protects women against
Supplementary notes: cervical cancer.

Human Papillomavirus (HPV)


46. Correct Option: (d)
 According to the World Health Organization
(WHO), human papillomavirus (HPV) is Explanation:
the most common viral infection of the
 Statement 1 is incorrect: India along
reproductive tract. Most sexually active
with Japan is the only country to have won
women and men will be infected at some
at-least one gold medal in every event of
point in their lives, and some may be
Asian Games.
repeatedly infected. More than 90 per cent
of the infected populations eventually clear  Statement 2 is incorrect: The 19th edition
the infection. of the Asian games will be hosted in the

29
PTS2023/PPP-01/092022/11

Telegram - @upscmaterialonline1
.
Chinese city of Hangzhou. The Games were to propitiate Lord Ganesha, who, according
originally scheduled to take place from 10 to to Hindu belief, removes obstacles in one’s
25 September 2022, but due to the COVID- life.
19 pandemic, the event was postponed and
 This ritual was started to help elephants
a new date will be announced later.
paraded at 1982 Asian Games’ opening
Supplementary notes: ceremony regain health.

Asian Games
 The Asian Games is a multi-disciplinary
47. Correct Option: (a)
sports event held once in every four years. Explanation:
Athletes from all Asian countries are
welcomed to participate in this event.  Statement 2 is incorrect: Prices of coal
The Asian Games is recognised by the from all the sales channels of coal, including
International Olympic Committee and is import are taken into account for compiling
the second largest multi-sport event after the NCI.
the Olympics. Supplementary notes:
 Before the Asian Games, the Far Eastern
National Coal Index (NCI)
Championship Games were held.

RE
 Since the Ministry of Coal started
 After the Second World War, many Asian
commercial auction of coal mines on revenue
countries gained independence and Guru
share basis, the NCI was conceptualized in
Dutt Sondhi, a member of the Indian
order to arrive at the revenue share based
International Olympic Committee, proposed
on market prices of coal.
the idea of Asian Games as a sporting
event, where all Asian nations can be  The NCI is a price index which reflects the
represented. change of price level of coal on a particular

O
The first-ever Asian Games were held in month relative to the fixed base year.
New Delhi in 1951. Asian Games were  The base year for the NCI is FY 2017-18.
regulated by the Asian Games Federation
from 1951 to 1978. Since 1982, Olympic  Prices of coal from all the sales channels of
coal, including import are taken into account
SC
Council of Asia now regulates the Asian
Games. for compiling the NCI.

 The symbol for Asian Games is rising sun  The amount of revenue share per tonne of
with interlocking rings. coal produced from auctioned blocks would
be arrived at using the NCI by means of
 Nine nations haves hosted the Asian Games defined formula.
so far and 46 nations have participated in
the games. Israel has not taken part in  NCI was rolled out in June 2020.
GS

Asian Games since 1974. 44 sports have  NCI is composed of a set of five sub-indices:
been included in the Asian Games history. three for Non Coking Coal and two for
India is a founder member of Asian Games Coking Coal.
and also the host of the first Asian Games.
1982 Asian Games were also held in New  The three sub-indices for Non Coking Coal
Delhi. are combined to arrive at the Index for Non
Coking Coal and the two sub-indices for
 India is one of the seven countries to have Coking Coal are combined to arrive at the
participated in all the editions of the Asian Index for Coking Coal. Thus, indices are
Games. India along with Japan is the only separate for Non-coking and Coking Coal.
country to have won at-least one gold medal As per the grade of coal pertaining to a
in every event of Asian Games. India has mine, the appropriate sub-index is used to
always ranked in the top 10 in the medals arrive at the revenue share.
tally in Asian Games, except in the 1990
games. Why in News?
 The most recent games was held in Jakarta  For the last six months, the National Coal
and Palembang, Indonesia from 18 August Index (NCI) has jumped from about 165 to
to 2 September 2018. The next games are about 238 reflecting the sharp increase in
scheduled to be held in Hangzhou, China. international coal prices. The domestic coal
industry has responded to this situation
Why in News?
with an increase of over 30 per cent in coal
 Anayoottu is an annual ritual at Sree production from April to June this year. This
Vadakkunnathan Temple, Thrissur in which has helped reduce inflationary pressures in
more than 50 elephants are fed special food the economy.

PTS2023/PPP-01/092022/11 30

Telegram - @upscmaterialonline1
48. Correct Option: (c)

.
 Canada (21,597 in 2021 from 17,093 in
2020)
Explanation:
 United Kingdom (14,637 in 2021 from
 Statement 1 is correct: According to the 6,489 in 2020)
data released by the Union Ministry of
Home Affairs (MHA), the Indians who gave  Italy (5,986 in 2021 from 2,312 in 2020)
on their citizenships were highest in 2021,  As many as 362 Indians living in China also
if considered the data of last five years. acquired Chinese citizenship.
The number of Indians who gave up their
citizenship in 2021 stand at 1,63,370 as Why in News?
compared to the same in year 2020, which  Over 1.6 lakh Indians renounced their
stood at 85,256 in 2020. citizenship in 2021, which is highest in the
 Statement 2 is incorrect: As per Union past five years.
Ministry of Home Affairs the number of
Indians who gave up citizenship in the years
2017, 2018, 2019, 2020 and 2021 stood at
49. Correct Option: (b)
1,33,049, 1,34,561, 1,44,017, 85,248 and Explanation:
1,63,370 respectively.
 Statement 1 is correct: The cowpea (Vigna

RE
 The trend clearly indicates that the rise unguiculata) is an annual herbaceous
has not been consistent and a decline legume from the genus Vigna.
was observed in year 2020.
 Statement 2 is correct: it has tolerance
 Statement 3 is correct: In 2018, MHA for sandy soil and low rainfall, which makes
revised Form XXII under Citizenship Rules it an important crop in the semiarid regions
and included a column on “circumstances/ across Africa and Asia.
reasons due to which applicant intends to
Statement 3 is incorrect: Nigeria is the
Indian citizenship.”
Supplementary notes:
O
acquire foreign citizenship and renounce 
only country to approve open cultivation of
genetically modified (GM) Bt cowpea.
Supplementary notes:
Renunciation of Indian Citizenship:
SC
What is GM Technology?
 As per the citizenship rules 2009 of
Government of India, Indian nationals  GM is a technology that involves inserting
living abroad who intend to acquire a foreign DNA into the genome of an organism. To
citizenship are required to renounce produce a GM plant, new DNA is transferred
their Indian citizenship, because into plant cells. Usually, the cells are then
Government of India does not permit grown in tissue culture where they develop
dual nationality to Indian citizens. into plants. The seeds produced by these
plants will inherit the new DNA.
GS

 Renunciation/Surrender Certificate is
issued to applicants who surrender their About cowpea:
Indian passport after acquiring foreign  Cowpea, also known as Black Eyed Pea,
nationality. No police verification is required is a staple crop in the country and is an
for surrender certificate. important source of protein for over 200
 Getting back Indian citizenship back million people.
after renouncing it:
 Cowpea (Vigna unguiculata), an indigenous
 There is a provision to get back Indian legume to sub-Saharan Africa, is mainly
citizenship. He/she has to declare grown in the dry savanna areas as an
intention of renunciation in Form XXII intercrop with millets, sorghum, groundnut
to the Indian Mission/Post where OCI and maize.
registration was granted. After receipt
 Nigeria had become the first country to
of the declaration, the Indian Mission/
approve open cultivation of genetically
Post shall issue an acknowledgement in
modified (GM) Bt cowpea, in 2019.
Form XXII A.
 Now genetically modified (GM) cowpea has
 Top 5 countries where Indians
also cleared the first of the three regulatory
renounced their citizenship in 2021:
hurdles in the way of commercialization in
 United States (78,284 in 2021 from Ghana.
30,828 in 2020)
 Cowpea – black-eyed pea in some parts of
 Australia (23,533 in 2021 from 13,518 in the world – is a major source of protein in
2020) Ghana and the rest of the sub-continent. It

31
PTS2023/PPP-01/092022/11

Telegram - @upscmaterialonline1
.
is integral to Ghana’s food security. Ghana Supplementary notes:
is the fifth largest producer of cowpea in
Africa. What is Depreciation?

 Africa accounts for 84% of grain production;  Currency depreciation is a fall in the value
of a currency in a floating exchange rate
Nigeria being the largest cowpea producer in
system.
the world and accounts for over 2.5 million
tons of grain production from an estimated  Rupee depreciation means that the rupee
4.9 million ha. has become less valuable with respect to
the dollar.
Cultivation condition:
 It means that the rupee is now weaker than
 The optimum temperature for cowpea what it used to be earlier.
growth is 30 °C (86 °F), making it only
available as a summer crop for most of Impact of Depreciation of Rupee:
the world. It grows best in regions with an
 As money flows out of India, the rupee-dollar
annual rainfall between 400 and 700 mm
exchange rate gets impacted, depreciating the
(16 and 28 in). The ideal soils are sandy and rupee. Such depreciation puts considerable
it has better tolerance for infertile and acid pressure on the already high import prices
soil than most other crops of crude and raw materials, paving the path

RE
Why in News? for higher imported inflation and production
costs besides higher retail inflation.
 Genetically modified (GM) cowpea has
cleared the first of the three regulatory  The current account deficit is bound to
hurdles in the way of commercialization widen, depleting foreign exchange reserves
in Ghana. If it is commercialized, it will be and weakening the rupee.
the first GM crop grown in the country – a  With higher landed prices of crude oil
major milestone. and other crucial imports, the economy
O is definitely inching towards cost-push
inflation.
50. Correct Option: (d)
 Since a large proportion of India’s imports
Explanation: are dollar-denominated, these imports will
SC
 Statement 1 is correct: The government get costlier. A good example is the crude oil
(which is in charge of the fiscal policy) should import bill.
contain its borrowings”. Higher borrowings  Costlier imports, in turn, will widen the
(fiscal deficit) by the government eat up trade deficit as well as the current account
domestic savings and force the rest of the defict, which, in turn, will put pressure on
economic agents to borrow from abroad. the exchange rate.
This can further result in slide of rupee.
Impact of Depreciation of Indian Rupee:
GS

 Statement 2 is correct: The RBI, which Depreciation in rupee is a double-edged


is in charge of monetary policy, should sword for the Reserve Bank of India.
focus on containing inflation, as it is legally
mandated to do. It is important to note  Positive: While a weaker currency may
that containing inflation (that is, by raising support exports amid a nascent economic
interest rates) may amount to sacrificing recovery from the pandemic.
economic growth.  Negative: It poses risk of imported inflation,
 Statement 3 is correct: The Masala Bond and may make it difficult for the central
is directly pegged to the Indian currency. bank to maintain interest rates at a record
If Indian borrowers issue more rupee low for longer.
denominated Masala Bonds, this would Why in News?
increase liquidity in the market or increase
in the rupee stock against few currencies  The Indian rupee fell to a new low of 80
in the market and this would help in against the U.S. Dollar. It means that the
supporting the rupee. rupee is now weaker than what it used to
be earlier.
 Statement 4 is correct: External
Commercial Borrowing (ECB) is a type of
loan in foreign currencies, made by non- 51. Correct Option: (c)
resident lenders. Thus, easing conditions
Explanation:
of ECB’s helps in receiving more loans in
foreign currencies would increase the inflow  Statement 1 is incorrect: The largest
of forex, leading to rupee appreciation. lithium reserves are found in Latin

PTS2023/PPP-01/092022/11 32

Telegram - @upscmaterialonline1
.
America and Australia. Of the top ten Supplementary notes:
countries with the largest concentration of
lithium reserves, Chile tops the list, with India’s Antarctica Bill 2022
9.2 million metric tons.  Applicability: It will apply to any person,
 Statement 2 is incorrect: Korea and foreigners, corporations, firms, vessels or
Japan are the leading importers of Chinese aircraft that is a part of an Indian expedition
lithium. to Antarctica.

Supplementary notes:  Central Committee: 10 members (to


be nominated from various ministries) +
Lithium Imports two experts (on the Antarctic) + chairman
(Secretary of the Ministry of Earth Science)
 China and Hong Kong nations together
supply 96% of India’s lithium-ion cell and  It will give permits, ensure compliance, and
battery imports and almost 70% of non- review the information provided by parties
rechargeable lithium products. India to the treaty.
meets most of its lithium cell requirements
 Private tours and expeditions to Antarctica
through imports.
would be prohibited without a permit
 Lithium production in China grew from or written authorization by a member

RE
10.8K tonnes of lithium content in 2019 to country.
14K tonnes in 2020.
 Permit can be granted only after the
 MSP is an ambitious new alliance formed environmental impact assessment and waste
by the US to secure supply chains of critical management plan have been prepared.
minerals.
 Prohibited activities: The Bill prohibits
 The goal of the alliance is to ensure that certain activities in Antarctica including
critical minerals are produced, processed,
O
and recycled in a manner that supports
the ability of countries to realize the full
economic development benefit of their



nuclear explosion
radioactive wastes
or disposal

introduction of non-sterile soil


of

geological endowments.
SC
 discharge of garbage, plastic, or other
 The focus would be on the supply chains of substance into the sea which is harmful
minerals such as Cobalt, Nickel, Lithium, to the marine environment.
and also the 17 “rare earth” minerals.
 Offences and penalties (extend the
 However, India is not a part of this jurisdiction of Indian courts to
partnership. Antarctica):
Why in News?  Conducting a nuclear explosion in
GS

 Energy security and shift to a green future Antarctica will be punishable with an
have put the focus on rare earth elements. imprisonment of 20 years which may
However, India is import dependent on extend to life imprisonment and a fine of
critical minerals. Thus, it needs “mining at least Rs 50 crore.
reforms, US partnership”.  Drilling for mineral resources or
 In order to break the dominance and introducing non-native animals or
reduce dependence on China in mining and plants in Antarctica without a permit
processing rare earth minerals, the United will be punishable with imprisonment of
States recently announced the formation up to seven years and a fine between Rs
of a global alliance called the Mineral 10 lakh and Rs 50 lakh.
Security Partnership (MSP).  The central government may notify one or
 Apart from the US, the other countries to more Sessions Courts to be the Designated
have joined this partnership are: Australia, Court under the Bill and specify its territorial
Canada, Finland, France, Germany, Japan, jurisdiction to try offenses punishable under
the Republic of Korea, Sweden, the UK, and the Bill.
the European Commission.  Antarctic Fund: For the welfare of
Antarctic research work and the protection
52. Correct Option: (c) of the Antarctic environment.
 Establishes a ‘Committee on Antarctic
Explanation:
Governance and Environmental
 All statements are correct Protection.’

33
PTS2023/PPP-01/092022/11

Telegram - @upscmaterialonline1
.
Why in News? voyage in 1982 (Under Operation
 The Antarctic treaty (signed in 1959 and Gangotri)
implemented in 1961) made it mandatory
for the 54 signatory countries to specific
53. Correct Option: (c)
laws governing territories on which their
stations are located. India signed the treaty Explanation:
in 1983 and therefore a law was needed to
preserve the pristine Antarctic environment  Option (c) is correct
and ocean around it.
Supplementary notes:
 Initially, India was left out of 1959’s
Antarctica Treaty, but it made a maiden MTP amendment Act, 2021

The MTP Act 1971 and The MTP Act Amendments 2021
MTP Act 1971 The MTP Amendment Act 2021
Indications (Contraceptive Only applies to married women Unmarried women are also vocered
failure)
Gestational Age Limit 20 weeks for all indications 24 weeks for rape survivors Beyond 24 weeks

RE
for substantial fetal abnormalities
Medical practitioner One RMP till 12 weeeks One RMP till 20 weeks
opinions required before Two RMPs till 20 weeks Two RMPs 20-24 weeks
termination
Medical Board approval after 24 weeks
Breach of the women’s Fine up to Rs. 1000 Fine and/or Imprisonment of 1 year
confidnetiality
O
 Increased Gestational Limit: The  The abortion procedure has to be done
gestational limit for termination of a with five days of the board receiving the
pregnancy has been increased from 20 to request for the same.
SC
24 weeks for certain categories of women.
Why in News?
The seven specific categories are:
 An unmarried woman suffers an unwanted
 Survivors of sexual assault or rape pregnancy, why should she be excluded from
or incest. termination up to 24 weeks if a married
 Minors. woman is allowed it… The danger to life
is as much in the case of an unmarried
 Change of marital status during the woman as in the case of a married woman,”
ongoing pregnancy (widowhood and Justice Chandrachud, asked the Centre,
GS

divorce). represented by Additional Solicitor General


Aishwarya Bhati.
 Women with physical disabilities.
 The court noted that the Rules permit
 Mentally ill women. termination of pregnancies of up to 24
 Foetal malformation that has a weeks in seven specific categories, including
substantial risk of being incompatible survivors of rape or sexual assault, minors,
with life or if the child is born, he/ she in case of physical disabilities and foetal
may suffer from serious physical or malformation.
mental abnormalities.
 Women with pregnancy in humanitarian 54. Correct option: (c)
settings or disaster or emergency Explanation:
situations.
 Both statements are incorrect
 State-level Medical Board: A state-level
medical board will be set up to decide if Supplementary notes:
pregnancy may be terminated after 24 Open Network Digital Commerce
weeks in cases of foetal malformation. (ONDC)
 The medical boards are to either accept or  It is an initiative aimed at promoting open
reject the proposal for medical termination networks for all aspects of exchange of
of pregnancy within three days of receiving goods and services over digital or electronic
the request. networks.

PTS2023/PPP-01/092022/11 34

Telegram - @upscmaterialonline1
.
 ONDC is to be based on open-sourced  It should be noted here that the Parliament,
methodology, using open specifications and till now, has not made any special law to
open network protocols independent of any exhaustively codify all the privileges. They
specific platform. are based on five sources, namely,
 It is being developed as a counter to the  Constitutional provisions
current duopoly in the Indian e-commerce
 Various laws made by Parliament
market which is largely dictated by Amazon
and Walmart-owned Flipkart.  Rules of both the Houses

How does ONDC work?  Parliamentary conventions

 The ONDC platform lies in the middle of  Judicial interpretations.


the interfaces hosting the buyers and the Who has the entitlement to such
sellers. So far, the buyer side interface is
privileges?
being hosted by Paytm, whereas the seller
side interface is being hosted by other  Other than members of Parliament
players like GoFrugal, etc. and members of the state assemblies,
the Constitution has also extended
 When a buyer searches for an item on the parliamentary privileges to those persons
Paytm app, from where ONDC has gone live, who are entitled to speak and take part in

RE
the app will connect to the ONDC platform, the proceedings of a House of Parliament or
which will connect it to seller side interfaces any of its committees. These include the
that will list all the companies from where attorney general of India and Union
you can buy the particular item. ministers.
 On ONDC, there will be several other  It must be clarified here that
backend partners such as logistics service the parliamentary privileges do not
providers, enterprise resource planners,
O extend to the president who is also an
e-commerce store hosting service providers, integral part of the Parliament. Article
etc. 361 of the Constitution provides for
Why in News? privileges for the President.

US firm Microsoft has become the first big Parliamentary Privileges:


SC

tech company to join the Open Network for  Freedom of Speech: The members of
Digital Commerce (ONDC), a government- Parliament/state assembly enjoy the
backed project which is aimed at enabling freedom of speech and expression. No
small merchants and mom-and-pop stores member can be taken to task anywhere
in parts of the country to access processes outside the four walls of the House (e.g.,
and technologies that are typically deployed court of law) or cannot be discriminated
by large e-commerce platforms such as against for expressing his/her views in the
Amazon and Flipkart.
GS

House and its Committees.


 The software giant intends to introduce social  Freedom from Arrest: No member shall
e-commerce — group buying experience — be arrested in a civil case 40 days before and
in the Indian market, which would include a after the adjournment of the House and also
shopping app for Indian consumers along when the House is in session. It also means
with their social circle, harnessing the that no member can be arrested within the
ONDC network to discover the best pricing precincts of the Parliament without the
among retailers and sellers. permission of the House to which he/she
belongs. This privilege is also incorporated
under Section 135A of the Civil Procedure
55. Correct Option: (c) Code, 1908.
Explanation:  Exemption from Attendance as
 All options are correct Witnesses: The members of Parliament/
assemblies also enjoy freedom from
Supplementary notes: attendance as witnesses.
Parliamentary Privileges  Right to Publish Debates and
Proceedings: Parliament/Assembly can
 Parliamentary privileges are special rights,
prohibit the press to publish its proceedings
immunities, and exemptions enjoyed
when needed.
by the two Houses of Parliament,
their committees, and their members so  Right to exclude strangers: The object
that they can “effectively discharge their of including this right was to exclude any
functions”. chances of daunting or threatening any of

35
PTS2023/PPP-01/092022/11

Telegram - @upscmaterialonline1
.
the members. The strangers may attempt to  Compliance Intensive: The Bill was
interrupt the sessions. also seen as being too “compliance
intensive” by startups of the country.
 Right to Punish Members and Outsiders:
In India, the Parliament/Assembly has been  The revamped bill will be much easier to
given punitive powers to punish those who comply with, especially for startups.
are judged guilty of contempt of the House.  Issues with Data Localizations: The tech
companies questioned a proposed provision
in the Bill called Data Localizations.
56. Correct Option: (a)
 Under data localisation, it would have
Explanation: been mandatory for companies to
 Statement 1 is incorrect: Data localization store a copy of certain sensitive personal
rights have not been transferred to the data within India, and the export of
Central government. undefined “critical” personal data from
the country would be prohibited.
Supplementary notes:
 The activists had criticised that it would
Personal Data Protection Bill 2019 allow the central government and its
agencies blanket exemptions from adhering
 The Personal Data Protection Bill, to any and all provisions of the Bill.

RE
2019 was introduced in Lok sabha by the
Minister of Electronics and Information Why in News?
Technology, on December 11, 2019.  The government of India has withdrawn
 Commonly referred to as the “Privacy the Personal Data Protection Bill from
Bill”, it intended to protect individual Parliament as it considers a “comprehensive
legal framework” to regulate the online space
rights by regulating the collection,
to boost innovation in the country through a
movement, and processing of data that
O
is personal, or which can identify the
new bill.
individual.
 Challenges: 57. Correct Option: (b)
Explanation:
SC
 Many contend that the physical
location of the data is not relevant in  Statement 1 is incorrect: Markets
the cyber world as the encryption keys regulator SEBI has proposed the concept of
may still be out of reach of national blue bonds as a mode of sustainable finance
agencies. as such securities can be utilised for various
blue economy-related activities, including
 National security or reasonable purposes
oceanic resource mining and sustainable
are open-ended and subjective fishing.
terms, which may lead to intrusion of the
GS

state into the private lives of citizens. Supplementary notes:


 Technology giants like Facebook Blue Bonds
and Google are against it and have
 Blue bonds are pioneering financial
criticised the protectionist policy of data instruments that are designed to support
localisation as they are afraid it would sustainable marine and fisheries projects.
have a domino effect in other countries as
well.  They are a subset of the green bonds.

Why has the Bill been withdrawn?  The World Bank defines blue bonds “as
a debt instrument issued by governments,
 Too Many Amendments: development banks or others to raise capital
from impact investors to finance marine
 The Joint Committee of Parliament
and ocean-based projects that have positive
analysed the Personal Data Protection
environmental, economic and climate
Bill, 2019 in detail. benefits.”
 81 amendments were proposed and 12  In 2018, the Republic of Seychelles launched
recommendations were made towards the world’s first sovereign blue bond.
a comprehensive legal framework on
the digital ecosystem.  Markets regulator Sebi has proposed
the concept of blue bonds as a mode of
 Considering the report of the JCP, a sustainable finance as such securities can
comprehensive legal framework is be utilised for various blue economy-related
being worked upon. Hence, it is activities, including oceanic resource mining
proposed to withdraw. and sustainable fishing.

PTS2023/PPP-01/092022/11 36

Telegram - @upscmaterialonline1
.
 Also, the watchdog has suggested Why in News?
strengthening the framework for green
 The RBI’s value of financial inclusion index
bonds by amplifying the definition of green
(FI-Index), capturing the extent of financial
debt securities and enhancing disclosures,
inclusion across the country for March 2022
according to a consultation paper.
stands at 56.4 vis-a-vis 53.9 in March 2021,
 The proposals are aimed at aligning with with growth witnessed across all the sub-
the updated Green Bond Principles (GBP) indices, the central bank noted.
published by the International Capital
Market Association (ICMA).
59. Correct Option: (b)
 Since the framework of green debt securities
was laid down by the Securities and Explanation:
Exchange Board of India (SEBI), there have  Pair 1 is incorrectly matched: Dandeli
been multiple events in the sustainable Elephant Reserve is in Karnataka.
finance space around the world.
 Pair 2 is incorrectly matched: Singphan
Why in News? Elephant Reserve is in Nagaland.
 India has tremendous scope for deployment Supplementary notes:
of blue bonds in various aspects of the blue

RE
economy” like oceanic resource mining, Elephant Reserves
sustainable fishing, and national offshore
 The Indian elephant Elephas maximus
wind energy policy and in the area of blue
occurs in the central and southern Western
flag beach eco-tourism model that provides
Ghats, North-east India, eastern India
the tourists clean and hygienic bathing
and northern India and in some parts of
water facilities.
southern peninsular India.
It is included in Schedule I of the Indian
58. Correct Option: (b)
Explanation:
O 
Wildlife (Protection) Act, 1972; in Appendix
I of the Convention on International Trade
in Endangered Species of Flora and Fauna
 Option (b) is correct (CITES) and listed as ‘Endangered’ in IUCN
SC
Red List.
Supplementary notes:
 It occurs in 16 of the 28 states in the country
Financial Inclusion Index 2022
and is showing an increasing trend across
 The Reserve Bank of India had constructed its distributional range.
a composite Financial Inclusion Index (FI-
 India, which is home to over 60% of the wild
Index) to capture the extent of financial
Asian elephant population, has a recorded
inclusion across the country, in consultation
elephant population of 29,964 (according to
with the concerned stakeholders including
GS

the last elephant census of 2017). According


the Government.
to the 2017 census, Karnataka had the
 The index captures information on various highest number of elephants (6,049),
aspects of financial inclusion in a single followed by Assam (5,719) and Kerala
value ranging between 0 and 100, where 0 (3,054).
represents complete financial exclusion and
 Project Elephant was launched by the
100 indicates full financial inclusion.
Government of India in the year 1992 as a
 The FI-Index comprises of three broad Centrally Sponsored Scheme with following
parameters (weights indicated in brackets) objectives:
viz., Access (35%), Usage (45%), and
 To protect elephants, their habitat &
Quality (20%) with each of these consisting
corridors
of various dimensions, which are computed
based on a number of indicators.  To address issues of man-animal
conflict
 The Index is responsive to ease of access,
availability and usage of services, and  Welfare of captive elephants
quality of services, comprising in all 97
 The project is being mainly implemented
indicators.
in 16 States / UTs , viz. Andhra Pradesh,
 A unique feature of the Index is the Quality Arunachal Pradesh, Assam, Chhattisgarh,
parameter which captures the quality Jharkhand, Karnataka, Kerala,
aspect of financial inclusion as reflected by Maharashtra, Meghalaya, Nagaland,
financial literacy, consumer protection, and Orissa, Tamil Nadu, Tripura, Uttarakhand,
inequalities and deficiencies in services. Uttar Pradesh, West Bengal.

37
PTS2023/PPP-01/092022/11

Telegram - @upscmaterialonline1
.
 The Ministry of Environment, Forest and  Delhi and Kolkata were ranked first and
Climate Change provides the financial second in the list of top 10 most polluted
and technical support to major elephant cities when PM2.5 levels were compared,
range states in the country through Project with Delhi and Kolkata reporting an
Elephant. average annual exposure of (relative to
Why in News? population) of 110 μg/m3 and 84 μg/m3
respectively. μg/m3 refers to microgram
 Union Minister for Environment, Forest per cubic metre.
and Climate Change recently announced
the notification of one more elephant  However no Indian city appeared in the
reserve (ER) in the country in Tamil Nadu list of top 10 – or even top 20 - polluted
during a programme in the Periyar Wildlife cities when N0 2 levels were compared.
Sanctuary in Kerala. This list saw Shanghai at the top with
an average annual exposure of 41 μg/m3.
 The new reserve will be spread over an area of
Average NO 2 levels for Delhi, Kolkata
1,197 square kilometers in Agasthyamalai.
and Mumbai, according to the report,
 The Agasthyamalai elephant reserve will ranged from 20-30 μg/m3.
add to the existing 31 such protected areas
in the country. Among these, three elephant  This paradoxical situation in India
reserves – Dandeli in Karnataka, Singphan was likely due to the relatively lower

RE
in Nagaland and Lemru in Chhattisgarh adoption of high-efficiency engine
– were notified in the last three years. vehicles. Complete combustion of fuel
Elephant reserves were currently spread results in higher NOx (nitrogen oxides)
over 76,508 sq m across 14 States in the where incomplete combustion sees other
country. kinds of emissions.
 NO 2 comes mainly from the burning
60. Correct Option: (a) of fuels in older vehicles, power plants,
O industrial facilities and residential
Explanation: cooking and heating. As city residents
 Statement 2 is incorrect: Delhi and tend to live closer to busy roads with
Kolkata were ranked first and second in dense traffic, they are often exposed to
SC
the list of top 10 most polluted cities when higher NO 2 pollution than residents of
PM2.5 levels were compared. However no rural areas.
Indian city appeared in the list of top 10 –
 In 2019, 86% of the more than 7,000
or even top 20 - polluted cities when N0 2
cities analysed in the report exceeded
levels were compared.
the WHO’s 10 μg/m3 guideline for NO
 Statement 3 is incorrect: NO 2 also has 2, impacting about 2.6 billion people.
a shorter lifetime compared with PM2.5 “While PM2.5 pollution tends to get more
and other air pollutants. As a result, NO 2 attention on known hotspots around the
GS

levels show very high variability in space world, less data has been available for
and time. NO 2 at this global scale.”
Supplementary notes:  Due to their highly reactive nature,
Air Quality and Health in Cities nitrogen oxides also contributed to the
formation of other pollutants, including
 The report, “Air Quality and Health in ozone and particulate matter. NO 2
Cities”, released by US-based Health Effects also has a shorter lifetime compared
Institute, analyses pollution and global with PM2.5 and other air pollutants.
health effects for more than 7,000 cities As a result, NO 2 levels show very high
around the world, focusing on two of the variability in space and time — levels
most harmful pollutants - fine particulate
can vary significantly even across a few
matter (PM2.5) and nitrogen dioxide (NO
kilometres within the city. In comparison,
2).
PM2.5 levels tend to show less spatial
 Key findings of the report are: variation at the fine scale.
 Global patterns for exposures to the Why in News?
two key air pollutants were “strikingly
different.” While exposures to PM2.5  A global analysis of air quality recently
pollution tend to be higher in cities found that Indian cities, while recording
located in low- and middle-income particulate matter emissions (PM2.5) that
countries, exposure to NO 2 is high are among the highest in the world, do
across cities in high-income as well as relatively better on nitrogen dioxide (NO 2)
low- and middle-income countries. emissions.

PTS2023/PPP-01/092022/11 38

Telegram - @upscmaterialonline1
61. Correct Option: (c)

.
 Born in 1939, Hazarika was the youngest
of five children. She started learning dance
Explanation: from a very early age, under Sattriya
 Option (c) is correct stalwarts such as Raseswar Saikia Borbayan
and Ghanakanta Bora. Sattriya is a dance
Supplementary notes: form that has its roots in the Vaishnavite
movement which was propagated by scholar
Sattriya Dance Sankaradeva in the 15th century in Assam.
 Sattriya dances are devotional in nature
as they were intended for propagation of
neo-Vaishnavism. Its highlights are intense
62. Correct Option: (a)
emotional fervour, and in its solo avatar now Explanation:
dramatic abhinaya is prominent in contrast
 Statement 3 is incorrect: EEZ does not
to nritta, pure dance.
give a coastal state the right to prohibit or
 It was introduced in Assam by the great limit freedom of navigation or overflight,
Vaishnava saint and reformer of Assam, subject to very limited exceptions.
Mahapurusha Srimanta Sankaradeva in
the 15th century A.D. Supplementary notes:

RE
 He propagated the “ek sharan naama United Nations Convention on laws of
dharma” (chanting the name of one God Seas (UNCLOS)
devotedly).  The United Nations Convention on the
 History and evolution Law of the Sea (UNCLOS), 1982 is an
international agreement that establishes
 Unlike other classical dances, the Sattriya the legal framework for marine and
dance has been left untouched in this regard
O maritime activities.
and has been the same since its birth.
 It is also known as Law of the Sea. It
divides marine areas into five main zones
namely- Internal Waters, Territorial Sea,
Contiguous Zone, Exclusive Economic Zone
SC
(EEZ) and the High Seas.
 It is the only international convention which
stipulates a framework for state jurisdiction
in maritime spaces. It provides a different
legal status to different maritime zones.
 It provides the backbone for offshore
governance by coastal states and those
GS

navigating the oceans.


 It not only zones coastal states’ offshore
areas but also provides specific guidance for
states’ rights and responsibilities in the five
concentric zones.
 The five maritime zones are:
 Baseline: It is the low-water line along
the coast as officially recognized by the
 Under the patronage of Sankardeva, the coastal state.
social and religious group known as the
‘Sattras’ (Vaishnava mathsor monasteries)  Internal Waters: Internal waters are
formulated this dance to celebrate their waters on the landward side of the
beliefs which were embedded in Hinduism baseline from which the breadth of
and its various teachings. the territorial sea is measured. Each
coastal state has full sovereignty over its
 It had its influences from folk dance forms internal waters as like its land territory.
like Ojapali, Devadasi, Bihu and Bodos etc. Examples of internal waters include
Why in News? bays, ports, inlets, rivers and even lakes
that are connected to the sea. There is
 Noted Sattriya and Odissi dancer Gorima no right of innocent passage through
Hazarika passed away at her residence in internal waters. The innocent passage
the Assam capital. She was 83, and was said refers to the passing through the waters
to be suffering from age-related ailments. which are not prejudicial to peace and

39
PTS2023/PPP-01/092022/11

Telegram - @upscmaterialonline1
.
security. However, the nations have the a new, legally-binding global treaty to
right to suspend the same. govern the use of the high seas. This
could be the last round of negotiations for
 Territorial Sea: The territorial sea
extends seaward up to 12 nautical miles the conservation and sustainable use of
(nm) from its baselines. A nautical marine biological diversity. The treaty will
mile is based on the circumference of help conserve biodiversity in areas beyond
the earth and is equal to one minute of national jurisdiction (BBNJ) that lie outside
latitude. It is slightly more than a land countries’ 322-kilometre exclusive economic
measured mile (1 nautical mile = 1.1508 zones.
land miles or 1.85 km). The coastal  This is the fifth round of negotiations on
states have sovereignty and jurisdiction the international instrument under the
over the territorial sea. These rights
UN Convention on the Law of the Sea
extend not only on the surface but also to
(UNCLOS).
the seabed, subsoil, and even airspace.
But the coastal states’ rights are limited
by the innocent passage through the 63. Correct Option: (c)
territorial sea.
Explanation:
 Contiguous Zone: The contiguous zone
All statements are correct

RE
extends seaward up to 24 nm from its 
baselines. It is an intermediary zone
Supplementary notes:
between the territorial sea and the high
seas. The coastal state has the right to Combined Maritime Forces (CMF)
both prevent and punish infringement
of fiscal, immigration, sanitary, and  It is a multinational maritime partnership,
customs laws within its territory and which exists to uphold the Rules-Based
territorial sea. Unlike the territorial International Order (RBIO) by countering
sea, the contiguous zone only gives
O illicit non-state actors on the high seas
jurisdiction to a state on the ocean’s and promoting security, stability, and
surface and floor. It does not provide air prosperity across approximately 3.2 million
and space rights. square miles of international waters,
SC
which encompass some of the world’s most
 Exclusive Economic Zone (EEZ):
important shipping lanes.
Each coastal State may claim an EEZ
beyond and adjacent to its territorial sea  Counter-narcotics, counter-smuggling,
that extends seaward up to 200 nm from suppressing piracy, encouraging regional
its baselines. Within its EEZ, a coastal cooperation, and engaging with regional
state has sovereign rights for the purpose and other partners to strengthen relevant
of exploring, exploiting, conserving and capabilities in order to improve overall
managing natural resources, whether security and stability, and promoting a safe
GS

living or nonliving, of the seabed and maritime environment free from illicit non-
subsoil. Rights to carry out activities like state actors.
the production of energy from the water,
currents and wind. Unlike the territorial  When requested, CMF assets at sea will also
sea and the contiguous zone, the EEZ only respond to environmental and humanitarian
allows for the above-mentioned resource incidents.
rights. It does not give a coastal state
 It consists of 34 member nations, including
the right to prohibit or limit freedom of
countries such as Australia, Bahrain, Brazil,
navigation or over flight, subject to very
France, Japan, Pakistan, United States,
limited exceptions.
Yemen, and others.
 High Seas: The ocean surface and the
water column beyond the EEZ are referred  Task Forces: It comprises of three task
to as the high seas. It is considered as “the forces: CTF 150 (maritime security and
common heritage of all mankind” and is counter-terrorism), CTF 151 (counter
beyond any national jurisdiction. States piracy) and CTF 152 (Arabian Gulf security
can conduct activities in these areas as and cooperation).
long as they are for peaceful purposes,
Why in News?
such as transit, marine science, and
undersea exploration.  At the India-US 2+2 in April 2022, India had
announced that it would join the CMF as
Why in News? an Associate Partner and will strengthen
 United Nations member states have cooperation in regional security in the
recently gathered in New York to create Western Indian Ocean.

PTS2023/PPP-01/092022/11 40

Telegram - @upscmaterialonline1
64. Correct Option: (c)

.
 India has a positive Balance of Trade with
the Maldives.
Explanation:
 Development Assistance Programme:
 Statement 1 is incorrect: United States India has helped the Maldives in many
has the largest Indian diaspora in the diverse areas to bolster the development of
world. the Maldives e.g.
Supplementary notes:  Indira Gandhi Memorial Hospital, Maldives
Institute of Technical Education (now called
India-Maldives the Maldives Polytechnic),
 India and Maldives share ethnic, linguistic,  There are 25,000 Indian nationals living in
cultural, religious and commercial links. the Maldives (the second largest expatriate
 India was among the first to recognize the community).
Maldives after its independence in 1965  The proximity of location and improvements
and later established its mission at Male in in air connectivity in recent years has led to
1972. a very substantial increase in the number
 They officially and amicably decided their of Indians visiting the Maldives for tourism
maritime boundary in 1976. and business.

RE
 Both nations are founding members of  India is a preferred destination for
the South Asian Association for Regional Maldivians for education, medical
Cooperation (SAARC), the South Asian treatment, recreation, and business.
Economic Union and signatories to the
Why in News?
South Asia Free Trade Agreement.
 Recently, India and Maldives vowed to
 They have consistently supported each
strengthen their defence and security ties
other in multilateral areas such as the UN,
O with a major focus on maritime security.
the Commonwealth, the NAM, and the
This was concluded during the Maldives
SAARC.
President Ibrahim Solih’s visit to India.
 The Memorandums of Understanding
(Mou) signed between both the countries
covers areas such as hydrography, health, 65. Correct Option: (d)
SC
passenger and cargo services by sea, Explanation:
capacity building in customs and civil
service training.  All statements are correct
 Strategic Importance: The Maldives holds Supplementary notes:
strategic importance for India under the
government’s ‘Neighbourhood First’ policy
Swachh Sagar, Surakshit Sagar
due to its location in the Indian Ocean.  It is the first-of-its-kind and longest running
GS

coastal clean-up campaign in the world with


 In the Indian Ocean, the Maldives
the highest number of people participating
archipelago comprising 1,200 coral islands
in it.
lies next to key shipping lanes which ensure
uninterrupted energy supplies to countries  The Campaign will culminate on
like China, Japan, and India. “International Coastal Clean-up Day” (17th
September 2022).
 Both nations are working together to
counter China’s presence in the Indian  The day aims to increase public awareness
Ocean Region (IOR). about the accumulation and negative
impacts of litter in oceans, on coastlines and
 Trade and Economy: India and Maldives
on beaches.
signed a trade agreement in 1981, which
provides for the export of essential  Globally, “International Coastal Clean Up
commodities. Day” is celebrated on the third Saturday of
September, every year.
 Under the bilateral agreement, India
provides essential food items like rice,  Through this campaign, a mass behavioural
wheat flour, sugar, dal, onion, potato and change among the masses is intended by
eggs and construction material such as raising awareness about how plastic usage
sand and stone aggregates to the Maldives is destroying the marine life.
on favourable terms.
 A mobile app “Eco Mitram” has been
 India and Maldives signed the $800 million launched to spread awareness about the
Line of Credit Agreement in March 2019, for campaign and also for the common people
assisting the Maldives to achieve sustainable for voluntary registration for the beach
social and economic development. cleaning activity.

41
PTS2023/PPP-01/092022/11

Telegram - @upscmaterialonline1
.
 Objectives:  At 100°C, the dissociation pressure of
diammonium phosphate is approximately 5
 Running a cleanliness campaign along
mmHg.
India’s entire coastline.
 According to the diammonium phosphate
 To reduce marine litter.
MSDS from CF Industries, Inc.,
 Minimising the use of plastics. decomposition starts as low as 70 °C:
“Hazardous Decomposition Products:
 Encouraging segregation of waste at Gradually loses ammonia when exposed to
source and waste management. air at room temperature. Decomposes to
What is the Significance of a Clean ammonia and monoammonium phosphate
Ocean? at around 70 °C (158 °F). At 155 °C (311
°F), DAP emits phosphorus oxides,
 The ocean is a vital source of nourishment, nitrogen oxides and ammonia.
especially to people in the world’s poorest
nations. Uses:

 Many depend on fish for their primary  DAP is used as a fertilizer. When applied
as plant food, it temporarily increases the
source of protein; fisheries and aquaculture
soil pH, but over a long term the treated
support the livelihoods of about 540 million
ground becomes more acidic than before,

RE
people (8% of the world’s population) directly
upon nitrification of the ammonium. It
or indirectly.
is incompatible with alkaline chemicals
 Overfishing, loss of biodiversity and the because its ammonium ion is more likely
possible extinction of species put a stress to convert to ammonia in a high-pH
on these limited resources. This could lead environment. The average pH in solution is
to famine, increased poverty and conflicts, 7.5–8.
including war. The typical formulation is 18-46-0 (18% N,

O
Learning to manage the ocean sustainably

46% P2O5, 0% K2O).
is the only path to global prosperity and  DAP can be used as a fire retardant. It
peace. lowers the combustion temperature of
the material, decreases maximum weight
SC
Why in News?
loss rates, and causes an increase in the
 Recently, the Ministry of Earth Sciences has production of residue or char.
launched a Coastal Clean Up Drive under
 These are important effects in fighting
Swachh Sagar Surakshit Sagar Campaign
wildfires as lowering the pyrolysis
to clean 75 beaches across the country in 75
temperature and increasing the amount
days. of char formed reduces that amount of
available fuel and can lead to the formation
GS

66. Correct Option: (d) of a firebreak. It is the largest component


of some popular commercial fire fighting
Explanation: products and is the ingredient in “fire
retardant” cigarettes.
 Statement 4 is incorrect: The Central
government is responsible for regulating  DAP is also used as a yeast nutrient in
the subsidy prices of DAP. The state winemaking and mead-making; as an
government allows for procurement and additive in some brands of cigarettes
distribution purposes. purportedly as a nicotine enhancer; to
prevent afterglow in matches, in purifying
Supplementary notes: sugar; as a flux for soldering tin, copper,
zinc and brass; and to control precipitation
Diammonium Phosphate (DAP)
of alkali-soluble and acid-insoluble colloidal
 Diammonium phosphate (DAP); IUPAC dyes on wool.
name diammonium hydrogen phosphate;
chemical formula (NH4)2(HPO4) is one Why in News?
of a series of water-soluble ammonium  India has taken initiative to become
phosphate salts that can be produced when Aatmanibhar in Diammonium Phosphate
ammonia reacts with phosphoric acid. (DAP). Union Minister of Chemicals and
Fertilizers, Dr. Mansukh Mandaviya said
 Solid diammonium phosphate shows a
that Government of India continuously
dissociation pressure of ammonia as given
strengthening the raw material supply
by the following expression and equation: security to become Aatmanirbhar in DAP,
(NH4)2HPO4(s) NH3 (g) + (NH4)H2PO4(s) by investing in mineral rich countries.

PTS2023/PPP-01/092022/11 42

Telegram - @upscmaterialonline1
.
 India is highly dependence on raw material since 2016 and defence cooperation is a key
like rock phosphate and phosphoric acid, for pillar of this partnership.
production of DAP and NPK fertilizers.
 Bilateral defence engagements have
 Now India securing its supplies through expanded over a period of time to include
investments and long terms import deals wide-ranging contacts between the two
with mineral-rich countries. This will help countries, including Defense Policy
India to hedge against global price rises in Dialogue, military-to-military exchange,
Diammonium Phosphate (DAP) and NPK high-level visits, capacity building and
Complex. training programmes.
 The acquisition of 45% equity share in  The two countries signed a MoU on mutual
a Senegal based rock phosphate mining logistics support, the first of its kind that
company Baobab Mining and Chemicals Vietnam has signed with any country.
Corporation (BMCC) is formalized by India and Vietnam also agreed to the early
Coromandel International Ltd, India, in the finalization of the USD 500 million defence
presence of Indian Minister of Chemicals line of credit to Vietnam by India.
and Fertilizers.
 The Indo-Vietnam Cooperation assumes
significance in view of the US and several

RE
67. Correct Option: (b) world powers’ efforts to ensure a free, open
and thriving Indo-Pacific region, especially
Explanation: at a time when China has shown scant
 Statement 1 is incorrect: VINBAX is the respect for international laws to resolve the
joint military exercise between the military territorial disputes in the South China Sea.
of India and Vietnam. Why in News?
Supplementary notes:
Exercise VINBAX

O
VINBAX is the joint military exercise
 The 3rd edition of Vietnam India Bilateral
Army Exercise “Ex VINBAX 2022” will be
conducted recently. The exercise is a sequel
to previously conducted bilateral exercise
between the military of India and Vietnam. in Vietnam in 2019 and a major milestone
SC
in strengthening the bilateral relations
 VINBAX exercise is the first military
between India and Vietnam.
exercise cooperation helps reinforce the
India-Vietnam Defence and Security  The theme of Ex VINBAX – 2022 is
Cooperation and furthermore endorses the employment and deployment of an Engineer
India-Vietnam relations. Company and a Medical Team as part
of United Nations Contingent for Peace
 India and Vietnam signed a “Joint Vision”
Keeping Operations.
document to “significantly enhance the scope
GS

and scale” of bilateral ties by 2030 during  The conduct of Ex VINBAX – 2022 as a
the recent Vietnam visit of Indian Defence field training exercise with enhanced scope
Minister Rajnath Singh to Vietnam. from previous editions of bilateral exercise
will strengthen mutual confidence, inter-
 The two countries have many things in
operability and enable sharing of best
common. Both of them are listed among the
practices between the Indian Army and
fastest-growing economies of Asia and both
Vietnam People’s Army. The joint exercise
aim at enhancing peace and stability in the
will also provide an opportunity to the troops
Indo-Pacific region through multi-faceted
of both the Contingents to learn about the
cooperation.
social and cultural heritage of each other.
 India and Vietnam have a broad convergence Indian Army is being represented by troops
of interest as the emerging economies from the 105 Engineer Regiment.
and important countries with respect to
 A 48 hours Validation Exercise is part of the
creating alternative supply chains in Asia
schedule to assess the standards achieved
after global disillusionment with the China-
by both contingents while executing
centric supply chain during the COVID
technical military operations under similar
pandemic.
scenarios in UN missions. A Humanitarian
 Besides, the two countries are equally Assistance & Disaster Relief demonstration
concerned about the aggressive and and equipment display will showcase India’s
expansionist policies of China in the Indo- capacity to undertake rescue and relief
Pacific region. The two countries also share operations during natural and manmade
a Comprehensive Strategic Partnership disasters utilizing indigenous solutions.

43
PTS2023/PPP-01/092022/11

Telegram - @upscmaterialonline1
68. Correct Option: (b)

.
 IAS occurs in all taxonomic groups, including
animals, plants, fungi and microorganisms,
Explanation: and can affect all types of ecosystems. While
 Statement 1 & 3 is incorrect: It a small percentage of organisms transported
publishes Red data book which contains to new environments become invasive, the
a list of ‘Threatened species’ (vulnerable, negative impacts can be extensive and over
endangered and critically endangered). time, these additions become substantial. A
species introduction is usually vectored by
Supplementary notes: human transportation and trade.
Red Data Books  If a species’ new habitat is similar enough
 IUCN is an NGO. It publishes Red data book to its native range, it may survive and
which contains a list of ‘Threatened species’ reproduce. However, it must first subsist at
(vulnerable, endangered and critically low densities, when it may be difficult to find
endangered). mates to reproduce. For a species to become
invasive, it must successfully out-compete
 The red data book is maintained by the native organisms, spread through its new
International Union for Conservation of
environment, increase in population density
Nature.
and harm ecosystems in its introduced
This organization was founded in 1948 and range. To summarize, for an alien species

RE

works in the field of conservation of nature to become invasive, it must arrive, survive
and sustainable use of natural resources. and thrive.
 The red list contains only the names of the  Common characteristics of IAS include rapid
endangered species; however, the Red Data reproduction and growth, high dispersal
Book contains information about the species ability, phenotypic plasticity (ability to
that are on the verge of extinction. adapt physiologically to new conditions),
Why in News?
O and ability to survive on various food types
and in a wide range of environmental
 Lok Sabha recently passed by voice vote the conditions.
Wild Life (Protection) Amendment Bill-2021
that seeks to provide for implementation  A good predictor of invasiveness is whether
SC
of the Convention on International Trade a species has successfully or unsuccessfully
in Endangered Species of Wild Fauna and invaded elsewhere.
Flora (CITES). Why in News?
 Scientists have recently found that soil
69. Correct Option: (c) microbial activity plays an important role
Explanation: in making invasive species flourish in non-
native regions.
GS

 Statement 1 is correct: IAS occurs in


all taxonomic groups, including animals,  A recent study published in Nature indicates
plants, fungi and microorganisms, and that the plant species considered as invasive
can affect all types of ecosystems. While a in non-native regions, use characteristics to
small percentage of organisms transported survive and flourish that are different from
to new environments become invasive, the the ones in their native regions.
negative impacts can be extensive and over
time, these additions become substantial.
70. Correct option: (b)
 Statement 2 is correct: Islands are
especially vulnerable to IAS because they are Explanation:
naturally isolated from strong competitors  Option (b) is correct: G219 runs along the
and predators. Islands often have ecological course from Mazha in Xinjiang in the north,
niches that have not been filled because of through Aksai Chin.
the distance from colonizing populations,
increasing the probability of successful Supplementary notes:
invasions.
G695 Highway
Supplementary notes:
 G695 is another highway that China is
Invasive Alien Species planning to build through Aksai Chin,
running along the India border and
 Invasive alien species (IAS) are species
connecting Xinjiang with Tibet.
whose introduction and/or spread outside
their natural past or present distribution  The G695 national expressway will be only
threaten biological diversity. the second national highway through the

PTS2023/PPP-01/092022/11 44

Telegram - @upscmaterialonline1
71. Correct Option: (d)

.
disputed Aksai Chin region, where China
controls 38,000 sq km of land claimed by
India, since the controversial construction Explanation:
of the G219 highway in the 1950s, and is  Option (d) is correct
expected to be completed by 2035.
Supplementary notes:
 The new highway will run even closer to the
Line of Actual Control (LAC) than G219 and South Korea n Tattoo based Technology
is likely to broadly run along the course of  The device could be used to send readout
G219 from Mazha in Xinjiang in the north, of the wearer’s vital signs if connected
through Aksai Chin, which is currently to biosensors, including for instance an
administered under the Xinjiang Uygur electrocardiogram.
Autonomous Region, heading south along
 Alongside heart rates it could be used to
the borders with India, Nepal and Bhutan,
read glucose or lactate levels for people with
and down to Lhunze in southeastern Tibet diabetes or sepsis.
right across the border from Arunachal
Pradesh.  The monitors could in theory be located
anywhere, including in patients’ homes.
 Xinjiang is technically an autonomous
region within China — its largest region,  Working: The device is linked with an

RE
rich in minerals, and sharing borders with electrocardiogram (ECG) device or other
biosensor and it can send a read out of a
eight countries, including India, Pakistan,
patient’s heart rate and other vital signs
Russia, Afghanistan, Mongolia, Kazakhstan,
such glucose and lactate to a monitor. The
Kyrgyzstan and Tajikistan.
researchers eventually aim to be able to
 While a map of the proposed highway has dispense with biosensors.
not been released, the route described by
O Electronic Tattoo Ink
the report will mean the highway will likely
cut across Aksai Chin. Its course will bring  The electronic tattoo ink is made of liquid
it close to several disputed areas that have metal and carbon nanotubes that can deliver
seen recent tensions, from Eastern Ladakh vital body information.
down to near Doklam close to the India- The ink is non-invasive and made from
SC

China-Bhutan tri-junction. Details of the particles based on gallium, a soft, silvery
new construction remain unclear, but the metal also used in semiconductors or in
highway, when completed, may also go near thermometers.
hotly contested areas such as the Depsang
 Platinum-decorated carbon nanotubes
Plains, Galwan Valley and Hot Springs on help conduct electricity while providing
the LAC.” durability.
 The plan for G695 follows long-held When it is applied to the skin, even with
GS


discussions to build another border highway rubbing the tattoo doesn’t come off, this is
to supplement G219 and act as another not possible with just liquid metal.
key artery that could help China mobilise
resources rapidly towards border areas. Why in News?
China has also stepped up the construction  A team of scientists at the Korea Advanced
of secondary highways at the provincial level Institute of Science and Technology (KAIST)
that will connect the national highways to in the city of Daejeon southwest of Seoul
remote border areas. have developed an electronic tattoo ink
made of liquid metal and carbon nanotubes
Why in News? that functions as a bio-electrode.
 According to a recently released highway  According to a report by news agency
construction plan, China is planning to Reuters, South Koreans may soon be able to
build another highway through Aksai carry a device inside their own bodies in the
Chin, running along the India border and form of a bespoke tattoo that automatically
connecting Xinjiang with Tibet. alerts them to potential health problems, if
the outcomes are successful.
 The new national road network plan aims
to build 4, 61,000 km of roads — including
1,62,000 km of national expressways and 72. Correct Option: (c)
2,99,000 km of provincial highways — by
2035. Under the plan, all cities with a 1 lakh Explanation:
population will be connected to a national  Statement 3 is incorrect: James Webb
highway in 30 minutes. Space Telescope caught Cartwheel Galaxy

45
PTS2023/PPP-01/092022/11

Telegram - @upscmaterialonline1
.
that is located about 500 million light-years Murgi at Rdanag, Leh; Kaleshwar Mahadev
away and appears like a wheel of a wagon. Temple, Kalesar (Manyala Panchayat),
Himachal Pradesh has el=nlisted for the
Supplementary notes:
National Importance Tag.
Cartwheel Galaxy  ASI is the premier organization for the
 Cartwheel Galaxy, located about 500 archaeological research, scientific
million light-years away in the Sculptor analysis, and excavation of
constellation, has been caught with the archaeological sites, conservation and
James Webb Space Telescope. preservation of protected monuments.
 The structure appears like a wheel of  It is an attached office under the Department
a wagon, and Webb reveals the galaxy’s of Culture (Ministry of Culture).
central black hole along with the information
 ASI was founded in 1861 by Alexander
about star formation.
Cunningham who became its first Director-
 Astronomers called the galaxy a ‘ring General.
galaxy’ because of its two rings – a bright
inner ring surrounded by a colourful one. Why in News?

 The appearance of the galaxy is justified by  Twenty heritage sites, including two ancient
mounds at Haryana’s Rakhigarhi and

RE
the high-speed collisions that have taken
place internally between a large spiral the age-old Anangtal in Delhi, have been
galaxy and a smaller galaxy which is not identified for the national importance tag.
visible in the image.
 The rings have been discovered with 74. Correct option: (b)
an extremely hot dust filled bright core
consisting of gigantic young star clusters. Explanation:

O
The outer ring, expanded for 440 million  Statement 1 & 4 are incorrect: Ganges
years, consists of star formation and Canal was declared open in 1854 and
supernovas. separate public works department in every
province was established during the tenure
 Cartwheel galaxy has been of Lord Dalhousie as Governor-General
SC
explored earlier with the Hubble Space (1848-1856).
Telescope but the same had failed may be
due to the thick layer of dust which obstructs Supplementary notes:
the view. Webb with an infrared gaze
explored the uncovered part of the Lord Curzon
Cartwheel galaxy.  Born in 1859, George Nathaniel Curzon was
 Webb Telescope had not just revealed a British conservative politician who was
the observations of the galaxy’s present educated at the elite institutions of Eton and
GS

structural formations but also determined Oxford. He served as Under-Secretary of


that Cartwheel is in a very transitory State for India (1891-1892), and for Foreign
stage. Affairs (1895-1898), before being appointed
Viceroy of India in 1899.
Why in News?
 As viceroy, his administration was known
 Cartwheel Galaxy, located about 500 for intense activity and emphasis on
million light-years away in the Sculptor efficiency. He stated in his budget speech in
constellation, has been caught with the 1904, “Efficiency of administration is, in my
James Webb Space Telescope. The structure view, a synonym for the contentment of the
appears like a wheel of a wagon, and Webb governed.”
reveals the galaxy’s central black hole along
with the information about star formation.  Curzon created a separate Muslim majority
province of the North-West Frontier
Province, sent a British expedition to Tibet,
73. Correct Option: (c) established a separate police service, and
established the Archeological Survey of
Explanation:
India, in order to study and protect historical
 All statements are correct monuments.
Supplementary notes:  Early on in his career, Curzon earned some
praise from his colonial subjects, for taking
National Importance Tag
action against Europeans in a number of
 The sites include rock painting at high-profile racist attacks against Indians.
Chintakunta, Andhra Pradesh; rock art site In 1899, he punished white soldiers for

PTS2023/PPP-01/092022/11 46

Telegram - @upscmaterialonline1
.
raping a woman in Rangoon; he disciplined our [white settlement] dominions and still
soldiers of the 9th Lancers for beating an survive, but if we lost India, our sun would
Indian cook in Sialkot to death in 1902; sink to its setting.”
and he tried unsuccessfully to get the
Calcutta High Court to change the meagre Why in News?
punishment given to an Assam tea manager  The 119-year-old Curzon Gate in Bardhaman
for murdering a “coolie”. in West Bengal is at the centre of a political
 Appointment of Police Commission (1902) row after it emerged recently that the
under Sir Andrew Frazer to review police Trinamool Congress (TMC)-run Bardhaman
administration; Appointment of Universities municipality has decided to erect a statue
Commission (1902) and passing of Indian of Bardhaman’s Maharaja Bijay Chand
Universities Act (1904); Establishment of Mahatab and his wife Radharani in front of
Department of Commerce and Industry; the city’s prominent landmark.
Calcutta Corporation Act (1899); Ancient
 Politicians, historians, and heritage experts
Monuments Preservation Act (1904);
Partition of Bengal (1905); Young husband’s have questioned the ruling party’s decision,
Mission to Tibet (1904) etc. took place during with some claiming that the statutes would
his tenure. block the view of the structure, which
was built by Mahatab in 1903, when Lord

RE
 Curzon was both vexed and enraged by the Curzon, then Viceroy of India, visited
growing nationalist movement in India and Bardhaman.
he sought to throttle the growing aspirations
of the educated Indian middle class.
 A staunch imperialist, he took a series of 75. Correct Option: (b)
extremely unpopular measures, including Explanation:
passing, in 1899, the Calcutta Municipal
Statement 3 is incorrect: Section 45 of
Amendment Act which reduced the number
O
of elected representatives in the Calcutta
Corporation; the Indian Universities Act
(1904), that placed Calcutta University

the Prevention of Money Laundering Act
(PMLA) made offences to be cognizable and
non-bailable and no person accused of an
under government control, and the Indian offence shall be released on bail or on his own
SC
Official Secrets Amendment Act (1904) bond unless the Public Prosecutor is given
that reduced the freedom of the press even the opportunity to oppose the application
further. for such release and where the Public
 Curzon believed that the Indian National Prosecutor opposes the application, the
Congress had lost its influence and appeal court is satisfied that there are reasonable
amongst the Indians, and in 1900 declared grounds for believing that the accused is not
that the organisation was “tottering to its likely to commit any offence while on bail.
fall”. Ironically though, it was his biggest
GS

Supplementary notes:
and most reviled decision — to partition
Bengal in 1905 — that led to a spurt in Prevention of Money Laundering Act,
nationalist sentiment and revitalized the 2002
Congress.
 Prevention of Money Laundering Act, 2002
 In 1905, Curzon resigned and returned to is an Act of the Parliament of India enacted
England after losing a power struggle with by the NDA government to prevent money-
the commander-in-chief of the British Army,
laundering and to provide for confiscation of
Lord Kitchener. The protests continued after
property derived from money-laundering.
his exit, and the colonial government in
1911 announced the reunification of Bengal,  The provisions of this act are applicable to
and the capital of the Raj was shifted from all financial institutions, banks (Including
Calcutta to Delhi. RBI), mutual funds, insurance companies,
 Of all the Viceroys of India, Curzon is and their financial intermediaries.
possibly the most criticised — he is the  The Act enables government authorities to
man who partitioned Bengal in 1905, and confiscate property and/or assets earned
triggered a wave of Bengali nationalism from illegal sources and through money
that contributed to the wider Indian laundering.
national movement. He was also one of the
more openly imperialist of viceroys, and a  Under the PMLA, the burden of proof lies
man who saw Britain’s rule over India as with the accused, who has to prove that
critical to the survival of empire. In 1900, the suspect property/assets have not been
Curzon famously stated, “We could lose all obtained through proceeds of crime.

47
PTS2023/PPP-01/092022/11

Telegram - @upscmaterialonline1
.
 The basic objectives of the PMLA are:  In upholding the reverse burden of proof
condition for grant of bail, the Court also
 Preventing money laundering.
overruled its own order of November
 Combating the channelising of money 2017 that had declared the bail criteria
into illegal activities and economic unconstitutional.
crimes
 In Nikesh Tarachand Shah vs Union
 Providing for the confiscation of property of India (2017), the two-judge bench of
derived from or involved in money Justices Rohinton Nariman and Sanjay
laundering Kishan Kaul, had declared the ‘twin test’ of
 Providing for any other matters bail under PMLA as unconstitutional since
connected with or incidental to the act of it was manifestly arbitrary.
money laundering
 The Act prescribes that any person found 76. Correct Option: (b)
guilty of money-laundering shall be
punishable with rigorous imprisonment Explanation:
from three years to seven years and where  Statement 2 and 3 are incorrect:
the proceeds of crime involved relate to any These are the disqualifications laid by
offence under paragraph 2 of Part A of the the Parliament in the Representation of

RE
Schedule (Offences under the Narcotic Drugs People Act (1951) and not in the Indian
and Psychotropic Substance Act, 1985), the Constitution.
maximum punishment may extend to 10
years instead of 7 years. Supplementary notes:
 Section 43 of Prevention of Money Disqualifications for Membership of
Laundering Act, 2002 (PMLA) says that Parliament
the Central Government, in consultation
As per Article 191 (1), a person shall be
O
with the Chief Justice of the High Court,
shall, for trial of offence punishable under

disqualified for being chosen as, and for
Section 4, by notification, designate one or being, a member of the Legislative Assembly
more Courts of Session as Special Court or or Legislative Council of a State if:
Special Courts for such area or areas or for He holds any office of profit under the
SC

such case or class or group of cases as may Government of India or the Government
be specified in the notification. These special
of any State, other than an office declared
courts may declare a person as a Fugitive
by Parliament by law not to disqualify
Economic Offender.
its holder;
 Section 45 of the Prevention of Money
 He is of unsound mind and stands so
Laundering Act (PMLA) made offences to be
declared by a competent court;
cognizable and non-bailable and no person
accused of an offence shall be released on He is an undischarged insolvent;
GS


bail or on his own bond unless the Public
Prosecutor is given the opportunity to  He is not a citizen of India, or has
oppose the application for such release voluntarily acquired the citizenship
and where the Public Prosecutor opposes of a foreign State, or is under any
the application, the court is satisfied that acknowledgement of allegiance or
there are reasonable grounds for believing adherence to a foreign State;
that the accused is not likely to commit any  He is so disqualified by or under any law
offence while on bail. made by Parliament.
 Directorate of Enforcement is a specialized  Article 191 (2) states that a person shall
financial investigation agency under the be disqualified for being a member of the
Department of Revenue, Ministry of Finance, Legislative Assembly or Legislative Council
Government of India, which enforces the of a State if he is so disqualified under the
Prevention of Money Laundering Act, 2002
Tenth Schedule.
(PMLA).
 The Parliament has laid down the
Why in News? following additional disqualifications in the
 Making clear that the State has a compelling Representation of People Act (1951):
interest in imposing stringent bail conditions  He must not have been found guilty
for economic offences, the Supreme Court
of certain election offences or corrupt
recently upheld the Prevention of Money
practices in the elections.
Laundering Act, including its stringent bail
conditions that impose a reverse burden of  He must not have been convicted for any
proof on the accused. offence resulting in imprisonment for

PTS2023/PPP-01/092022/11 48

Telegram - @upscmaterialonline1
.
two or more years. But, the detention of Supplementary notes:
a person under a preventive detention
law is not a disqualification. Monkeypox Vaccine
 The monkeypox virus belongs to a family
 He must not have failed to lodge an
of viruses called orthopoxviruses, which is
account of his election expenses within
different from that of the coronaviruses.
the time.
 According to the WHO, it is a viral zoonosis
 He must not have any interest in
— a virus transmitted to humans from
government contracts, works or
animals — with symptoms similar, but less
services. severe to smallpox.
 He must not be a director or managing  It is also an enveloped double-stranded
agent nor hold an office of profit in a DNA virus, unlike the RNA virus, that
corporation in which the government makes it far more stable and less prone to
has at least 25 per cent share. rapid mutations.
 He must not have been dismissed from  There are two distinct genetic clades of the
government service for corruption or monkeypox virus: the central African (Congo
disloyalty to the State. Basin) clade and the West African clade. The
He must not have been convicted for Congo Basin clade has historically caused

RE

promoting enmity between different more severe disease and was thought to be
groups or for the offence of bribery. more transmissible.
 There is yet no dedicated monkeypox
 He must not have been punished for
vaccine, but vaccinations against smallpox
preaching and practicing social crimes
was found to be 85% effective in preventing
such as untouchability, dowry and sati.
monkeypox. In 2019, the United States Food
 On the question whether a member
O and Drugs Administration (FDA), approved
has become subject to any of the above the JYNNEOS vaccine for the prevention
disqualifications, the governor’s decision of smallpox, monkeypox and other diseases
is final. However, he should obtain the caused by orthopoxviruses, including
opinion of the Election Commission and act vaccinia virus, in adults 18 years of age and
older and categorised as having a “high risk
SC
accordingly.
of infection.”
Why in News?
 JYNNEOS, developed by Danish
 The Election Commission of India has biotechnology company, Bavarian Nordic,
recently recommended disqualification of contains a live vaccinia virus that does not
Jharkhand Chief Minister Hemant Soren replicate efficiently in human cells. The
from state Assembly for “violating electoral vaccinia virus is the smallpox virus but
law” by allegedly extending a mining lease made incapable of replicating within the
GS

to himself. body.
 The poll panel sent its recommendation to  There is no data yet on JYNNEOS’
the Jharkhand Raj Bhavan on Soren’s office effectiveness. This is because smallpox
of profit matter. has been eradicated and the monkeypox
outbreak has risen too rapidly for a
traditional phase 3 trial to have evaluated
77. Correct Option: (b) the vaccine’s effectiveness.
Explanation: Why in News?
 Statement 3 is incorrect: When a host is  The Indian Council of Medical Research
exposed to antigens, which may be in the has recently invited tenders from local
form of living or dead microbes or other companies to develop a vaccine.
proteins, antibodies are produced in the host
 The Serum Institute of India is also in talks
body. This type of immunity is called active
with international partners regarding a
immunity. Active immunity is slow and potential vaccine.
takes time to give its full effective response.
Injecting the microbes deliberately during
immunization or infectious organisms 78. Correct Option: (d)
gaining access into body during natural
infection induce active immunity. When Explanation:
ready-made antibodies are directly given to  Statement 1 is incorrect: The Right of
protect the body against foreign agents, it is Children to Free and Compulsory Education
called passive immunity. (RTE) Act, 2009 prohibits ‘physical

49
PTS2023/PPP-01/092022/11

Telegram - @upscmaterialonline1
.
punishment’ and ‘mental harassment’ under  While there is no statutory definition of
Section 17(1) and makes it a punishable ‘corporal punishment’ targeting children in
offence under Section 17(2). Section 17 of the Indian law, the Right of Children to Free
the Right to Education Act, 2009, imposes and Compulsory Education (RTE) Act, 2009
an absolute bar on corporal punishment. It prohibits ‘physical punishment’ and ‘mental
prohibits physical punishment and mental harassment’ under Section 17(1) and makes
harassment to children and prescribes it a punishable offence under Section 17(2).
disciplinary action to be taken against the  Section 17 of the Right to Education
guilty person in accordance with the service Act, 2009, imposes an absolute bar on
rules applicable to such person. corporal punishment. It prohibits physical
 Statement 2 is incorrect: The National punishment and mental harassment to
Commission for Protection of Child Rights children and prescribes disciplinary action
(NCPCR) guidelines for eliminating corporal to be taken against the guilty person in
punishment against children require every accordance with the service rules applicable
to such person.
school to develop a mechanism and frame
clear cut protocols to address grievances of  Section 75 of the Juvenile Justice Act
students. Drop boxes are to be placed where prescribes punishment for cruelty to
the aggrieved person may drop his complaint children.
and anonymity is to be maintained to protect

RE
 Whenever a child is assaulted, abused,
privacy. Every school has to constitute a exposed or neglected in a manner to cause
‘Corporal Punishment Monitoring Cell’ physical or mental suffering by any person
consisting of two teachers, two parents, one employed by or managing an organisation,
doctor, one lawyer (nominated by DLSA), which is entrusted with the care and
counsellor, an independent child rights protection of the child, the punishment
activist of that area and two senior students would be rigorous imprisonment upto five
from that school. This CPMC shall look into years and fine up to Rs 5 lakh.
complaints of corporal punishments.
O  If the child is physically incapacitated or
Supplementary notes: develops a mental illness or is rendered
mentally unfit to perform regular tasks or
Corporal Punishment has risk to life or limb, then imprisonment
SC
 By definition, corporal punishment means may extend upto ten years.
punishment that is physical in nature.  Section 23 of the JJ Act, 2000 states:
 According to the Guidelines for Eliminating “Whoever, having the actual charge of,
Corporal Punishment in Schools issued by or control over, a juvenile or the child,
the National Commission for Protection of assaults, abandons, exposes or wilfully
Child Rights (NCPCR), physical punishment neglects the juvenile or causes or procures
is understood as any action that causes him to be assaulted, abandoned, exposed or
neglected in a manner likely to cause such
GS

pain, hurt/injury and discomfort to a child,


however light. Examples include hitting, juvenile or the child unnecessary mental or
physical suffering shall be punishable with
kicking, scratching, pinching, biting, pulling
imprisonment for a term which may extend
the hair, boxing ears, smacking, slapping,
to six months, or fine, or with both.”
spanking, hitting with any implement (cane,
stick, shoe, chalk, dusters, belt, whip), giving  While Section 23 is likely to be applied most
electric shock and so on. It includes making often to personnel in childcare institutions
children assume an uncomfortable position regulated by the JJ Act, it arguably applies
(standing on bench, standing against the to cruelty by anyone in a position of authority
wall in a chair-like position, standing with over a child, which would include parents,
school bag on head, holding ears through guardians, teachers and employers.
legs, kneeling, forced ingestion of anything,  Meanwhile, the RTE Act does not preclude
detention in the classroom, library, toilet or the application of other legislation that
any closed space in the school. relates to the violations of the rights of the
 Mental harassment, meanwhile, is child, for example, booking the offenses
understood as any non-physical treatment under the IPC and the SC and ST Prevention
that is detrimental to the academic and of Atrocities Act of 1989.
psychological well-being of a child including  Several provisions of the Indian Penal
sarcasm, calling names and scolding using Code (IPC) relating to varying degrees
humiliating adjectives, intimidation, using of physical harm and intimidation can
derogatory remarks for the child, ridiculing be used to prosecute perpetrators of
or belittling a child, shaming the child and corporal punishment against children in
more. an institutional setting, depending on

PTS2023/PPP-01/092022/11 50

Telegram - @upscmaterialonline1
.
the situation and extent of the crime/act, high salinity and frequent cyclonic storms
including Section 305 pertaining to abetment and tidal surges. Mangroves are important
of suicide committed by a child, Section refuges of coastal bio-diversity and also
323 pertaining to voluntarily causing hurt act as bio-shields against extreme climatic
or Section 325 which is about voluntarily events. Large populations, primarily rural,
causing grievous hurt, and so on. depend on mangrove ecosystems for a wide
variety of biomass dependent livelihoods.
 The NCPCR guidelines for eliminating
corporal punishment against children  Important species of Mangrove ecosystems
require every school to develop a mechanism in India include Avicenna officinalis,
and frame clear cut protocols to address Rhizophora mucronata, Sonneratia alba,
grievances of students. Avicennia alba, Bruguiera cylindrica,
Heritiera littorals, Phoenix paludosa,
 Drop boxes are to be placed where the
Morinda citrifolia & Ceriops tagal.
aggrieved person may drop his complaint
and anonymity is to be maintained to  Sunderban, located in the Northern Bay
protect privacy. of Bengal is the world’s largest single
patch of Mangrove Forests. Spread over
 Every school has to constitute a ‘Corporal
approximately 10,000 sq km, in Bangladesh
Punishment Monitoring Cell’ consisting of
and India, Sundarban is the first mangrove
two teachers, two parents, one doctor, one

RE
forest in the world, which was brought
lawyer (nominated by DLSA), counsellor, an
under scientific management, as early as in
independent child rights activist of that area
1892.
and two senior students from that school.
This CPMC shall look into complaints of  As per the India State of Forest Report,
corporal punishments. 2021, the mangrove cover is classified into
3 categories:
 While a parent or caregiver can take the
protection of the IPC and the JJ Act to  Very Dense Mangroves [1,475 sq km

earmarked to ensure the protection of


children in schools.
O
file a police complaint in cases of corporal
punishment, there are relevant authorities

(29.55 %) of the Mangrove cover]
Moderately Dense Mangroves [1,481 sq
km (29.67 %)]
SC
 Open Mangroves [2,036 sq km (40.78
 Under Section 31 of the RTE Act, the National
%)]
Commission for Protection of Child Rights
(NCPCR) and the State Commissions for  The current assessment shows that
Protection of Child Rights (SCPCRs) have Mangrove cover in the country is 4,992 sq
been entrusted with the task of monitoring km, which is 0.15 % of the country’s total
children’s right to education. geographical area.
Why in News?  There has been a net increase of 17 sq km
in the Mangrove cover of the country as
GS

 Three private school teachers in Pune have


compared to 2019 assessment.
been recently booked under the Juvenile
Justice Act over allegedly thrashing three  The States that show significant gain in
Class 10 students, and threatening to grade Mangrove cover are Odisha (8 sq km) and
them poorly in internal assessments. Maharashtra (4 sq km). The reason for
the increase in Mangrove cover in Odisha,
is mainly due to the natural regeneration,
79. Correct Option: (a) plantation activities in suitable land like
Explanation: on the banks of the rivers near the estuary
and on intertidal mud-flats associated with
 Option (a) is correct the areas that are inundated by sea water
on a daily cycle. The increase in mangrove
Supplementary notes:
cover has been observed in the districts of
Mangroves in India Kendrapara, Jagatsinghpur and Balasore
in Odisha. In Maharashtra, the increase in
 Mangroves are salt tolerant plant
Mangrove cover is mainly due to natural
communities found in tropical and sub-
regeneration. The increase has also been
tropical intertidal regions of the world.
observed in the South 24 Parganas district
Such areas are characterized by high
of West Bengal.
rainfall (between 1,000 to 3,000 mm) and
temperature (ranging between 26°C-35°C).  Area wise mangrove cover distribution is as
Mangrove species exhibit a variety of follows: West Bengal > Gujarat > Andaman
adaptations in morphology, anatomy and & Nicobar > Andhra Pradesh > Maharashtra
physiology to survive in water logged soils, > Odisha.

51
PTS2023/PPP-01/092022/11

Telegram - @upscmaterialonline1
.
Why in News? Energy Conversion plant with a capacity of
65kW in Kavaratti Lakshadweep.
 The United States National Aeronautics
and Space Administration (NASA) recently  This plant will power the Low Temperature
highlighted the loss of mangrove cover on Thermal Desalination (LTTD) based
Katchal Island, a part of India’s Nicobar desalination plant for conversion of Sea
archipelago. water into Potable water.
 The NASA Earth Observatory showed a  The capacity of this LTTD plant is 1 lakh
map of the island depicting tidal wetland litre of potable water per day.
loss from 1999 through 2019. After the
magnitude-9.2 Aceh-Andaman earthquake
in December 2004, the islands experienced 81. Correct Option: (b)
up to 3 meters (10 feet) of land subsidence.
This submerged many mangrove ecosystems,
Explanation:
resulting in a loss of more than 90 percent  Statement 1 is incorrect: Carbon
of mangrove extent in some areas. nanotubes can exhibit remarkable electrical
conductivity. They also have exceptional
tensile strength and thermal conductivity
80. Correct Option: (c) because of their nanostructure and strength
of the bonds between carbon atoms. In fact,

RE
Explanation:
their thermal conductivity is better than
 Option (c) is correct that of diamond.
Supplementary notes: Supplementary notes:
Ocean Thermal Energy Carbon Nano Tubes (CNTs)
 A large amount of solar energy is stored in
 Carbon nanotubes (CNTs) are cylindrical
the oceans and seas.
O molecules that consist of rolled-up sheets
of single-layer carbon atoms (graphene).
 The process of harnessing this energy
is called OTEC (ocean thermal energy They can be single-walled (SWCNT) with a
conversion). It uses the temperature diameter of less than 1 nanometer (nm) or
differences between the surface of the ocean multi-walled (MWCNT), consisting of several
SC
and the depths of about 1000m to operate a concentrically interlinked nanotubes, with
heat engine, which produces electric power. diameters reaching more than 100 nm.
 Ocean thermal energy conversion (OTEC) is Their length can reach several micrometers
a process or technology for producing energy or even millimeters.
by harnessing the temperature differences  Apart from their electrical properties,
(thermal gradients) between ocean surface which they inherit from graphene, CNTs
waters and deep ocean waters. also have unique thermal and mechanical
GS

 Energy from the sun heats the surface properties that make them intriguing for
water of the ocean. In tropical regions, the development of new materials:
surface water can be much warmer than  Their mechanical tensile strength can be
deep water. This temperature difference 400 times that of steel;
can be used to produce electricity and to
desalinate ocean water. Ocean Thermal  They are very light-weight and their
Energy Conversion (OTEC) systems use density is one sixth of that of steel;
a temperature difference (of at least 77°  Their thermal conductivity is better than
Fahrenheit) to power a turbine to produce that of diamond;
electricity. Warm surface water is pumped
through an evaporator containing a working  They have a very high aspect ratio
fluid. The vaporized fluid drives a turbine/ greater than 1000, i.e. in relation to their
generator. The vaporized fluid is turned length they are extremely thin;
back to a liquid in a condenser cooled with
 Just like graphite, they are highly
cold ocean water pumped from deeper in the
chemically stable and resist virtually
ocean. OTEC systems using seawater as the
any chemical impact unless they
working fluid can use the condensed water
are simultaneously exposed to high
to produce desalinated water.
temperatures and oxygen - a property
Why in News? that makes them extremely resistant to
corrosion;
 The National Institute of Ocean Technology
(NIOT), an autonomous Institute under  Their hollow interior can be filled with
Ministry of Earth Sciences (MoES) is various nanomaterials, separating and
recently establishing an Ocean Thermal shielding them from the surrounding

PTS2023/PPP-01/092022/11 52

Telegram - @upscmaterialonline1
.
environment - a property that is Why in News?
extremely useful for nanomedicine
 Researchers at the Korea Advanced Institute
applications like drug delivery.
of Science and Technology (KAIST) in the
 All these properties make carbon nanotubes city of Daejeon southwest of Seoul have
ideal candidates for electronic devices, recently developed an electronic tattoo ink
chemical/electrochemical and biosensors, made of liquid metal and carbon nanotubes
transistors, electron field emitters, lithium- that functions as a bio-electrode.
ion batteries, white light sources, hydrogen
storage cells, cathode ray tubes (CRTs),  Hooked up to an electrocardiogram (ECG)
electrostatic discharge (ESD) and electrical- device or other biosensor, it can send a
shielding applications. readout of a patient’s heart rate and other
vital signs such glucose and lactate to a
 Various applications of CNTs are: monitor.
 Currently, CNTs are mainly used  The researchers eventually aim to be able to
as additives to synthetics. CNTs are dispense with biosensors.
commercially available as a powder, i.e.
in a highly tangled-up and agglomerated
form. For CNTs to unfold their particular 82. Correct Option: (c)
properties they need to be untangled and

RE
Explanation:
spread evenly in the substrate.
 Option (c) is correct
 Carbon nanotubes can also be spun into
fibers, which not only promise interesting Supplementary notes:
possibilities for specialty textiles.
Classical Languages in India
 Despite the rise of graphene and
other two-dimensional (2D) materials,  Currently, six languages enjoy the ‘Classical’
status: Tamil (declared in 2004), Sanskrit
O
semiconducting single-walled carbon
nanotubes are still regarded as strong
candidates for the next generation of
high-performance, ultra-scaled and 
(2005), Kannada (2008), Telugu (2008),
Malayalam (2013), and Odia (2014).
According to the Ministry of Culture, the
thin-film transistors as well as for opto- guidelines for declaring a language as
SC
electronic devices to replace silicon ‘Classical’ are:
electronics.
 High antiquity of its early texts/recorded
 Ink formulations based on CNT history over a period of 1500-2000
dispersions are attractive for printed years;
electronics applications such as
transparent electrodes, RFID tags, thin-  A body of ancient literature/texts, which
film transistors, light-emitting devices, is considered a valuable heritage by
and solar cells generations of speakers;
GS

 Carbon nanotubes have been widely used  The literary tradition be original and
as electrodes for chemical and biological not borrowed from another speech
sensing applications and many other community;
electrochemical studies.  The classical language and literature
 There is considerable interest in being distinct from modern, there may
using CNTs for various biomedical also be a discontinuity between the
applications. The physical properties classical language and its later forms or
of CNTs, such as mechanical strength, its offshoots.
electrical conductivity, and optical
 Benefits once a language is notified as a
properties, could be of great value for
Classical language are:
creating advanced biomaterials. Carbon
nanotubes can also be chemically  Two major annual international awards
modified to present specific moieties for scholars of eminence in classical
(e.g., functional groups, molecules, and Indian languages
polymers) to impart properties suited
 A Centre of Excellence for studies in
for biological applications, such as
Classical Languages is set up
increased solubility and biocompatibility,
enhanced material compatibility and  The University Grants Commission is
cellular responsiveness. Nitrogen- requested to create, to start with at least
doped carbon nanotubes for instance in the Central Universities, a certain
have been developed for drug delivery number of Professional Chairs for the
applications. Classical Languages so declared.

53
PTS2023/PPP-01/092022/11

Telegram - @upscmaterialonline1
.
 The University Grant Commission the environment to sustain life and grow.
(UGC) also awards research projects for
 The primary source of living is agriculture.
promoting these languages.
On small land holdings, the villagers grow
 Ministry of Culture has listed the maize, wheat and vegetables like okra,
institutions that have been dedicated to ridged gourd, bottle gourd, tomato, etc.
Classical languages. These are:
 The smallholding nature of farms allows
 Sanskrit: Rashtriya Sanskrit Sansthan, them to use mechanical agricultural
New Delhi; Maharishi Sandipani equipment easily. The women in the region
Rashtriya Ved Vidya Pratishthan, play a key role in promoting a sustainable,
Ujjain; Rashtriya Sanskrit Vidyapeetha, integrated farming system.
Tirupati; and Sri Lal Bahadur Shastri
 Soil conservation practices like farm
Rashtriya Sanskrit Vidyapeeth, New
bunding and crop rotation are followed,
Delhi
which have helped in soil conservation and
 Telugu and Kannada: Centres of promoted the diversification of crops. Even
Excellence for Studies in the respective during the lockdown brought on by the
languages at the Central Institute of COVID-19 pandemic, tribal communities
Indian Languages (CIIL) established by faced no shortage of food owing to their age-
the HRD Ministry in 2011. old grain storage practices.

RE
 Tamil: Central Institute of Classical  When it comes to food and dietary
Tamil (CICT), Chennai habits, the communities most commonly
consume foodgrains like kodra (Paspalum
Why in News? scrobiculatum), bati (foxtail millet), kang,
 On the occasion of World Sanskrit Day, Prime (barnyard millet), cheena (proso millet),
Minister Shri Narendra Modi acknowledged hama, hamli and gujro (little millet), along
the efforts of those working for popularizing with local vegetables. The minor millets are
O
Sanskrit and also highlighted the growing rich in fibre and iron and improves their
popularity of Sanskrit amongst the youth. immunity.
 The immense amount of knowledge these
communities have regarding food gathering
83. Correct Option: (b)
SC
and cultivation sets an example, especially
Explanation: for those on the path of industrial farming.
 Option (b) is correct  A look inside the huts of these communities
is a delight. There are special devices to
Supplementary notes: protect valuables during festivals and store
Tribal Culture in Banswara Villages grains in case of crop failures. Grains are
stored in a special container called Kothi,
 Time has not affected tribal traditions and which has a hidden space for keeping
GS

practices in Rajasthan’s Banswara district. valuables like money, jewellery, etc.


The region, abundant in natural resources
and beautiful landscapes, lies at the junction  The kitchen tools, cookware and utensils
of Rajasthan, Gujarat and Madhya Pradesh used by the tribal women are made of
states. environment-friendly materials. The
chullah (stove) is typically made of clay.
 The rhythm of tribal life in the region is Some tribal homes also feature double
isolated from the developing urban world chullahs, where one side is used for making
and one can see forms of cultural practices chapattis and the other side for cooking rice
dating several hundred years. or vegetables.
 The region’s semi-arid and mixed-  The homes also have a traditional ghatti,
miscellaneous forests are home to a large which is used for grinding grains, minor
population of tribals from the Bhil and millets and pulses. A masaladani (spice box)
Damor communities. Trees like Anogeissus and anajdani (Grain box) are also present
pendula (Dhok), Terminalia arjuna in all kitchens. The utensils are made of
(Arjun), Albizia lebbeck (Saras), Dalbergia black clay or grey soil. Special ones are used
paniculata (Palash) etc. make up the forests during the festivals or to welcome guests.
in the region.
 The communities worship Sherabavs
 The hamlets, farms and the simple life of Khankhaliya Dev and Shira Bavasi 14 days
these communities are clearly visible while after Diwali. The communities also pray to
walking around the deciduous forests. The Shira Bavasi after the first crop of the year
villagers and nature live side-by-side in this — the kharif crop — is ready and ancestors
region, mutually exchanging the secrets of are offered bhog.

PTS2023/PPP-01/092022/11 54

Telegram - @upscmaterialonline1
.
 The Gobind Guru Tribal University, is its high ash fusion temperature of about
Banswara, has set up a special museum 1,500 degree C.
to preserve the rich tribal wealth. Antique
 The ash chemistry of Indian coal is such
agricultural equipment, dresses, weapons
that it is high in silica and alumina. The
and daily items are kept for conservation in
ash is also highly abrasive because of high
the museum.
quartz content, which can lead to erosion
Why in News? of the syngas cooling system when it gets
fused.
 August 9 is celebrated as International Day
of the World’s Indigenous People.  However, Indian coals sulphur content is
low, about 0.5 per cent. So, from a gas clean-
 Traditional way of living dating back several up perspective, the flue gas desulphurization
hundred years is still practiced in Banswara (removal of SOx gases) and NOx removal
villages. Thus, there is ample scope for skill system is not economically justifiable and,
development in the region as tribes possess therefore, not important.
unique skills in pottery, archery, etc.
 As per the present import policy, coal can
be freely imported (under Open General
84. Correct Option: (d) Licence) by the consumers themselves
considering their needs based on their
Explanation:

RE
commercial prudence.
 All statements are correct
 Coking Coal is being imported by Steel
Supplementary notes: Authority of India Limited (SAIL) and
other Steel manufacturing units mainly to
Coal Sector in India bridge the gap between the requirement
 Coal is one of the most important primary and indigenous availability and to improve
fossil fuels, a solid carbon-rich material that the quality of production.


O
is usually brown or black and most often
occurs in stratified sedimentary deposits.
Coal is a natural resource that can be used
 Coal based power plants, cement plants,
captive power plants, sponge iron plants,
industrial consumers and coal traders are
as a source of energy, and in the production importing non-coking coal.
SC
of coke for metallurgical processes. Coal is a
Why in News?
major source of energy in the production of
electrical power using steam generation.  The parliamentary standing committee on
energy has recently said that the union
 In addition, gasification and liquefaction of
ministries of power and coal should ensure
coal produce gaseous and liquid fuels that
optimum utilization of locally-produced
can be transported by pipeline and stored
coal through effective collaboration so that
in tanks.
imports can be phased out over time.
GS

 India is rich in ancient coal fields like


 In its 26th report, the panel said efforts made
Gondwana coalfields (250 million years old)
by both ministries led to a 56% drop in coal
and tertiary coalfields (15-60 million years
imports for blending in FY21, but higher
old).
international prices hit imported coal-based
 The most significant characteristic of Indian power plants. This led to increased demand
coal is its high ash content, which varies for domestic coal, and consequently reduced
from 35 to 45 per cent, compared with that domestic coal stocks at power plants.
of coal in other parts of the world, which is
 The recommendations on phasing out
around 15 per cent.
imports tracks India’s emergence out
 With legislation requiring, since June 2001, of a severe coal crisis during April and
thermal power plants to use coal with less May when the country was in the midst
than 35 per cent ash content, and with the of an unprecedented heatwave leading
quality of Indian coal deteriorating over to a surge in power demand. This forced
the years, development of more efficient the government to float tenders for coal
technologies that can handle high-ash imports.
Indian coal, such as Integrated Gasification
 The committee urges the ministry to learn
Combined Cycle, becomes important.
appropriate lessons from the 2021 episode,
 Besides high ash content, another reason it reiterates its recommendation that the
for entrained gasifiers (that operate at ministry of power and the ministry of coal
higher temperatures than fluidized bed should collaborate effectively on this issue
gasifiers), commonly used in IGCC plants and make concerted efforts for optimum
abroad, being not suitable for Indian coal utilization of domestic coal reserves keeping

55
PTS2023/PPP-01/092022/11

Telegram - @upscmaterialonline1
.
in view the basic objective of doing away with sell. This is done through the Fertiliser
the import of coal in a phased manner. (Movement) Control Order, 1973. Under
this, the department of fertilisers draws
an agreed monthly supply plan on all
85. Correct Option: (b) subsidized fertilisers in consultation
Explanation: with manufacturers and importers.

 Statement 1 is incorrect: Under the  When the government is spending vast


new “One Nation One Fertiliser” scheme, sums of money on fertiliser subsidy (the
companies are allowed to display their bill is likely to cross Rs 200,000 crore
name, brand, logo and other relevant in 2022-23), plus deciding where and at
product information only on one-third space what price companies can sell, it would
of their bags. On the remaining two-thirds obviously want to take credit and send
space, the “Bharat” brand and Pradhan that message to farmers.
Mantri Bharatiya Jan Urvarak Pariyojana  A couple of issues are immediately
logo will have to be shown. apparent:
 Statement 3 is incorrect: “Model  It will disincentivize fertiliser companies
Fertilizer Retail Shop” also called “Pradhan from undertaking marketing and brand
Mantri Kisan Samridhi Kendra” function as promotion activities. They will now

RE
“One Stop Shop” for Indian farmers for all be reduced to contract manufacturers
quality Agri Inputs like fertilizers, seeds, and importers for the government.
pesticides at reasonable prices under one Any company’s strength ultimately is
roof, providing services of small and large its brands and farmer trust built over
agri and farm implements through sale or decades.
custom Hiring, Testing Facilities for Soil/
Seed/Fertilizer or connecting the farmers  Currently, in case of any bag or batch
O
to those testing Facilities centres and of fertilisers not meeting the required
standards, the blame is put on the
Advisory Services etc. These shops are not
established under this scheme. company. But now, that may be passed
on fully to the government.
Supplementary notes:
SC
Why in News?
One Nation One Fertiliser Scheme
 The Ministry of Chemicals and Fertilisers
 Under the new “One Nation One Fertiliser” announced recently that it has been decided
scheme, companies are allowed to display to implement One Nation One Fertiliser by
their name, brand, logo and other relevant introducing a “Single Brand for Fertilisers
product information only on one-third space
and Logo” under the fertiliser subsidy
of their bags. On the remaining two-thirds
scheme named “Pradhan Mantri Bharatiya
space, the “Bharat” brand and Pradhan
Janurvarak Pariyojna” (PMBJP).
GS

Mantri Bharatiya Jan Urvarak Pariyojana


logo will have to be shown.  The single brand name for UREA, DAP,
MOP and NPK etc. would be BHARAT
 The government’s logic for introducing a
UREA, BHARAT DAP, BHARAT MOP
single ‘Bharat’ brand for all subsidized
and BHARAT NPK etc. respectively for
fertilisers being marketed by companies is
all Fertiliser Companies, State Trading
as follows:
Entities (STEs) and Fertiliser Marketing
 The maximum retail price of urea is Entities (FMEs).
currently fixed by the government,
which compensates companies for the
higher cost of manufacturing or imports 86. Correct Option: (a)
incurred by them. The MRPs of non-urea Explanation:
fertilisers are, on paper, decontrolled.
But companies cannot avail of subsidy  Statement 1 is incorrect: The
if they sell at MRPs higher than that Mānavadharmaśāstra, also known as
informally indicated by the government. Manusmriti or the Laws of Manu, is a
Simply put, there are some 26 fertilisers Sanskrit text belonging to the Dharmaśāstra
(inclusive of urea), on which government literary tradition of Hinduism. Unlike
bears subsidy and also effectively decides Dharmasutras which are based upon
the MRPs Vedas, Dharmaśāstras are mainly based on
Puranas.
 Apart from subsidizing and deciding
at what price companies can sell, the  Statement 2 is incorrect: Manusmriti has
government also decides where they can 4 major divisions: 1) Creation of the world.

PTS2023/PPP-01/092022/11 56

Telegram - @upscmaterialonline1
.
2) Sources of dharma. 3) The dharma of the religious significance as well. It was the
four social classes. 4) Law of karma, rebirth, first Sanskrit text to be translated into
and final liberation. The third section is the a European language, by the British
longest and most important section. The philologist Sir William Jones in 1794.
text is deeply concerned with maintaining Subsequently, it was translated into French,
the hierarchy of the four-fold varna system German, Portuguese and Russian.
and the rules that each caste has to follow.
It is encyclopedic in scope, covering subjects  For colonial officials in British India, the
such as the social obligations and duties translation of the book served a practical
of the various castes and of individuals in purpose. In 1772, Governor-General Warren
different stages of life, the suitable social Hastings decided to implement laws of
and sexual relations of men and women of Hindus and Muslims that they believed
different castes etc. to be continued, unchanged from remotest
antiquity. For Hindus, the dharmasastras
Supplementary notes: were to play a crucial role, as they were
Manusmriti seen by the British as ‘laws,’ whether or not
it was even used that way in India.
 The Mānavadharmaśāstra, also known
as Manusmriti or the Laws of Manu, is a  The ancient text has 4 major divisions: 1)
Sanskrit text belonging to the Dharmaśāstra Creation of the world. 2) Sources of dharma.

RE
literary tradition of Hinduism. 3) The dharma of the four social classes. 4)
Law of karma, rebirth, and final liberation.
 Composed sometime between the 2nd The third section is the longest and most
century BCE and 3rd century CE, the important section. The text is deeply
Manusmriti is written in sloka verses, concerned with maintaining the hierarchy
containing two non-rhyming lines of 16 of the four-fold varna system and the rules
syllabus each. O that each caste has to follow.
 The text is attributed to the mythical figure
of Manu, considered to be ancestor of the
Why in News?
human race in Hinduism. There has been  The Vice Chancellor of Jawaharlal Nehru
considerable debate between scholars on the University, Santishree Dhulipudi Pandit,
authorship of the text. Many have argued recently criticised the Manusmriti, the
SC
that it was compiled by many Brahmin ancient Sanskrit text, over its gender bias.
scholars over a period of time. However, Her remarks came while delivering the
some argue that Manusm iti’s “unique and keynote address at the B R Ambedkar Lecture
symmetrical structure,” means that it was Series organised by the Union Ministry of
composed by a “single gifted individual,” or Social Justice and Empowerment.
by a “strong chairman of a committee” with
the aid of others.  Pandit said that the Manusmriti has
categorised all women as “shudras”, which
GS

 The Manusmriti is encyclopedic in scope, is “extraordinarily regressive”. She also


covering subjects such as the social
referred to recent case in which a Dalit
obligations and duties of the various castes
boy in Rajasthan was allegedly beaten by
and of individuals in different stages of
his teacher for touching a water pot, and
life, the suitable social and sexual relations
subsequently died due to his injuries.
of men and women of different castes, on
taxes, the rules for kingship, on maintaining  Referring to Ambedkar’s landmark
marital harmony and the procedures for “Annihilation of Caste”, she said, “If Indian
settling everyday disputes. society wants to do well, annihilation of
 At its core, the Manusmriti discusses life caste is extraordinarily important.”
in the world, how it is lived in reality, as
well as how it ought to be. It also discusses 87. Correct Option: (c)
aspects of the Arthashashtra, such as issues
relating to statecraft and legal procedures. Explanation:
 The aim of the text is to “present a blueprint  Option (c) is correct
for a properly ordered society under the
sovereignty of the king and the guidance of Supplementary notes:
Brahmins.” Aashwasan Campaign
 It was meant to be read by the priestly  Aashwasan Campaign was a case-finding
caste.
campaign to detect instances of Tuberculosis
 European Orientalists considered the (TB) among tribal populations under ‘Tribal
Manusmriti to be of great historical and TB Initiative’.

57
PTS2023/PPP-01/092022/11

Telegram - @upscmaterialonline1
88. Correct Option: (a)

.
 Tribal TB Initiative is a joint initiative of
the Ministry of Tribal Affairs and Central
TB Division, Ministry of Health, supported Explanation:
by USAID as technical partner and Piramal  Statement 2 is incorrect: ‘Zorawar’ is
Swasthya as implementing partner. an indigenous Indian light tank designed
 The Aashwasan Campaign was started to operate in varying terrain from high
for active case finding for TB in 174 tribal altitude areas and marginal terrains to
districts of India, under the ambit of the island territories.
Tribal TB Initiative. It was flagged off in Supplementary notes:
Nandurbar District, Maharashtra.
Swarm Drones
 Under the initiative, door-to-door screening
for TB was undertaken covering 68,019  Swarm robotics is an approach to the
villages. Based on verbal screening of coordination of multiple autonomous robots
1,03,07,200 persons, 3,82,811 people were as a system which consists of a large number
identified for Presumptive TB. Of these, of mostly physical robots, controlled by
2,79,329 (73%) samples were tested for TB minimal human intervention. These exhibit
and 9,971 people were found to be positive collective self organising (SO) behaviour
for TB and put on treatment as per GoI through interaction and cohesion between
robots, as well as interaction of robots with

RE
protocols.
the environment.
 The Aashwasan Campaign brought together
 Swarming algorithms are empowered by
about 2 lakh community influencers who
biological studies of swarm behaviour of
wholeheartedly participated to make the
insects, fishes, birds and animals. Swarming
campaign successful. These include tribal
R&D across the world is focused on
leaders, tribal healers, PRI members, SHGs
development of distributed artificial swarm
and youth in the tribal areas- who were part
intelligence capability, commodification
O
of this campaign in the screening process
of technology for lesser cost impact and
and community awareness. increasing state of autonomy between the
 As a way forward, to mark 75 years’ of agents in a swarm.
India’s Independence, 75 high burden tribal  While massed drones in spectacular light
SC
districts have been selected for focused shows are all controlled centrally, in a true
interventions in the coming months. A swarm, each of the drones flies itself following
three-pronged strategy for the 75 Districts onboard AI to maintain formation and avoid
was presented, to be centered on: collisions with algorithms mimicking nature
 Generating demand for TB services — there is no true leader and follower, with
through continued engaged with all agents in a swarm having their own
community influencers who have ‘mind’ able to undertake collective decision-
been mapped during this process for making, adaptive formation flying, and self-
GS

community mobilization, increasing healing. The benefit of such a swarm is that


awareness on TB, symptoms, spread and if one drone drops out — and a few appear
to crash — the group can rearrange itself
treatment processes, and addressing
to continue undertaking the mission till the
stigma and fear associated with TB.
last UAV in air.
 Improving the delivery of TB services by
 The swarm drone system should be able to
enhancing the TB testing and diagnosis
scan and detect sub-centimeter size foreign
infrastructure, leveraging PIPs and
objects on aircraft operating surfaces. The
other sources of funding to address
specifications of the surfaces have been
implementation gaps and provision of defined.
customized solutioning
 The system should be able to carry a range of
 Decreasing the risk of transmission and payloads and sensors, have image correction
decreasing the pool of infections through ability, provide alerts and provide accurate
active case finding campaigns location and discernable image of foreign
Why in News? objects.

 Ministry of Tribal Affairs and TB division  The system should also be able to operate
of Ministry of Health and Family welfare in GPS denied environment, complete
recently organized a national conclave at the operating cycle within 30 minutes and
operate day and night and in reduced
National Tribal Research Institute (NTRI),
visibility as well as light windy conditions.
New Delhi to disseminate the learnings of
the 100-day Aashwasan Campaign under  There are a many uses for such a drone
‘Tribal TB Initiative’. swarm. The drones could be released by

PTS2023/PPP-01/092022/11 58

Telegram - @upscmaterialonline1
.
fighters to provide reconnaissance for troops expenditure for implementation of the
on the ground, hunting enemy forces and scheme will be met from the “Senior Citizens’
reporting their location. They could also jam Welfare Fund”.
enemy communications, form a wide-area
 The Scheme will be implemented through the
flying communications network, or provide
sole implementing agency - Artificial Limbs
persistent surveillance of a particular area.
Manufacturing Corporation (ALIMCO), a
They could be loaded with small explosive
PSU under the Ministry of Social Justice
charges and attack individual enemy
and Empowerment.
soldiers. In air-to-air combat, they could
spoof enemy radars on aircraft, ground  Under the scheme, the physical aids will be
vehicles, and missiles by pretending to be provided only to the senior citizens of the
much larger targets. nation. This implies those who are aged
above 60 years will get free assisted living
Why in News? aids and physical devices which are required
 The Indian Army has recently procured for their sustainability.
swarm drones from two Indian start-ups. In  Also the govt. has selected the list of cities
addition, a Make-II case has been initiated, where the scheme will be implemented. At
the Autonomous Surveillance and Armed
present, a total of 325 Districts have been
Drone Swarm (A-SADS), this includes a
selected for the implementation of Rashtriya
number of improvements and also a version

RE
Vayoshri Yojana.
for high altitude areas.
 The main criteria for the senior citizens to get
 The Indian Army is also prioritising the
full benefits of Rashtriya Vayoshri Yojana
procurement of the indigenous Indian
(RVY) scheme is that they must belong to
light tank, aptly named ‘Zorawar’, for
BPL family and should hold valid BPL card
deployment in the mountains, in addition to
issued by the concerned authority.
inducting niche technologies like loitering
munitions, anti-drone capabilities, and Salient features of the scheme are:
O
next generation Intelligence, Surveillance
and Reconnaissance (ISR) capabilities to
outmanoeuvre the movement of Chinese

 Free of cost distribution of the devices,
commensurate with the extent of
disability/infirmity that is manifested
forces during the stand-off in eastern
among the eligible senior citizens.
SC
Ladakh.
 Zorawar is designed to operate in varying  In case of multiple disabilities/infirmities
terrain from high altitude areas and manifested in the same person, the
marginal terrains to island territories. assistive devices will be given in respect
It will be highly transportable for rapid of each disability/impairment.
deployment to meet any operational  Artificial Limbs Manufacturing
situation, and feature niche technologies, Corporation (ALIMCO) will undertake
including Artificial Intelligence (AI), drone one year free maintenance of the aids &
GS

integration, active protection systems, and assisted living devices.


a high degree of situational awareness.
 Beneficiaries in each district will be
identified by the State Governments/UT
89. Correct Option: (d) Administrations through a Committee
chaired by the Deputy Commissioner/
Explanation: District Collector.
 All statements are correct
 As far as possible, 30% of the beneficiaries
Supplementary notes: in each district shall be women.

Rashtriya Vayoshri Yojana  The devices are distributed in the camp


mode. For the senior citizens aged 80
 Rashtriya Vayoshri Yojana is a scheme for years or more, the devices are provided
providing Physical Aids and Assisted-living at their doorsteps.
Devices for Senior citizens belonging to BPL
category. Under the Scheme, assisted living Why in News?
devices such as Walking Sticks, Elbow
 A ‘Samajik Adhikarita Shivir’ for distribution
Crutches, Walkers/ Crutches, Tripods/
of aids and assistive devices to Senior citizens
Quadpods, Hearing Aids, Wheelchairs,
under ‘Rashtriya Vayoshri Yojana’ (RVY
Artificial Dentures and Spectacles are
Scheme) and to ‘Divyangjan’ under the ADIP
provided free of cost to the beneficiary senior
Scheme of the Ministry of Social Justice and
citizens.
Empowerment, Government of India was
 This is a Central Sector Scheme, fully recently organized by the Department of
funded by the Central Government. The SJ&E in association with ALIMCO, Nagpur

59
PTS2023/PPP-01/092022/11

Telegram - @upscmaterialonline1
.
Municipal Corporation (NMC) and District  Consecutive La Ninas following a transition
Administration Nagpur at Reshimbagh through ENSO neutral conditions are
Ground in Nagpur (Maharashtra). referred to as a “Double-Dip.”
 La Niña usually aids in the formation,
90. Correct Option: (d) intensification and propagation of hurricanes
in the North Atlantic Ocean.
Explanation:
 La Niña generally brings cooler than normal
 Statement 1 is incorrect: While La Niña temperatures across the world and heavy
conditions enhance the rainfall associated rainfall to places like India while causing
with the Southwest monsoon; it has a drought in many regions of North America.
negative impact on rainfall associated with
the Northeast monsoon. During La Niña  The wind patterns over the Atlantic Ocean
years, the synoptic systems — low pressure during La Niña also become conducive for
or cyclones — formed in the Bay of Bengal the genesis of hurricanes.
remain significantly to the north of their  La Niña events are also associated with
normal position. Besides, instead of moving rainier-than-normal conditions are over
westwards, these systems recurve. As they southeastern Africa and northern Brazil.
lie to the north of their normal position, not
much rainfall occurs over southern regions  La Niña is also characterized by higher-

RE
than-normal pressure over the central and
like Tamil Nadu.
eastern Pacific. This results in decreased
Supplementary notes: cloud production and rainfall in that region.
Drier-than-normal conditions are observed
La Nina along the west coast of tropical South
 La Nina is the opposite of El Nino America, the Gulf Coast of the United
characterized by occurrence of cooler waters States, and the pampas region of southern
in East Pacific.
O South America.
 The word La Nina means ‘Little Girl’.  Strong La Niña events are associated with
catastrophic floods in northern Australia.
 Since the waters are colder than normal in
East Pacific, there is no reversal of pressure Why in News?
SC
conditions; instead the normal conditions
 Australia’s Bureau of Meteorology (BOM)
get intensified during La Nina years.
has recently predicted that a third
 While La Niña conditions enhance the consecutive event of La Nina could be
rainfall associated with the Southwest underway, leading to additional rainfall
monsoon, it has a negative impact on rainfall along its east coast this summer.
associated with the Northeast monsoon.
 The BOM predicted a 70 per cent chance
 During La Niña years, the synoptic systems of the La Nina event. However, the United
GS

— low pressure or cyclones — formed in States and UK weather agencies have


the Bay of Bengal remain significantly to claimed that the phenomenon is already
the north of their normal position. Besides, underway.
instead of moving westwards, these systems
recurve. As they lie to the north of their
normal position, not much rainfall occurs 91. Correct Option: (b)
over southern regions like Tamil Nadu. Explanation:
 The SOI (Southern Oscillation Index)  Statement 1 is incorrect: The Convention
calculated on the basis of atmospheric did not require countries to take concrete
pressure differences between Tahiti and actions to control ozone-depleting
Darwin; gives an indication of development substances. Instead, a provision was added
of El Nino or La Nina. that Protocols to control these substances
 Sustained positive values of SOI are would be adopted if and when warranted.
indicative of La Nina conditions while Supplementary notes:
negative values suggest El Nino conditions.
Vienna Convention and Montreal
 La Niña is the cooling phase of the El
Protocol
Niño Southern Oscillation phenomenon in
which cooler sea surface temperatures in  By 1985, the globe had already seen
the eastern and central equatorial Pacific advancements in the scientific understanding
Ocean disrupt the trade winds over the sea of ozone depletion and its impacts on
surface. This disturbance is then carried to human health and the environment. It was
various regions with varied impacts. then that the Vienna Convention for the

PTS2023/PPP-01/092022/11 60

Telegram - @upscmaterialonline1
.
Protection of the Ozone Layer was created levels in 2022 are back to those observed in
in response. 1980 before ozone depletion was significant.
However, the pace of reduction in ODSs
 The Vienna Convention was the first
over Antarctica, which experiences a large
convention of any kind to be signed by every
ozone hole in spring, has been slower.
country involved, taking effect in 1988.
 This slow but steady progress over the past
 The Convention did not require countries
three decades was achieved by international
to take concrete actions to control ozone-
compliance with controls on production and
depleting substances. Instead, a provision
trade of ODS in the Montreal Protocol on
was added that Protocols to control these
Substances that Deplete the Ozone Layer —
substances would be adopted if and when an international agreement made in 1987.
warranted.
 Two years after the adoption of the Vienna
Convention, the Montreal Protocol on
92. Correct Option: (b)
Substances that Deplete the Ozone Layer Explanation:
was agreed upon, in an effort to apply
limits to the production and consumption of  Statement 1 is incorrect: Anang Tal is
the main chemicals that were causing the situated “to the north of Jog Maya temple
destruction of the Earth’s protective ozone and approximately 500 metres to the

RE
layer. The Protocol, which entered into force northwest of Qutub Complex”, and dates
in 1989, defines a schedule for reducing or back to 1,060 AD. Tradition ascribes this
eliminating the use of these chemicals. tank to a Tomar King, Anangpal II, the
builder of Lal Kot. It is said that Alauddin
 Since adoption of the Montreal Protocol, the Khalji, in 1296-1316 AD, utilised the water
Parties also adopted several amendments of this tank when he built Qutub minar and
to that Protocol: the London Amendment extended the Qutub-ul-Islam mosque.
(1990), the Copenhagen Amendment (1992),
the Montreal Amendment (1997), the

Amendment (2016).
O
Beijing Amendment (1999), and the Kigali
Supplementary notes:
Anang Tal Lake
 The lake is situated in Mehrauli, Delhi and
 The Kigali Amendment, called for the phase- is claimed to be created by Tomar King,
SC
down of hydro-fluorocarbons (HFCs) in 2016. Anangpal II, in 1,060 AD.
These HFCs were used as replacements
for a batch of ozone-depleting substances  He is known to have established and
eliminated by the original Montreal populated Delhi in the 11th century.
Protocol. Although they do not deplete the  The millennium old Anang Tal signifies the
ozone layer, they are known to be powerful beginning of Delhi.
greenhouse gases and, thus, contributors to
climate change.  Anang Tal has a strong Rajasthan
GS

connection as Maharaja Anangpal is known


 In 2009, the Convention and the Montreal as nana (maternal grandfather) of Prithviraj
Protocol became the first treaties of any Chauhan whose fort Rai Pithora is on the
kind to achieve universal ratification. list of the ASI.
Why in News?  Anang Tal is situated “to the north of Jog
Maya temple and approximately 500 metres
 The scientists of the National Oceanic and
to the northwest of Qutub Complex”, and
Atmospheric Administration (NOAA), an
dates back to 1,060 AD.
American scientific and regulatory agency
within the United States Department of  It is said to have been a place of a general
Commerce, recently announced that the resort but now it is dried up and used for
concentration of ozone-depleting substances cultivation.
in the atmosphere has reduced to reach a
 It is also said Alauddin Khalji, in 1296-1316
significant milestone this year.
AD, utilised the water of this tank when he
 NOAA’s Ozone Depleting Gas Index tracks built Qutub minar and extended the Qutub-
the overall stratospheric concentration ul-Islam mosque.
of ozone-depleting chlorine and bromine
Why in News?
from long-lived ozone-depleting substances
(ODS) relative to its peak concentration in  The Ministry of Culture recently declared
the early 1990s. Anang Tal Lake as a monument of national
importance.
 They found the overall concentration of ODSs
in the mid-latitude stratosphere had fallen  In exercise of the powers conferred by
just over 50 per cent. This means the ODS sub-section (1) of section 4 of the Ancient

61
PTS2023/PPP-01/092022/11

Telegram - @upscmaterialonline1
.
Monuments and Archaeological Sites and any existing state health insurance schemes
Remains Act, 1958, the central government as well, with the National Health Authority
declared the said ancient site and remains (NHA) recently allowing co-branding with
to be of national importance. logos and names of both schemes on the new
cards.

93. Correct Option: (c)  At least 31 states and union territories have
already agreed to the scheme, with final
Explanation: concurrence awaited from Tamil Nadu and
 Statement 2 is incorrect: At least 31 states Telangana. Delhi, West Bengal, and Odisha
and union territories have already agreed to have stayed away from the scheme.
the scheme, with final concurrence awaited  The same card can be used by beneficiaries
from Tamil Nadu and Telangana. Delhi, under Ayushman Bharat and the state
West Bengal, and Odisha have stayed away scheme. If a state wants to provide more
from the scheme. than Rs 5 lakh benefit, partial funding for
upto Rs 5 lakh will be provided by the Centre
 Statement 3 is incorrect: The beneficiary
as per the existing scheme. If states have
card for Ayushman Bharat-–Pradhan
additional beneficiaries (other than those
Mantri Jan Arogya Yojana (AB-PMJAY)
identified under the 2011 socio-economic
can now be used to avail benefits for any

RE
and caste census), they will be also be able
existing state health insurance schemes as
to use the same card.
well, with the National Health Authority
(NHA) recently allowing co-branding with  The co-branded cards will allot space to
logos and names of both schemes on the new both PM-JAY ad state specific logos. At
cards. the bottom of the card, the name of the
scheme will be mentioned as “Ayushman
Supplementary notes: Bharat Pradhan Mantri Jan Arogya Yojna-
Ayushman Bharat - Pradhan Mantri Jan
O Mukyamantri Yojna (or the name of the
state scheme).”
Arogya Yojana (AB-PMJAY)
 Under this scheme, around 10.74 crore poor
and vulnerable families identified as per 94. Correct Option: (d)
SC
Socio-Economic Caste Census are entitled
for health cover of Rs. 5.00 lakh per family
Explanation:
per year for secondary and tertiary care  All statements are correct
hospitalization.
Supplementary notes:
 PMJAY provides cashless and paperless
access to services for the beneficiary at the Mandala Art
point of service.  Mandala patterns are motifs that are
GS

 It is a Centrally Sponsored Scheme and the centuries old, and are used to depict the
nodal agency to implement it is the National cosmos. They have been adapted by artists
Health Authority (NHA). the world over, each of whom has added
their own interpretation to these designs.
 It covers up to 3 days of pre-hospitalization
and 15 days of post-hospitalization expenses  Literally meaning the “circle” or the
such as diagnostics and medicines. “centre” in Sanskrit, mandala is defined
by a geometric configuration that usually
 There is no restriction on the family size, incorporates the circular shape in some
age or gender. form. While it can also be created in the
 PM-JAY was earlier known as the National shape of a square, a mandala pattern is
Health Protection Scheme (NHPS) before essentially interconnected.
being rechristened. It subsumed the then-  Mandala is rooted in Hinduism and
existing Rashtriya Swasthya Bima Yojana Buddhism. Mandala imagery first appeared
(RSBY) which had been launched in 2008. in the Vedas (c. 1500-500 BC), and
The coverage under PM-JAY, therefore, Buddhist missionaries travelling along the
also includes families that were covered in Silk Road are believed to have taken it to
RSBY but are not present in the SECC 2011 regions outside India. By the sixth century,
database. mandalas were recorded in China, Korea,
Japan, Indonesia and Tibet. Separately,
Why in News?
native American peoples are believed to
 The beneficiary card for Ayushman Bharat- have used the mandala as representation
–Pradhan Mantri Jan Arogya Yojana (AB- of a deity or the cosmos, and as a spiritual
PMJAY) can now be used to avail benefits for form.

PTS2023/PPP-01/092022/11 62

Telegram - @upscmaterialonline1
.
 It is believed that by entering the mandala  The revolt by peasants was not related to
and moving towards its centre, one the freedom movement, though, but against
experiences a cosmic process of transforming the high land revenue (tax) assessment and
the universe and that of moving from the absence of a lucrative market for farm
emotions of suffering to the feeling of joy. produce. But it did motivate people to join
the freedom movement after 1850.
 In Hindu philosophical systems, a mandala
or yantra is usually in the shape of a square  The uprisings, which began in Bellare in
with a circle at its centre. March, 1837, spread to Sullia, Puttur,
Bantwal, Kumble, Kasaragod, and finally to
 A traditional Buddhist mandala is a circular
Mangalore (now Mangaluru). The treasury
painting that is meant to help its creator
of Britishers was present at Bellare of
discover their true self.
Sulya. Freedom fighters came directly from
 There are various elements incorporated Maduvegadde to Bellare Bungalow and
within the mandala, each of which has seized the treasury.
its own meaning. For instance, the eight
 The revolutionaries lowered the British
spokes of the wheel (the Dharmachakra)
flag at Bavutagudda, now in the heart of
represent the eight-fold path of Buddhism
Mangaluru, on April 5, 1837, and hoisted
that brings liberation from the cycle of birth
their own flag.
and death. The lotus flower depicts balance,

RE
and the sun represents the universe. Facing  The peasants had captured Mangalore for
up, triangles represent action and energy, 13 days. Kedambady Ramayya Gowda is
and facing down, they represent creativity said to be one among those who revolted
and knowledge. against the British then.
 Deep-rooted in ancient philosophy, the  Gowda had formed an army of farmers to
mandala has attained varied forms in the fight against the British through Amara
hands of modern and contemporary Indian
O Sullia Rebellion
artists.
 Ubaradka Mittur village in Sullia taluk
 While it continues to appear in Buddhist has been designated as a Heritage Village
Thangka paintings, it has a central place by the Government of Karnataka, for its
in the works of even mainstream artists contribution to freedom movement.
associated with tantric and neo-tantric
SC
spiritual movements. Why in News?
 A bronze statue of Kedambady Ramayya
Why in News?
Gowda, mounted on a horse, who is said
 A huge mandala has recently been carved to have revolted against the British in a
into parkland to show people the realms of mass insurrection in 1837, will recently be
what art can be. brought to Dakshina Kannada for installing
it at Bavutagudda or Light House Hill in
 The artwork, which covers an area the
the city.
GS

size of one-and-a-half football pitches, has


been cut into Knowsley’s Halewood Park in
Liverpool in the United Kingdom by artist 96. Correct Option: (c)
James Brunt.
Explanation:
95. Correct Option: (a)  Both statements are correct

Explanation: Supplementary notes:

 Option (a) is correct Narmada Canal

Supplementary notes:  The Narmada Canal is a contour canal in


Western India that brings water from the
Amara Sullia Rebellion Sardar Sarovar Dam to the state of Gujarat
and then into Rajasthan state.
 The Amara Sullia Rebellion was an armed
uprising against the British government  It is the largest lined irrigation canal in the
organized by the people of Arebhashe and world.
Tulunadu that took place in 1837, twenty
 The main canal has a length of 532
years before the Sepoy Mutiny of 1857.
kilometers (331 mi) (458 kilometers (285 mi)
 Peasant uprisings, called ‘koots’ by the in Gujarat and then 74 kilometers (46 mi) in
British, which broke out against the East Rajasthan). It is the second longest canal in
India Company in the coastal belt during India (after the Indira Gandhi Canal) and
1830-31, reached the stage of an armed the largest canal by water carrying capacity
struggle in 1837. (40,000 cusec at source).

63
PTS2023/PPP-01/092022/11

Telegram - @upscmaterialonline1
.
 The Main Canal is lined with plain cement  Historically both the People’s Republic of
concrete to minimize seepage losses to China (PRC) and the Republic of China
attain higher velocity and to control the (ROC) on Taiwan espoused a One-China
water logging in future. The lining work is Policy that considered the strait part of
carried out with the mechanized pavers. the exclusive economic zone of a single
“China”. In practice, a maritime border of
 The Narmada canal will bring water to
control exists along the median line down
parched Kutch from the Narmada Sarovar
the strait.
dam in south Gujarat.
 The median line was defined in 1955 and the
 The Narmada, the largest west flowing river
US pressured both sides into entering into
of the Peninsula, rises near Amarkantak
a tacit agreement not to cross the median
range of mountains in Madhya Pradesh.
line.
It is the fifth largest river in the country
and the largest one in Gujarat. It traverses  The median line is also known as the Davis
Madhya Pradesh, Maharashtra and Gujarat line after US Air Force General Benjamin O.
and meets the Gulf of Cambay. The total Davis Jr. who drew a line down the middle
length of the river from source to sea is 1312 of the strait.
kilometers.
Why in News?

RE
 PM Modi recently inaugurated Narmada
canal in Gujarat.

97. Correct Option: (b)


Explanation:
 Option (b) is correct
O
Supplementary notes:
Taiwan Strait
SC
 The Taiwan Strait is a 180-kilometer wide Why in News?
strait separating the island of Taiwan and
continental Asia.  India has for the first time referred to what
it called “the militarisation of the Taiwan
 The strait is part of the South China Sea Strait”, marking a rare instance of New
and connects to the East China Sea to the Delhi appearing to comment on China’s
north. actions towards Taiwan.
GS

 South China Sea is a marginal sea, an arm  The reference to Taiwan, in a recent
of western Pacific Ocean in Southeast Asia. statement issued by the Indian High
 The People’s Republic of China, the Commission in Sri Lanka, marked a more
Republic of China (Taiwan), the Philippines, pronounced expression of New Delhi’s views
Malaysia, Brunei, Indonesia, Singapore and over the situation in the Taiwan Strait than
Vietnam border the South China Sea. its previous response to China’s military
drills, conducted in the wake of the visit by
 Taiwan Strait connects South China Sea U.S. House Speaker Nancy Pelosi.
with the East China Sea while the Luzon
Strait connects it with the Philippine Sea.  Earlier, the Ministry of External Affairs
(MEA) did not refer to “militarisation” of the
 The Taiwan Strait is itself a subject of Strait, only saying India was “concerned at
international dispute over its political recent developments” and “urged exercise of
status. As the People’s Republic of China restraint, avoidance of unilateral actions to
claims to enjoy “sovereignty, sovereign change status quo, de-escalation of tensions
rights and jurisdiction over the Taiwan and efforts to maintain peace and stability
Strait” and regards the waterway as in the region.”
“internal territorial waters” instead of
 The latest reference to Taiwan came amid
being international waters, this means that
a spat with China over the recent visit of
the Chinese government denies any foreign
a Chinese military tracking vessel Yuan
vessel having the freedom of navigation in
Wang 5 to Hambantota in Sri Lanka.
the strait. This position has drawn strong
objections from the United States, Australia,  India has followed a “One China policy”
France and Taiwan. since its recognition of the PRC in 1949, and

PTS2023/PPP-01/092022/11 64

Telegram - @upscmaterialonline1
.
only maintains trade and cultural relations  The service providers shall prepare
with Taiwan. India routinely reiterated this a comprehensive waste management
policy until 2008, after which it stopped plan for waste generated within their
mentioning it in official statements, a jurisdiction, within six months from
demand that China usually asks of most the date of notification of these rules,
countries in official declarations.
 Shall remove all construction and
demolition waste in consultation with
98. Correct Option: (b) the concerned local authority on their
own or through any agency
Explanation:
 Duties of State Government and
 Statement 1 is incorrect: As per the Local Authorities:
rules, the Bureau of Indian Standards need
 Shall procure and utilize 10-20%
to prepare code of practices and standards
materials made from construction
for products of construction and demolition
and demolition waste in municipal
waste
and Government contracts.
Supplementary notes:  Local Authority shall place
Construction and Demolition Waste appropriate containers for collection
of waste, removal at regular intervals,

RE
Management Rules, 2016
transportation to appropriate sites
 These rules, notified for the first time in for processing and disposal.
2016, aims to effectively tackle the issues of
 LA shall seek detailed plan or
pollution and waste management.
undertaking from large generator of
 The rules consider construction & demolition construction and demolition waste
waste not as a waste, but a resource and and sanction the waste management
thus aims to recover, recycle and reuse the plan


waste generated through construction and
demolition.
O
The salient features of the Construction &
 Seek assistance from concerned
authorities for safe disposal of
construction and demolition waste
Demolition Waste Management Rules, 2016 contaminated with industrial
SC
are: hazardous or toxic material or nuclear
waste if any;
 The rules applies to everyone who
generates construction and demolition  LA shall establish a data base and
waste. update once in a year,
 Duties of waste Generators:  Duties of Central Pollution Control
Board, State Pollution Control Board
 Every waste generator shall segregate or Pollution Control Committee:
GS

construction and demolition waste


and deposit at collection centre  The Central Pollution Control
or handover it to the authorized Board shall prepare operational
processing facilities guidelines related to environmental
management of construction and
 Shall ensure that there is no littering demolition waste.
or deposition so as to prevent
obstruction to the traffic or the public  SPCB shall grant authorization to
or drains. construction and demolition waste
processing facility
 Large generators (who generate more
than 20 tons or more in one day or  Monitor the implementation of these
300 tons per project in a month) rules by the concerned local bodies
shall submit waste management  Standards for products of
plan and get appropriate approvals construction and demolition waste:
from the local authority before
starting construction or demolition  The Bureau of Indian Standards
or remodeling work need to prepare code of practices and
standards for products of construction
 Large generators shall pay relevant and demolition waste
charges for collection, transportation,
 Indian Roads Congress need to
processing and disposal as notified by
prepare standards and practices
the concerned authorities
pertaining to products of construction
 Duties of Service providers and and demolition waste in roads
Contractors: construction

65
PTS2023/PPP-01/092022/11

Telegram - @upscmaterialonline1
.
 Facility for processing / recycling  The temple is located five miles from
facility: Anantnag in the Indian union territory
of Jammu and Kashmir.
 The operator of the facility shall
obtain authorization from State  From the ruins and related archaeological
Pollution Control Board or Pollution findings, it can be said it was an excellent
Control Committee. specimen of Kashmiri architecture, which
 The processing / recycling site shall be had blended the Gandharan, Gupta and
away from habitation clusters, forest Chinese forms of architecture.
areas, water bodies, monuments,  The temple appears in the list of centrally
National Parks, Wetlands and places protected monuments as Kartanda (Sun
of important cultural, historical or Temple).
religious interest.
Why in News?
 The processing/recycling facility
exceeding five Tones per day capacity,  Archaeological Survey of India (ASI) red-
shall maintain a buffer zone of no flagged a religious ceremony participated by
development around the facility. the Lieutenant-Governor Manoj Sinha.

Why in News?  Recently, a group of Hindu pilgrims entered


its premises and held an hour-long prayer

RE
 The Super tech Twin towers in Sector 93A session. The site is classified as a ‘non-
of Noida, Uttar Pradesh were demolished living’ protected monument, where no ritual
recently by controlled implosion. The can be held unless permitted.
operation was carried out by Edifice
Engineering based in Mumbai and South
African company Jet Demolition. The 100. Correct Option: (a)
demolition was done by a method known
as the ‘waterfall technique’ to minimize Explanation:
damage to the surroundings.
O  Option (a) is correct
 Their being located in a residential
Supplementary notes:
neighborhood of Noida makes it even
more essential to introduce interventions Johar
SC
to mitigate pollution and waste, post-
demolition.  ‘Johar’, which essentially means ‘salutation
and welcome’, is used within the tribal
 The concerned authorities had given an communities of Jharkhand, and in parts of
NOC for the demolition to go ahead and post Chhattisgarh and Odisha.
the demolition; the standard protocols for
disposal of debris will be followed according  According to several tribal leaders from
to The Construction and Demolition Waste Jharkhand, the word ‘Johar’ also means
Management Rules, 2016. ‘paying respect’.
GS

 Tribal communities are nature worshippers


and follow Sarna religion code, although it
99. Correct Option: (b)
is not an official religion.
Explanation:  There are 32 tribal communities in
 Option (b) is correct: Martand Sun Jharkhand who speak different dialects.
Temple is located in Anantnag, Jammu and Almost all, including tribal Christians, use
Kashmir. the word ‘Johar’ along with some other words
for salutation. Johar, is predominantly used
Supplementary notes: by Santhali, Munda and Ho communities
Martand Sun Temple that share some similarities. People
belonging to the Oraon community use the
 The Martand Sun Temple also known word ‘Jai Dharam’, apart from Johar, as a
as Pandou Laidan is a Hindu temple salutation.
dedicated to Surya (the chief solar deity in
Hinduism) and built during the 8th century  There are at least four types of Johar
CE. Martand is another Sanskrit synonym greeting. One of them is ‘Doboh Johar’,
for Surya. which is used between people where one
among them has a higher standing. In
 It was built by the third ruler of the Karkota
‘Doboh Johar’ there is a ritual where the
Dynasty, Lalitaditya Muktapida.
person with a tumbler full of water bows in
 It is now in ruins, as it was destroyed by front of the person of higher standing. “The
the orders of Muslim ruler Sikandar Shah person who bows will touch the earth and
Miri. in return the other person will wash his/

PTS2023/PPP-01/092022/11 66

Telegram - @upscmaterialonline1
.
her hand (using water in the tumbler) and ‘time immemorial’, adding that it is hard to
let the water drop on the earth. The entire pinpoint when it actually began.
exercise means that the hospitality being
shown has been accepted.” Why in News?

 Leaders from the tribal communities say  The 15th President of India, Droupadi
that the greeting has been in use since Murmu, assumed office recently with a
‘Johar’ greeting to the country.

™™™™™

RE
O
SC
GS

67
PTS2023/PPP-01/092022/11

Telegram - @upscmaterialonline1

You might also like